Sie sind auf Seite 1von 139

https://t.me/pdf4exams https://t.

me/allupscmaterials

A. Polity 1- 19 C. ECONOMY 26-41


1. Government Initiatives /Acts / 1. Economic development 26
Policies 1 1.1 Joblessness rises to 3year high 26
1.1 LGs take charge in J&K, Ladakh 1 1.2 Real Estate reforms 27
1.2 CJI under RTI Act 4 1.3 Moody’s Investors Service 28
1.3 Registration mandatory for non- 1.4 Retail inflation rise 29
resident visitors to Meghalaya 5 1.5 Arcelor Mittal’s takeover of Essar
1.4 Citizenship (Amendment) Bill 6 cleared by SC 30

2. Executive and legislative 7 2. Industries 31

2.1 President’s Rule 7 2.1 Core sector output falls 5.2% in


Sept 31
2.2 Disqualification of MLAs 8
2.2 Index of Industrial Production
3. Judiciary 9
(IIP) 32
3.1 Ayodhya Verdict 9
2.3 Industrial Relations Code Bill,
3.2 Finance Act 2017 11
2019. 33
3.3 Doctrine of Essentiality 12
3. Infrastructure 34
4. Union and States 13
3.1 ` 25,000-cr. fund to help housing
4.1 Destination North East Festival13
sector 34
4.2 Over the states 14
3.2 Road Accidents in India-2018 35
5. Governance 15
4. Agriculture 36
5.1 Justice delivery 15
4.1 Seed treaty 36
5.2 Social Audit 16
4.2 Zero Budget Natural Farming 37
6. In-News 17
4.3 Skill development programmes
6.1 State of Policing in India 17
for Farmers and Women 38
6.2 Quid pro quo 19
5. Government initiatives 38
6.3 Tourist arrivals in India 19
5.1 Three years of DeMO 38
B. Social Issues 21-25 5.2 Sabka Vishwas – Legacy Dispute
1. Women and Child 21 Resolution Scheme 40
1.1 Maternal death rate 21 6. In-News 41
1.2 National Child Labor Project 22 6.1 Harmonised system of Code 41
2. Education 22 D. ENVIRONMENT 42-61
2.1 Quality Education in schools 22 1. Ecosystem/Biodiversity 42
2.2 NISHITHA programme 23 1.1 Sunder bans mangroves 42
3. In News 23 1.2 Butterfly survey in western
3.1 Draft Social Security code 23 Ghats 43
3.2 Maintenance and Welfare of 1.3 Avian Botulism 44
Parents and Senior Citizens 1.4 Pliosaur 44
Amendment Bill 24 1.5 Idris Elba 45
3.3 Hunar Haat 25 1.6 Paris bans wild animals from
3.4 Aadi Mahotsav 25 circuses 45
Join Telegram groups
To Boost Your Preparation
PDF4Exams One stop solution for study
Click Here materials of all competitive exams

The Hindu Zone Official


Newspapers & study Click Here
materials

TestSeries4Exam Aall paid test series


Click Here availabble without any cost

All e-Magazines
Estore
in your hand Click Here
Hindi Books
All study materials
Click Here in Hindi

eSandesh (An Indian App)

For More download eSandesh App from play store


https://t.me/pdf4exams https://t.me/allupscmaterials

2. Pollution 45 3. International Events 74


2.1 Delhi Pollution 45 3.1 Iran announces latest nuclear
2.2 Health emergency declared in deal violations 74
the Capital 48 3.2 Kalapani dispute 75
2.3 ICAR report on Stubble Burning 3.3 Towards a Colombo reset 76
49 F. SCIENCE AND TECHNOLOGY 78-94
2.4 Indian Air Quality Interactive 1. Health/Nutrition/Diseases 78
Respository (IndAIR) 51 1.1 Sex ratio improves in country 78
3. Geographical Phenomenon 52 1.2 Indian lungs under extreme
3.1 Cyclone Maha batters stress 78
Lakshadweep and Kerala coast 52
1.3 Anaemia 79
3.2 Cyclone ‘Bulbul’ 52
1.4 Pneumonia, diarrhoea still a big
3.3 DANKALI DEPRESSION 53
threat 80
3.4 Acqua alta 54
1.5 Parlimentary Panel Report of
4. Government Initiatives 54
Cancer Patients 82
4.1 Cloud seeding 54
1.6 India is home to 77 million
4.2 Water policy 56
diabetics 83
4.3 Swachh – Nirmal Tat Abhiyan 56
1.7 National Health Profile 2019 84
4.4 Red Atlas Action Plan Map 57
1.8 National Health Stack (NHS) and
4.5 Geochemical Baseline Atlas of
National Digital Health Blueprint
India. 58
84
4.6 Indian Forest Act 2019 59
1.9 Jansankhya Sthirata Kosh 85
5. In-News 60
1.10 Mental Health Awareness
5.1 ETHANOL PRODUCTION IN INDIA
60 Initiatives 86

5.2 The Lancet Countdown on Health 2. Space 87

and Climate Change 61 2.1 Navigation in Indian


E. INTERNATIONAL RELATIONS 62-77 Constellation 87
1. Bilateral Relations 62 2.2 Dwarf planets 88
1.1 India- Uzbekistan 62 2.3 Spicule in sun 88
1.2 India-Russia 64 2.4 Arrokoth 89
1.3 H-1B visa 64 2.5 Space Internet 90
1.4 Tiger Triumph 65 3. Technology 91
1.5 U.S. trade negotiators to visit 3.1 Govt. questions WhatsApp after
Delhi for more talks 65 spyware is used for snooping 91
2. International Institutions 66 3.2 Economic slowdown may lighten
2.1 ICJ 66 India’s carbon burden 91
2.2 RCEP 67 3.3 Internet penetration 92
2.3 BRICS 70 4. In news 93
2.4 BIMSTEC Ports 72 4.1 NuGen Mobility Summit 93
2.5 International Criminal Court 72 4.2 Global Bio-India Summit 2019 94
https://t.me/pdf4exams https://t.me/allupscmaterials

G. HISTORY 95-100
1. In-news 95
1.1 Berlin Wall 95
1.2 Suranga Bawadi 96
1.3 Lutyens Delhi 97
1.4 Karim Shahi region 98
1.5 Thiruvalluvar 99
1.6 Lala Lajpat Rai 99
1.7 National Mission of Cultural
Mapping 100
H. TIT-BITS 101-105
1. Hyderabad figures in Unesco list
101
2. World Tsunami Awareness Day
101
3. Vigyan Samagam 101
4. Chinese experts to help in
restoration of Notre-Dame 102
5. Mother Tongue 102
6. MK 45 Gun System 104
I. Yojana Essence 106-107
J. Editor’s Pick 108-119
K. Practice Prelims Questions 120-133

Disclaimer
The views and opinions in the magazine are those of the editors and do not necessarily reflect or represent the views and opinions by the
LAEX.
LAEX doesn’t claim copyright to the said content. Images are taken by various newspapers and website like The Hindu, Indian Express,
Live Mint, Economic times etc.
The editor has put best efforts in preparing and arranging this Monthly Magazine. This magazine is solely designed to provide helpful
information on the current topics to help UPSC aspirants in any possible manner.
For any suggestions or corrections please email us: blr@laex.in
–R&D
https://t.me/pdf4exams https://t.me/allupscmaterials

A. Polity
1. Government Initiatives/Acts/Policies
1.1 Union Territory of J & K and Ladakh
The Indian Union now has 28 States and nine UTs
Legislative powers of the Union Territory of Jammu and Kashmir:
 The Legislative Assembly may make laws for the whole or any part of the Union Territory of Jammu and
Kashmir with respect to any of the matters enumerated in the state list except on subjects “public
order” and “police” which will remain in the domain of the Centre vis-a-vis the LG.
 In case of inconsistencies between laws made by Parliament and laws made by the Legislative
Assembly, earlier law shall prevail and law made by the Legislative Assembly shall be void.
 The role of the Chief Minister will be to communicate to the L-G all decisions of the Council of Ministers
relating to the administration of affairs of the Union Territory and proposals for legislation and to
furnish such information relating to the administration of affairs as the L-G may call for.
Role and powers of the Lieutenant Governor of Jammu and Kashmir (J&K) and Ladakh:
 In the case of Union Territory of Jammu and Kashmir, the L-G shall “act in his discretion” on issues
which fall outside the purview of powers conferred on the Legislative Assembly, in which he is
required to exercise any judicial functions, and/or matters related to All India services and the Anti-
Corruption Bureau
 The Chief Minister shall be appointed by the L-G who will also appoint other ministers with the aid of
the CM. The L-G shall also administer the oath of office and of secrecy to ministers and the CM.
 The L-G will have the power to promulgate ordinances which shall have the same force and effect as
an act of the Legislative Assembly assented by the L-G.
 Role of L-G in Ladakh: The President shall appoint the L-G under Article 239 of the Indian
constitution. The L-G will be assisted by advisors appointed by the Centre since the Union Territory
will not have a Legislative Assembly.
Both Lieutenant Governors of J&K and Ladakh were appointed after Article 370 was abrogated from our
Indian Constitution.
What was Article 370?
 Article 370 of the Indian constitution gave special status to Jammu and Kashmir. The special status
conferred certain powers to the state such as- separate constitution, a state flag and autonomy over the
internal administration of the state.
 The Constituent Assembly of Jammu and Kashmir, after its establishment, was empowered to
recommend the articles of the Indian constitution that should be applied to the state or to abrogate the
Article 370 altogether. After consultation with the state's Constituent Assembly, the 1954 Presidential
Order was issued, specifying the articles of the Indian constitution that applied to the state. Since the
Constituent Assembly dissolved itself without recommending the abrogation of Article 370, the article
was deemed to have become a permanent feature of the Indian Constitution
 This article, along with Article 35A, defined that the Jammu and Kashmir state's residents live under a
separate set of laws, including those related to citizenship, ownership of property, and fundamental
rights, as compared to residents of other Indian states
 On 5 August 2019, the Government of India issued a constitutional order superseding the 1954 order,
and making all the provisions of the Indian constitution applicable to Jammu and Kashmir based on the
resolution passed in both houses of India's parliament.
 In addition, the Jammu and Kashmir Re-organisation Act was passed by the parliament, enacting the
division the state of Jammu and Kashmir into two union territories to be called Union Territory of
Jammu and Kashmir and Union Territory of Ladakh

La Excellence IAS 1 Website: www.laex.in/testprep


9052 29 29 29 / 9052 49 29 29 (Hyderabad)
9121 41 29 29 / 9121 44 29 29 (Bangalore)
https://t.me/pdf4exams https://t.me/allupscmaterials

Source: The Hindu


TIMELINE of ARTICLE 370 abrogation
What does the abrogation mean?

State UT

Special Powers Exercised by J&K No special power such as separate constitution


Dual Citizenship Single Citizenship
Separate flag for J&K Tricolor will be the only flag
Article 360 (Financial Emergency) not applicable Article 360 is applicable

No reservation for minorities such as Hindu’s and Now 16% reservation is applicable
Sikhs.
Indian Citizens from other states cannot buy land Can buy land or property
or property in J&K
RTI not Applicable RTI now applicable
Assembly duration-6 years Assembly duration of UT of J&K-5 years
Panchayats did not have any rights It has same rights as other states
Right To Education not applicable RTE now Applicable

La Excellence IAS 2 Website: www.laex.in/testprep


9052 29 29 29 / 9052 49 29 29 (Hyderabad)
9121 41 29 29 / 9121 44 29 29 (Bangalore)
https://t.me/pdf4exams https://t.me/allupscmaterials

Is REVOCATION OF ARTICLE 370 A RIGHT STEP?


The essence of Article 370 was that it was only a temporary, transitional arrangement and was never intended
to be a permanent provision.
1. Article 370 never served its purpose: History shows that instead of bringing people of Kashmir closer to
the rest of India, Article 370 has only widened the chasm. While Article 370 has failed to benefit the
people in a meaningful way, it was used by separatists to drive a wedge between those living in J&K and
the rest of India.
2. Equal rights to Kashmiri women: With Article 35A becoming void, the decades old discrimination against
the women of J&K has been eliminated. They can now purchase and transfer property to their children
and can get married to a non-resident of J&K.
3. Unity and integrity of India: The abrogation of Article 370 is indeed a step in the right direction to
safeguard the unity and integrity of India.
4. Opens to economic vistas: The Government’s decision would facilitate greater investments by both
individual entrepreneurs and major private companies in different sectors including hospitality, tourism,
education and health.
CRITICAL ANALYSIS OF THE MOVE TO REVOKE ARTICLE 370
 Question of legality: Under the existing Article 370(3), it had been provided that the President could
abrogate or amend Article 370 on the recommendation of the Constituent Assembly of the State
which had ceased to exist in 1957.
 Government substitutes the expression “Constituent Assembly of the State” with the expression
“Legislative Assembly of the State” in provison to Article 370(3).
 Attack on federal structure: The revocation of Article 370 militates against the idea of federalism,
which is a basic feature of the Indian Constitution. The move has undermined and weakened India’s
federal character by downgrading a State and territorially dividing it into two Union Territories
without the consent of the people of J&K. The moral authority of regional parties to safeguard India’s
federal system has been undermined.
In conclusion, it should be noted that the abrogation of Article 370 is a national issue involving our country’s
safety, security, unity and equitable prosperity. It is a step in the right direction that the Indian Parliament
has taken with an overwhelming majority. It is a step that opens up new vistas for the all-round
development in a State that was relatively neglected.
All said, Article 370 was a temporary provision and it had to go sooner rather than later but it would have
been better if it was done with the concurrence of Kashmiri people and within the provisions of
constitution.

Mains question
‘The abrogation of Article 370 has finally empowered those sections of Jammu and Kashmir society who
had remained marginalized since many decades’. Elucidate. (150W|10M)

La Excellence IAS 3 Website: www.laex.in/testprep


9052 29 29 29 / 9052 49 29 29 (Hyderabad)
9121 41 29 29 / 9121 44 29 29 (Bangalore)
https://t.me/pdf4exams https://t.me/allupscmaterials

1.2 CJI under RTI Act


Context:
In a welcome step, the Supreme Court of India’s verdict has brought the office of Chief Justice of India under
the ambit of Right to Information (RTI) act
Background on the case:
 A five-judge constitution bench headed by the CJI was hearing a petition filed against a 2010 judgment of
the Delhi High Court
 The high court in its order had declared the CJI's office a "public authority" and said that it should come
under the RTI Act.
 According to the Delhi High Court judgment all power including judicial power is subject to accountability
in a modern Constitution. A blanket judicial exemption from the RTI Act therefore would defeat the basic
idea of “open justice’’.
Why there was a need for transparency?
 India is one of the few countries where judges have the last word on judicial appointments, through the
mechanism of the Collegium.
 The Collegium itself is not mentioned in the text of the Constitution but it arose out of a judgment of the
Supreme Court, in response to increased executive interference in judicial appointments, particularly
during Indira Gandhi’s regime.
What is the collegium system of judicial appointments?
 The Collegium started as a tool to secure and guarantee the independence of the judiciary.
 The Collegium System is a system under which appointments/elevation of judges/lawyers to Supreme
Court and transfers of judges of High Courts and Apex Court are decided by a forum of the Chief Justice
of India and the four senior-most judges of the Supreme Court.'
 In 2015, the Supreme Court struck down a constitutional amendment establishing a National Judicial
Appointments Commission, which would have replaced the Collegium.
 A majority of the five-judge Bench held that judicial primacy in appointments was the only
constitutionally-authorized way of securing/ensuring judicial independence against an increasingly
powerful political executive.
 However appointments and transfers within the higher judiciary still remain a mystery. It is almost as
though the Supreme Court Collegium has many paths to opacity that can exist in the appointments
process. It is increasingly being perceived that judicial appointments have too often been made in an ad
hoc and arbitrary manner.

Significance of this verdict: All power, judicial power being no exception, should be held accountable in a
modern Constitution.
 A blanket judicial exemption from the RTI Act would defeat the basic idea of “open justice”: that the
workings of the courts, as powerful organs of state, have to be as transparent and open to public scrutiny
as any other body.
 Nor would bringing the judiciary under the RTI Act destroy the personal privacy of judges: as the High
Court judgment noted, the RTI Act itself has an inbuilt privacy-oriented protection, which authorizes
withholding the disclosure of personal information unless there is an overriding public interest.
 Courts have always been questioned for pending cases. RTI can place yardstick among judicial for timely
disposure of justice.
Concerns with the verdict delivered:
 Disclosing the correspondence of the judicial process might destroy judicial independence.
 It will challenge the decision-making power of Supreme Court.
 Creates extra burden on judiciary as every filed will be answerable by judiciary.

La Excellence IAS 4 Website: www.laex.in/testprep


9052 29 29 29 / 9052 49 29 29 (Hyderabad)
9121 41 29 29 / 9121 44 29 29 (Bangalore)
https://t.me/pdf4exams https://t.me/allupscmaterials

 It will compromise secrecy & security involved in certain cases. This may prove detrimental for our
country.
 Judiciary will become puppet in the hands of people rather than being the sole justice provider of the
country.
 It might increase political motivated files being filed on judiciary.
Observations/comments made by the court in this case
 The Supreme Court said that transparency does not undermine judicial freedom.
 The CJI office is a public authority. However, office secrecy will remain intact during RTI.
 The CJI-led bench said that no one wants a 'system of darkness', but the judiciary cannot be destroyed
in the name of transparency.

How it all started?


In 2007, RTI activist Subhash Chandra Aggarwal filed an RTI and asked for details of the assets of the
judges. When his application was rejected, the matter reached the Central Information Commissioner
(CIC) and CIC asked for information.
After that, the matter was challenged in Delhi High court. The High court said that the office of CJI comes
under RTI but it was again challenged in Supreme Court. A hearing was held in the Supreme Court in April
2019 and the court had reserved the verdict.

Mains question
“All power — judicial power being no exception — is held accountable in a modern Constitution.”
Comment. (150W|15M)

1.3 Registration mandatory for non-resident visitors to Meghalaya


Context: In a bid to protect the interest of tribal citizens, the Meghalaya cabinet approved the amendment to
an act that seeks mandatory registration of outsiders for entering the State.
Highlights:
 The State cabinet approved the amended Meghalaya Residents, Safety and Security Act, 2016.
 According to an amendment to the Act, any person who is not a resident of Meghalaya and intend to stay
more than 24 hours in the State will have to furnish document to the government.
 It will come into force with immediate effect but it will be regularized in the next session of the State
Assembly.
Who should get registered?
 Any person who is not a resident of Meghalaya and intend to stay more than 24 hours in the State will
have to furnish document to the government.
 Employees of the Centre, State and District Councils are exempted from the purview of the Act.
 Any person, who willfully fails to furnish the information or provide false document, will be liable to be
punished under various sections of the Indian Penal Code.
Why it was done?
 There was an increasing demand to enhance vigil against influx of non-indigenous people in the hill
state, following the implementation of the National Registry of Citizens (NRC) in Assam released in
August.
Other features of the Meghalaya Residents Safety and Security Act (MRSSA) 2016
 The act aims to ensure the security of the tenants as well as the safety and security of the citizens of the
state.
 It provides for verification and regulation the tenants residing in rented houses in the state.
 It also establishes District Task Force and Facilitation Centres for effective enforcement of various laws for
the safety and security of the citizens.

La Excellence IAS 5 Website: www.laex.in/testprep


9052 29 29 29 / 9052 49 29 29 (Hyderabad)
9121 41 29 29 / 9121 44 29 29 (Bangalore)
https://t.me/pdf4exams https://t.me/allupscmaterials

1.4 Citizenship Amendment Bill


The government intends to introduce The Citizenship (Amendment) Bill in Parliament’s ongoing Winter
Session.
Description:
Citizenship (Amendment) Bill
 The Bill seeks to amend The Citizenship Act, 1955 to make Hindu, Sikh, Buddhist, Jain, Parsi, and Christian
illegal migrants from Afghanistan, Bangladesh, and Pakistan, eligible for citizenship of India.
 In other words, the Bill intends to make it easier for non-Muslim immigrants from India’s three Muslim-
majority neighbours to become citizens of India.
 Under The Citizenship Act, 1955, one of the requirements for citizenship by naturalization is that the
applicant must have resided in India during the last 12 months, as well as for 11 of the previous 14 years.
 The amendment relaxes the second requirement from 11 years to 6 years as a specific condition for
applicants belonging to these six religions, and the aforementioned three countries.
 Under The Citizenship Act, 1955, a person who is born in India, or has Indian parentage, or has resided in
India over a specified period of time, is eligible for Indian citizenship.
Illegal migrants :
 Illegal migrants cannot become Indian citizens. Under the Act, an illegal migrant is a foreigner who:
(i) enters the country without valid travel documents like a passport and visa, or (ii) enters with valid
documents, but stays beyond the permitted time period.
 Illegal migrants may be put in jail or deported under The Foreigners Act, 1946 and The Passport (Entry
into India) Act, 1920.
 However, in 2015 and 2016, the government exempted specified groups of illegal migrants from
provisions of the 1946 and 1920 Acts.
 They were Hindus, Sikhs, Buddhists, Jains, Parsis and Christians from Afghanistan, Bangladesh and
Pakistan, who reached India on or before December 31, 2014.
 This meant that these particular categories of illegal migrants would not be deported or jailed for being
in India without valid documents.
 The Citizenship (Amendment) Bill, 2016 was introduced in Parliament to amend The Citizenship Act,
1955, so that these people could be made eligible for citizenship of India.
Controversy:
 The fundamental criticism of the Bill has been that it specifically targets Muslims.
 Critics argue that it is violative of Article 14 of the Constitution, which guarantees the right to equality.
 The government, however, maintains that the Bill aims to grant citizenship to minorities who have faced
religious persecution in Muslim-majority foreign countries.
 In the NE states, the prospect of citizenship for massive numbers of illegal Bangladeshi migrants has
triggered deep anxieties, including fears of demographic change, loss of livelihood opportunities, and
erosion of the indigenous culture.

Mains question
‘Citizenship Amendment bill’ as proposed is opposed to every fundamental value and ethos on
which our constitution stands. Critically analyze. (150W|10M)

La Excellence IAS 6 Website: www.laex.in/testprep


9052 29 29 29 / 9052 49 29 29 (Hyderabad)
9121 41 29 29 / 9121 44 29 29 (Bangalore)
https://t.me/pdf4exams https://t.me/allupscmaterials

2. Executive and Legislative


2.1 President’s Rule
Context: Maharashtra was placed under President’s Rule following a recommendation from the Governor.
Highlights:
1. Situation had arisen in the state in which it was impossible to constitute or form a stable government in
the State, and the government could not be carried on in accordance with the provisions of the
Constitution.
2. The President issued a proclamation under Article 356(1) of the Constitution based on the governor report
What is Article 356 of the Indian constitution?
Under Article 356 of the Constitution of India, in the event that a state government is unable to function
according to constitutional provisions, the Central government can take direct control of the state machinery.
1. If the President, on receipt of report from the Governor of the State or otherwise, is satisfied that a
situation has arisen in which the government of the State cannot be carried on in accordance with the
provisions of this Constitution, the President may Proclaim
(a) Assume to himself all or any of the functions of the Government of the State and all or any of the
powers vested in or exercisable by the Governor or anybody or authority in the State other than
the Legislature of the State
(b) Declare that the powers of the Legislature of the State shall be exercisable by or under the
authority of Parliament
(c) Make such incidental and consequential provisions as appear to the president to be necessary or
desirable for giving effect to the objects of the Proclamation, including provisions for suspending
in whole or in part the operation of any provisions of this constitution.

Other Grounds for imposition of President Rule:


1. Article 355 entrusts the duty upon Union to protect the states against “external aggression” and
“internal disturbance” to ensure that the government of every State is carried on in accordance with
the provisions of Constitution.
2. Article 365 says that where any State has failed to comply with, or to give effect to, any directions
given in the exercise of the executive power of the Union under any of the provisions of this
Constitution, it shall be lawful for the President to hold that a situation has arisen in which the
Government of the State cannot be carried on in accordance with the provisions of this Constitution
In which specific cases has the President’s rule been imposed so far in our country?
 State Legislature is unable to elect a leader as Chief Minister
 Collapse of a Coalition due to disagreements, parting ways within the members
 Serious breakdown law and order
 Elections postponed due to some unavoidable reasons
 Loss of majority in the state assembly
Parliamentary approval procedure:
 Every Proclamation to impose President Rule shall be laid down before each house of Parliament and
must get approval in two months from date of issue.
 Provided that if at time of proclamation Lok Sabha (LS) has been dissolved or dissolution of LS takes place
in mean time (i.e. within two months from date of issue) then must get approval of RS within 2 months
but such proclamation shall cease to operate after 30 days from first sitting of LS after its reconstitution if
not get approval of new LS in 30 days.

La Excellence IAS 7 Website: www.laex.in/testprep


9052 29 29 29 / 9052 49 29 29 (Hyderabad)
9121 41 29 29 / 9121 44 29 29 (Bangalore)
https://t.me/pdf4exams https://t.me/allupscmaterials

Duration of President Rule:


 If approved by both houses of Parliament then President Rule shall continue for 6 months and it can be
renewed for maximum of 3 yrs by approval of Parliament after every 6 months.
 But if dissolution of LS takes place in that 6 months and resolution for renewal of President Rule is under
consideration then President Rule survives till 30days from first sitting of newly elected LS provided that
already approved by RS.
 Any of above resolution related to proclamation or renewal of emergency must be passed by both houses
of Parliament by Simple majority.
 Termination – The President Rule can be revoked by President any time he thinks fit before it ceases to
operate.
 44th Constitutional Amendment 1978 – It introduced some constraints to impose President Rule beyond
a period of 1 year and states that the President Rule cannot be imposed in any state beyond 1 year
unless:-
 A Proclamation of National Emergency is in operation, in the whole of India or, as the case may be, in
the whole or any part of the State, at the time of the passing of such resolution.
 The Election Commission certifies that the continuance of President Rule is necessary on account of
difficulties in holding general elections to the Legislative Assembly of the State concerned.
What are some of the criticisms against this provision?
1. Article 356 has been widely criticized for giving provisions for the party/coalition in the Centre to misuse
democratic powers for political gains.
2. In 1994, the Supreme Court delivered a landmark judgment in the SR Bommai vs Union of India case,
introducing certain guidelines to check the unwarranted intrusion of the central government and the
imposition of Article 356 for political gains.

Mains question
‘The death letter of Indian Constitution'. Critically analyze the statement with regards to President Rule
provisions in our constitution. (150W|10M)

2.2 Disqualification of MLAs


Context: Supreme Court upheld the disqualification of 17 Karnataka MLAs but allowed them to contest by-
polls.

Why were these MLAs from Karnataka disqualified? These MLAs had violated party whip; citing this
as a main reason, the Speaker of the Karnataka Assembly disqualified them and also barred them from
contesting elections for a specific amount of time. The MLAs had hoped that their resignation would
topple the state government and also help them to prevent themselves from being subjected to
punishment under Anti-defection law.
Observations made by the Supreme Court in this case:
1. The SC observed that under the 10th schedule of the Constitution, the speaker “does not have the power
to either indicate the period for which a person is disqualified, or bar someone from contesting elections".
2. The Supreme Court also noted that there is a growing trend where the Speaker of the assembly has acted
in a partisan manner.
3. The Supreme Court of India also said Article 361 and Article 164 (1B) of the constitution stipulate the
disqualification of a member under the Tenth schedule does not bar him from contesting elections.

La Excellence IAS 8 Website: www.laex.in/testprep


9052 29 29 29 / 9052 49 29 29 (Hyderabad)
9121 41 29 29 / 9121 44 29 29 (Bangalore)
https://t.me/pdf4exams https://t.me/allupscmaterials

4. While the apex court noted that there was a growing trend of speakers failing to uphold the duty of
neutrality, it also said that “horse trading and corrupt practices associated with defection and change of
loyalty for lure of office or wrong reasons have not abated".
Opinions expressed on the judgment of the Supreme Court of India in this case: Constitutional
experts opine that the court missed a chance to set a precedent on laws that govern defections at a time
when politicians—both state and national—frequently switch sides.
What are the criteria for disqualification of MLAs? According to the constitution, a person shall be
disqualified as Member of Legislative Assembly (MLA) or Member of Legislative Council (MLC) if:
 He holds any office of profit under the Government of India or a state or an office declared by a law of the
state,
 Any competent court declares any member to be of unsound mind,
 He is charge-sheeted, bankrupt or insolvent,
 He is not a citizen of India,
 Has voluntarily acquired the citizenship of a foreign state or is under any acknowledgement of allegiance
or adherence to a foreign state.
According to Schedule 10 (Anti-Defection Act), a person shall be disqualified as Member of
Legislative Assembly (MLA) or Member of Legislative Council (MLC) if:
 An elected member voluntarily gives up his membership of a political party.
 An elected member votes or abstains from voting in such House contrary to any direction issued by his
political party or anyone authorized to do so, without obtaining prior permission.
According to Representation of the People (RP) Act, 1951 a person shall be disqualified as Member
of Legislative Assembly (MLA) or Member of Legislative Council (MLC) if:
 One is found guilty of an illegal practice in relation to election.
 A person convicted of any offence and sentenced to imprisonment for varying terms under
Sections 8 (1) (2) and (3).
Office of Profit
 The term office of profit has not been defined in the Constitution.
 But, articles 102 (1) and 191 (1), which give effect to the concept of office of profit, prescribe restrictions
at the central and state level on lawmakers accepting government positions.
 These cases are decided by the President of India on the recommendation of the Election Commission

Mains question
Does the Anti-defection law serve any purpose? Critically comment. (150W|10M)

3. Judiciary
3.1 Ayodhya Verdict
Context:
Supreme Court ruled that the 2.77 acres of disputed land in Ayodhya will be handed over to a trust for the
construction of a Ram Mandir.
Highlights of the verdict:
 A 5-acre land will be allotted to the Sunni Waqf Board in an alternate site in Ayodhya.
 The central government to frame a scheme within three months to allocate land for the mosque and
forming the temple trust.

La Excellence IAS 9 Website: www.laex.in/testprep


9052 29 29 29 / 9052 49 29 29 (Hyderabad)
9121 41 29 29 / 9121 44 29 29 (Bangalore)
https://t.me/pdf4exams https://t.me/allupscmaterials

 SC agreed that, the deity- Ram Lalla Virajman has legal character, but not the Janmasthan (birthplace of
Ram).
 Relying on reports of the Archeological Survey of India that there is sufficient proof that the Babri Masjid
was not built on an empty land but on an underlying structure that is not Islamic.
Chronology of the Ayodha dispute
 1528: First Mughal Emperor Babar is believed to have constructed Babri Masjid
 It is the belief that Lord Ram was born 900,000 years ago in the Treta Yuga, in a room located under what
was the central dome of the Babri Masjid built by Babur.
 1885: Mahant Raghbir Das moves to Faizabad court seeking permission to construct a temple in the
vicinity of the Babri Masjid. The plea is declined.
 December 22-23, 1949: Idols of Lord Ram is mysteriously found inside the mosque
 1950: A plea was submitted to Faizabad court to worship these idols
 1959: Nirmohi Akhara files plea seeking possession of the disputed land.
 1961: Central Sunni Waqf Board, U.P., moves court for declaration of title of the disputed land and
removal of the idols inside the mosque.
 1986: Permission given for Hindus to worship the idols.
 1989: Allahabad high court takes the title dispute and orders status quo
 November 1989: The Rajiv Gandhi government allows Vishwa Hindu Parishad (VHP) to perform puja near
the disputed site.
 1992: Right-wing activists demolish Babri Masjid. Justice Liberhan Commission appointed to probe.
 1993:P.V. Narasimha Rao government acquires 67 acres of land adjoining the disputed site. The Supreme
Court upholds the acquisition in its Dr. Ismail Faruqui judgment.
 2002: Allahabad High Court commences hearing the title suits.
 2003:SC bans religious activity in the acquired lands
 2009: Liberhan Committee submits inquiry report.
 2010: High Court delivers a majority judgment for three-way partition of the disputed property among
Hindus, Muslims and Nirmohi Akhara.
 2011: Supreme Court stays the order of High Court.
 2017: Matter moved to the constitution bench of the Supreme Court on other issue rather than just
adjudicating on title issue.
 January 2019: Resumption of adjudication process on title issue case. Mediation is suggested first though
rather than judicial scrutiny
 August 16, 2019: Mediation committee led by former Supreme Court judge, Justice F.M.I. Kalifulla fails to
draw a consensus and court hearing commences.
 The Final judgment (November 9, 2019): Constitution Bench cleared the way for constructing Ram
Temple at the disputed site. It ordered the government to provide five-acre land to Muslims at a
prominent place in Ayodhya.
Significance of this judgment:
 This judgment has brought closure to a long-drawn out controversial issue in our country
 The fact that the case is over at last must come as great relief to all peace-loving people.

La Excellence IAS 10 Website: www.laex.in/testprep


9052 29 29 29 / 9052 49 29 29 (Hyderabad)
9121 41 29 29 / 9121 44 29 29 (Bangalore)
https://t.me/pdf4exams https://t.me/allupscmaterials

However, some concerns remain with the verdict. Such as:


1. The verdict seems to justify the demolition that was carried out in an illegal manner.
2. The court seems to have relied on evidentiary evidences which were only made possible because of
demolition
3. Some of the acknowledged rights of Muslims in this case have been neglected with. Example: Their
rights over inner courtyard.
4. Individuals who were responsible for demolition and ensuing violence are yet to be convicted and
brought to justice.

Mains question
The recent judgment in the Ayodha case has brought a closure to a long-standing festering wound which
had polarized the society since many decades. Critically analyze. (150W|10M)

3.2 Finance Act 2017


Context: The Supreme Court passed a significant judgment on the functioning of Tribunals in the Country. In
the process, it ruled on the validity of the Finance Act, 2017 and the Rules framed under the same.
Highlights:
 The Finance Act of 2017 aimed to alter the appointments to 19 key judicial tribunals, including the Central
Administrative Tribunal.
 SC held that the Tribunal, Appellate Tribunal and other Authorities (Qualifications, Experience and other
Conditions of Service of Members) Rules, 2017 suffered from “various infirmities”.
 SC interpreted that rules formulated by the Central Government under Section 184 of the Finance Act,
2017, being contrary to the parent enactment and the principles envisaged in the Constitution.

Takeaways from the judgment:


 One Nodal Ministry for all Tribunals - The Ministry of Law and Justice, in turn, was required to appoint an
independent supervisory body to oversee the working of the Tribunals.
 Validity of Section 184 - Court held that It is always open to a Constitutional court on a challenge made to
the delegated legislation framed by the Executive to examine whether it conforms to the parent
legislation and other laws, and apply the “policy and guideline” test and if found contrary.
La Excellence IAS 11 Website: www.laex.in/testprep
9052 29 29 29 / 9052 49 29 29 (Hyderabad)
9121 41 29 29 / 9121 44 29 29 (Bangalore)
https://t.me/pdf4exams https://t.me/allupscmaterials

 Validity of Tribunal, Appellate Tribunal and Other Authorities (Qualifications, Experience and Other
Conditions of Service of Members) Rules, 2017 - The rules provide for various aspects concerning the
functioning of tribunals - Composition of the Search-cum-Selection Committees, Qualifications of
members and presiding officers, Term of Office and Maximum Age of Tribunal members, Procedure for
removal of Tribunal members etc. The Court held that the Rules impinge upon the independence of
judiciary and suffer from various other infirmities. It, therefore, directed that the Rules would require a
second look.
 Financial independence - The Court held that it is of paramount importance that every Tribunal enjoys
adequate financial independence for the purpose of its day to day functioning including the expenditure
to be incurred on (a) recruitment of staff; (b) creation of infrastructure; (c) modernization of
infrastructure; (d) computerization; (e) perquisites and other facilities admissible to the Presiding
Authority or the Members of such Tribunal.
 Judicial Impact Assessment - Court directed the Union of India to carry out a financial impact assessment
in respect of all the Tribunals referable to Sections 158 to 182 of the Finance Act, 2017 and undertake an
exercise to assess the need-based requirements and make available sufficient resources for each Tribunal
established by the Parliament.
 On Direct Statutory Appeals from Tribunals to Supreme Court - The Union government has to undertake
such an exercise expeditiously, preferably within a period of six months at the maximum, and place the
findings before Parliament for appropriate action, the Court ordered.
 Amalgamation of Existing Tribunals and Setting up of Benches - This ‘imbalance’ in distribution of case-
load and inconsistencies in nature, location and functioning of Tribunals require urgent attention. Thus,
the Court ordered that after conducting a Judicial Impact Assessment, such ‘niche’ Tribunals should be
amalgamated with other tribunals dealing with similar areas of law, to ensure effective utilization of
resources and to facilitate access to justice.
3.3 Doctrine of Essentiality:
Context:
The Supreme Court’s decision to refer the Sabarimala temple case to a larger 7-judge Bench will also re-
evaluate the “essential religious practice test”, a contentious doctrine evolved by the court to protect only
such religious practices which were essential and integral to the religion.
Highlights:
What is Doctrine of Essentiality?
 The doctrine of “essentiality” was invented by a seven-judge Bench of the Supreme Court in the ‘Shirur
Mutt’ case in 1954.
 The court held that the term “religion” will cover all rituals and practices “integral” to a religion, and
took upon itself the responsibility of determining the essential and non-essential practices of a religion.
Criticism:
 This doctrine has been criticized by several constitutional experts who argue that the doctrine has
tended to lead the court into an area that is beyond its competence, and given judges the power to
decide purely religious questions.
 The essentiality test impinges on this autonomy. The apex court has itself emphasized autonomy and
choice in its Privacy (2017), 377 (2018), and Adultery (2018) judgments.
 As a result, over the years, courts have been inconsistent on this question — in some cases they have
relied on religious texts to determine essentiality, in others on the empirical behaviour of followers, and
in yet others, based on whether the practice existed at the time the religion originated.

La Excellence IAS 12 Website: www.laex.in/testprep


9052 29 29 29 / 9052 49 29 29 (Hyderabad)
9121 41 29 29 / 9121 44 29 29 (Bangalore)
https://t.me/pdf4exams https://t.me/allupscmaterials

Concerns with respect to this review decision:


 The bench has clubbed various issues in deciding on this matter which are not related with
constitutionally protected religious beliefs. Example: Genital mutilation among Dawoodi Bohras.
 Such religious beliefs clearly violate the provisions of article 25 since they violate women’s health.
 Furthermore, some believe the review decision undermines the resolve among the judiciary to
standby its original verdicts.
 Some believe this review decision might lead to tensions in the society which we witnessed last year.
Example: politicizing of this issue; hate speeches; attack on women devotees

The Supreme Court Verdict on Sabarimala Women Entry Issue in 2018


 The Supreme Court verdict on 2018, paved the way for the entry of women of all ages into the Ayyappa
temple at Sabarimala in Kerala.
 The five-judge constitution bench headed by Chief Justice Dipak Misra, in its 4:1 verdict, said banning
the entry of women into the shrine is gender discrimination and the practice violates the rights of
Hindu women.
Sabarimala temple
 The Sabarimala temple is a temple complex located at Sabarimala inside the Periyar Tiger Reserve in
Pathanamthitta district, Kerala, India.
 Sabarimala Sree Dharma Sastha Temple is one of the most famous Hindu temples in India, located in
the Pathanamthitta district of Kerala. The temple is managed by the Travancore Devaswom Board.
 Lord Ayyappan is worshipped as a ‘Naishtika Bramhachari’ or a celibate for life. Therefore, as per a
notification by the Devaswom Board that manages the temple, women belonging to the menstruating
age are not permitted to enter the temple.

Mains question
‘Let justice be done, even if the heaven falls’. Analyze the statement with respect to some of the recent
judicial verdicts given by the Supreme Court of India. (150W|10M)

4. Union and States


4.1 ‘Destination North East’ Festival in Varanasi’
Context: A four-day 'Destination Northeast' festival was held in Varanasi in November to familiarize the
people with the rich, unexplored potential and opportunities available in the region
Highlights:
 This is a 4-day event at IIT-BHU grounds.
 The previous editions of ‘Destination North East’ were held in Delhi and Chandigarh.
 It will provide live experience to the audience.
 All 8 states will be present at the event along with their handicrafts, handlooms, organic products and
cultural troupes.
 The idea of organizing such events is to bring rest of India closer to North East and to have experience
of rich culture of North East.
Potential North-East region
 The North Eastern region has a lot to offer to visitors including Eco-tourism, culture, heritage and
business.
 North Eastern Region is ideal for both passive and adventure forms of Eco-tourism with wildlife

La Excellence IAS 13 Website: www.laex.in/testprep


9052 29 29 29 / 9052 49 29 29 (Hyderabad)
9121 41 29 29 / 9121 44 29 29 (Bangalore)
https://t.me/pdf4exams https://t.me/allupscmaterials

sanctuary/parks, good scenic beauty, waterfalls, forests etc.


 The region has many famous and old religious spots, which have been integrated with national tourism
circuits for the benefit of the pilgrims.
 In view of attractive natural environment and growing trends of outbound business meets, the NER also
offers ample opportunities for business tourism segment.

Mains question
Despite its advantages, the North East Region (NER) is categorized as one of the backward regions of
India. Discuss some measures that could be taken to change the prevalent situation in the NER
(150W|10M)

4.2 Over the states


With the economic centre of gravity shifting to states, India’s growth hinges on cooperative federalism not
just in the political sphere but also in the economic sphere.
Issues preventing constructive cooperative federalism:
 When the government amended the terms of reference of the 15th Finance Commission asking that
allocations for defence and internal security be carved out upfront, the states accused highhandedness of
the Centre.
 Government attempted to reform the land acquisition law by tweaking the balance in favor of investors
many states objected to this. This happened despite the land being on the concurrent list in the
Constitution.
Growing importance of states in India’s economic management:
 In the early years of our republic, the Centre dominated across all domains: political, economic and
administrative fields.
 The reaction to central dominance came in the early 1980s when strong regional leaders started agitating
against “the hegemony of the Centre”.
 As a consequence, the Centre yielded to the states, but largely in the political space.
 Much of the economic policy control stayed with the Centre which decided not just public investment but
even private investment through its industrial and import licensing policies, leaving the states on the
margins of economic management.
Economic reforms:
Starting from 1991, three trends, in particular, have shifted the economic centre of gravity from the Centre
to the states.
1. Change in the content of the reform agenda: 1991 reforms – Centre could push through the
reforms of the 1990s without even informing the states because they all pertained to subjects such as
industrial licensing, import permits, exchange rate and the financial sector, which were entirely within
its domain.
 Second-generation reforms – shift the emphasis from product to factor markets like land, labour
and taxation, which often need the consent of states.
 GST – it illustrates the increased clout of the states in driving reforms more than the GST
negotiations. There was a clash of interests between the Centre and states, producer and consumer
states, large and small states and coastal and inland states. The deal could not be finalized until the
Centre guaranteed to fill the revenue gap of states according to an agreed formula.
2. Fiscal federalism:
 Estimates suggest that the Centre collects about 60 per cent of the combined revenue (Centre and
states), but gets to spend only about 40 per cent of the combined expenditure. States collect 40 per
La Excellence IAS 14 Website: www.laex.in/testprep
9052 29 29 29 / 9052 49 29 29 (Hyderabad)
9121 41 29 29 / 9121 44 29 29 (Bangalore)
https://t.me/pdf4exams https://t.me/allupscmaterials

cent of the combined revenue, but spend as much as 60 per cent of the combined expenditure.
 States now enjoy greater autonomy in determining their expenditure. The states now not only get a
larger quantum of central transfers but also get to decide on how to spend that larger quantum.
3. Management of state finances: The RBI in its latest annual report on state finances raised several
red flags: states’ increasing weakness in raising revenue; their unsustainable debt burden; the tendency
to retrench capital expenditures in order to accommodate fiscal shocks such as farm loan waivers,
power sector loans under UDAY and a host of income transfer schemes.
 As the RBI pointed out, the quality of expenditure at the state level has a multiplier effect on overall
development outcomes.
 The market will penalize the mismanagement of public finances. Even if it is the Centre or the
states, for an unsustainable debt burden, market penalizes.
4. Economic federalism:
 There is states’ growing importance in economic federalism. They play a critical role in creating a
conducive investment climate in the country.
 Much of the responsibility for improving the ease of doing business rests not with Delhi but with the
states.

Mains question
Centre’s plans to achieve a $ 5 trillion dollar economy will greatly depend upon on how it empowers the
states financially in going forward. Elaborate. (150W|10M)

5. Governance
5.1 Justice delivery
Context: Maharashtra topped the list of India's first-ever ranking of states on justice delivery among the 18
large and mid-sized states.
Highlights:
 The India Justice Report (IJR) prepared by Tata Trusts in collaboration with Centre for social Justice,
Common Cause, Commonwealth Human Rights Initiative, Daksh, TISS-Prayas and Vidhi Centre for Legal
Policy.
 Maharashtra is followed by Kerala and Tamil Nadu, while among the small states Goa topped the list
followed by Sikkim and Himachal Pradesh.
 Uttar Pradesh and Bihar shared the last and second last spot among the 18 large and mid-sized states in
the report.

La Excellence IAS 15 Website: www.laex.in/testprep


9052 29 29 29 / 9052 49 29 29 (Hyderabad)
9121 41 29 29 / 9121 44 29 29 (Bangalore)
https://t.me/pdf4exams https://t.me/allupscmaterials

Key issues highlighted in the report:


The report looked at data indicators from the four pillars, covering themes like infrastructure, human
resources, diversity (gender, Scheduled Caste/Scheduled Tribe/Other Backward Class), budgets, workload and
trends over the last five years.
1. Vacancy issue: across the pillars of the police, prisons, and the judiciary. 23 % in judiciary, 22% in police.
2. Women representation: women are also poorly represented with only 7% representation in Police, 10%
as prisons staff and about 26.5% of all judges in the High Court and subordinate courts.
3. Prisons: over occupied at 114 per cent, where 68 per cent are under-trials awaiting investigation, inquiry
or trial.
4. Budget constraints: India’s per capita expenditure on free legal aid is only 75 paisa per annum.
Way forward
1. Improving the district courts in our country by improving the physical infrastructure and filling up the
vacancies in the court.
2. Identifying the number of pending cases and the status of each case and adopt suitable measures to
address them. Example: Using technology in deciding such cases.
3. Case and court management must be encouraged and embedded in the justice delivery system. (Case
management is a comprehensive system of management of time and events in a law suit as it proceeds
through the justice system, from initiation to resolution).
4. Implementing recommendations of Malimath committee given in this regard. Such as-
 The judge-population ratio in India is 10.5 per million populations as against 50 judges per million
population in many parts of the world.
 It suggested that a new post, Director of Prosecution, be created in every.
 The appointment of Assistant Public Prosecutors and Prosecutors, it was recommended, should be
made through competitive examination.
 The Committee felt that all cases in which punishment is three years and below should be tried
summarily and punishment that can be awarded in summary trials be increased to three years.
 The Committee proposed an ‘Arrears Eradication Scheme’ to tackle cases that are pending for more
than two years. Under the scheme, such cases will be settled through Lok Adalats on a priority basis.

Mains question
‘The system of trust in the institution of judiciary is fast eroding’. Analyze the reasons for this situation.
Suggest some measures to tackle this challenge. (250W|15M)

5.2 Social Audit


Context: Department of Rural development has decided to institutionalize social audits in major schemes of
Rural Development.
Highlights:
 To institutionalize social audits in rural development programmes, Department of Rural development has
decided to organize a 2-day National Seminar on Social Audit of Rural Development Programmes at Vigyan
Bhavan, New Delhi from 13-14th November, 2019.
 It is being run jointly by Department of Rural Development and National Institute of Rural Development
and Panchayati Raj.
 The objective of the seminar is to understand the current status of social audits and social audit units and
develop plan for roll out of social audit in other programmes.

La Excellence IAS 16 Website: www.laex.in/testprep


9052 29 29 29 / 9052 49 29 29 (Hyderabad)
9121 41 29 29 / 9121 44 29 29 (Bangalore)
https://t.me/pdf4exams https://t.me/allupscmaterials

SOCIAL AUDIT:
 It generally refers to engagement of stakeholders in measuring the achievement of objectives under
any or all of the activities of a government organization, especially pertaining to developmental
goals.
 It is a powerful tool to enforce transparency and accountability.
 Aim: To have an understanding of an activity from the perspective of vast majority of people in
society for whom the institutional or administrative system is designed and to improve upon it.
 MGNREGA is the first scheme in which Social Audit was made mandatory to validate the funds
allocated and disbursed in the scheme.
 Meghalaya was the first state to make Social Audit as compulsory mechanism for validation of all
government acts and implementation of schemes.
PMAY-G:
 It is the government’s flagship housing scheme for rural areas as a part of Housing for All by 2022
mission
 Under this scheme:
 Financial assistance will be provided for construction of dwelling units and upgradation of
existing unserviceable Kutcha houses for SCs/STs and non-SC/ST rural families living below
the poverty line in rural areas.
 A minimum of 60% funds under the scheme is earmarked for assistance to SC/ST families
living below poverty line.
 The unit assistance is 1.2 lakh in plain areas and 1.3 lakh in hilly states/difficult areas/IAP
districts.
 The scheme will be implemented throughout the country in rural areas except for Delhi and
Chandigarh.
 The cost of houses will be shared between Centre and states.
 It is implemented by the Ministry of Rural development
NSAP (National Social Assistance Program):
 Objective: To provide support to aged persons, widows, disabled persons and bereaved
families on death of primary earner belonging to below poverty line households.
 At present, it consists of
 Indira Gandhi National Old Age Pension Scheme(IGNOAPS)
 Indira Gandhi National Widow Pension Scheme(IGNWPS)
 Indira Gandhi National Disability Pension Scheme(IGNDPS)
 National Family Benefit Scheme(NFBS)
 Annapurna Scheme

6. In News
6.1 State of Policing in India
The first edition of the India Justice Report has ranked states and Union Territories on the four pillars of the
justice system: Police, Prisons, Judiciary and Legal Aid.
It was brought out by the Tata Trusts in partnership with the Centre for Social Justice, Common Cause,
Commonwealth Human Rights Initiative, DAKSH, TISS-Prayas and Vidhi Centre for Legal Policy.
Description:
 The report underscores the capacity deficit plaguing policing in the country.
 According to the report, only 1 of the 22 states for which data were available, was able to fully utilize its
police modernization fund.

La Excellence IAS 17 Website: www.laex.in/testprep


9052 29 29 29 / 9052 49 29 29 (Hyderabad)
9121 41 29 29 / 9121 44 29 29 (Bangalore)
https://t.me/pdf4exams https://t.me/allupscmaterials

Challenges to policing in India:


1. Understaffed and overburdened police force
 The police-population ratio, currently 192 policemen per lakh population, is less than what is
recommended by UN i.e. 222 policemen per lakh population.
 Policing in India is a state subject which means there is significant variation across states.
 Understaffing in turn results into overburdening of work that not only reduces the effectiveness
and efficiency of the police personnel but also leads to psychological distress
2. Decreased Expenditure: Between fiscal 2011 and 2015, states spent 4.4% of their budgeted
expenditure on policing on average but this has reduced to 4% over the last four years
3. Pendency: 30% of all cases filed in 2016 were pending for investigation by the end of the year
4. Police accountability: Second Administrative Reforms Commission in 2007 has noted poor
accountability mechanism in police system
5. Police-Public Relations
 2018 survey across 22 states on perceptions about policing, found that less than 25% of Indians trust
the police highly and the reason for the distrust is that interactions with the police can be frustrating,
time-consuming and costly.
 The Second Administrative Reforms Commission has noted that police-public relations is in an
unsatisfactory state because people view the police as corrupt, inefficient, politically partisan and
unresponsive.
6. Criminalization of Politics: Improper implementation of police reforms could be attributed to lack
of political will, which in turn could be linked to the growing criminalization of politics.
Reforms needed:
1. NITI Aayog suggested the following reforms
 A Task Force must be created under the MHA to identify non-core functions that can be outsourced to
save on manpower and help in reducing the workload of the police.
 The states should be encouraged to ensure that the representation of women in the police force is
increased.
 Moving police as well as public order to the Concurrent List to tackle increasing inter-state crime and
terrorism under a unified framework.
2. Supreme Court in the Prakash Singh vs Union of India case
 Constitute a State Security Commission (SSC) to ensure that state government does not exercise
unwarranted influence or pressure on the police.

La Excellence IAS 18 Website: www.laex.in/testprep


9052 29 29 29 / 9052 49 29 29 (Hyderabad)
9121 41 29 29 / 9121 44 29 29 (Bangalore)
https://t.me/pdf4exams https://t.me/allupscmaterials

 Ensure that the DGP is appointed through the merit-based transparent process and secure a minimum
tenure of two years
 Police officers on operational duties (including SP and SHO) are also provided a minimum tenure of two
year.
3. Community Policing Model such as Janamaithri Suraksha Project (Kerala), Joint Patrolling
Committees (Rajasthan), Meira Paibi (Assam) etc.
4. The Second Administrative Reforms Commission and the Supreme Court have observed that there is a
need to have an independent complaints authority to inquire into cases of police misconduct.
WAY FORWARD:
 Internal security is very much a prerogative of police and efficient policing is needed in order to tackle
threats arising in the form of cyber-attacks, bank frauds, and organized crimes, which need to be tackled
in a more specialized manner. But for that, the police system needs to be efficient, effective and
technologically sound.

Mains Question:
Policing in India marks much inefficiency which has severly affected the quality of justice delivered.
Elaborate. Suggest some measures to change this situation. (250W|15M)

6.2 Quid pro quo


Description:
As the dramatic proceedings to impeach Trump unfold in the United States Congress, one expression that has
been heard over and over again is “quid pro quo”.
Quid pro quo
 Quid pro quo, according to the Merriam-Webster dictionary, means “something given or received for
something else”.
 In the 1500s in England, it was often used in the sense of apothecaries substituting one medicine for the
other, by accident or design.
 It has also been part of trade lexicon as a term for the barter system.
 Quid pro quo, like many Latin phrases, made its way into legal terminology, where it is now used to
imply a mutually beneficial deal between two parties.
 In political contexts, like the one involving Trump currently, it is often seen as an essential requirement
to suggest or establish corruption, wrongdoing, or impropriety.

6.3 Tourist arrivals in India


Statistics provided by the Ministry of Tourism to Parliament shows that Bangladesh, the United States, and the
United Kingdom were the top three countries from where foreign tourists arrived in India in 2018.

La Excellence IAS 19 Website: www.laex.in/testprep


9052 29 29 29 / 9052 49 29 29 (Hyderabad)
9121 41 29 29 / 9121 44 29 29 (Bangalore)
https://t.me/pdf4exams https://t.me/allupscmaterials

 The Ministry’s data also exhibit a consistent increase in overall foreign tourist arrivals as well as foreign
exchange earnings over the years 2016, 2017, and 2018.
 Among individual states, Tamil Nadu saw the most visits by foreign tourists in 2018 — over 60 lakh in that
year. Maharashtra and Uttar Pradesh followed, with over 50 lakh and 37 lakh visits respectively.
 In 2017, arrivals from Bangladesh increased significantly to 2,156,557 from 1,280,409 in the previous year.

Factors helping the growth of tourism sector in India

Courtesy: IBEF
In spite of several measures taken, the tourism in our country suffers from various inefficiencies (Arrivals grew
1.9 per cent in first four months of 2019 after registering a subdued growth in 2018) which have resulted in
less tourism growth compared to other countries. Some of the reasons for this are:
1. Poor infrastructure
2. Lack of skilled manpower
3. Lack of trained manpower
4. Poor marketing by tourism ministry
5. Issuance of foreign tourist visas also suffers from delay and such other inefficiencies
6. In recent times, certain events and circumstances is also hampering the number of foreign tourists arriving
to our country. Example: Pollution in Delhi, ensuring safety of foreign tourists in our country etc

Mains question
The potentiality of Tourism in India is yet to be realized to its true potential. Identify the factors that have
hindered the process of growth in this industry. Also, suggest some measures the government has taken
to improve the same. (250W|15M)

La Excellence IAS 20 Website: www.laex.in/testprep


9052 29 29 29 / 9052 49 29 29 (Hyderabad)
9121 41 29 29 / 9121 44 29 29 (Bangalore)
https://t.me/pdf4exams https://t.me/allupscmaterials

B. Social Issues
Women and Child
1.1 Maternal death rate:
Context:
According to the latest Sample Registration System (SRS) 2015-2017 report for MMR, a decline of 8 points
(6.2%) was observed in India’s Maternal Mortality Ratio (MMR) during 2014-16.
Highlights:
 India’s Maternal Mortality Ratio (MMR) has seen a decline from 130 per 1 lakh live births in 2014-2016 to
122 per 1 lakh live births in 2015-2017.
 While Karnataka has shown the highest percentage decline in MMR, Uttar Pradesh and Madhya
Pradesh have shown an increase by 15 points each in MMR.
The SRS had categorised the States into three groups :
 The EAG States :: Bihar, Jharkhand, Madhya Pradesh, Chhattisgarh, Odisha, Rajasthan, Uttar Pradesh,
Uttarakhand, and Assam
 Southern States :: Andhra Pradesh, Telangana, Karnataka, Kerala, and Tamil Nadu.
 Other States:: cover the remaining States and Union Territories.

Maternal mortality situation in India:

 The decline has been most significant in EAG States from 188 to 175.
 The ratio has reduced considerably from 77 to 72 per 1,00,000 live births among southern States and in
the other States from 93 to 90.
 Rajasthan’s MMR has shown the highest decrease by 13 points, followed by Odisha (12 points) and
Karnataka (11 points).
 The States of Andhra Pradesh, Bihar and Punjab have not shown any change in the ratio.
 Retaining its first position, Kerala has reduced its MMR from 46 in 2014-2016 to 42 in 2015-2017.
 Likewise, Maharashtra retained its second position with 55 and Tamil Nadu its third position with 63.
 The Survey 68% of deaths were among women in the age group of 20-29.
 The three-year data also reveals a positive development of reduction in the maternal mortality ratio
(MMR) as compared to 2014-2016.
Do you know?
The WHO says the MMR dropped by 38% worldwide between 2000 and 2017. However, an estimated 810
women died every day in 2017 from preventable causes related to pregnancy and childbirth.

La Excellence IAS 21 Website: www.laex.in/testprep


9052 29 29 29 / 9052 49 29 29 (Hyderabad)
9121 41 29 29 / 9121 44 29 29 (Bangalore)
https://t.me/pdf4exams https://t.me/allupscmaterials

Sample Registration Survey:


 The Sample Registration System (SRS) is a large-scale demographic survey for providing reliable annual
estimates of Infant mortality rate, birth rate, death rate and other fertility & mortality indicators at the
national and subnational levels.
 The field investigation consists of continuous enumeration of births and deaths in selected sample units by
resident part time enumerators, generally anganwadi workers & teachers, and an independent survey
every six months by SRS supervisors.
 The data obtained by these two independent functionaries are matched. The unmatched and partially
matched events are re-verified in the field and thereafter an unduplicated count of births and deaths is
obtained.

Mains Question
Despite substantial progress, India still has the second-highest estimated number of maternal deaths in
the world. Discuss (150W|10M)

1.2 National Child Labor Project


Context: The Central Government has initiated the “National Child Labour Project (NCLP) Scheme” to
rehabilitate child labour in the country.
About the scheme
 It is a central sector scheme
 Under this scheme, District Project Societies (DPS) are being set up at the district level under District
Magistrate/Collector for overseeing the implementation of the project.
 Under this scheme, the children in 9-14 years age group are withdrawn from work and put into NCLP
Special Training Centres, where they are provided with bridge education, vocational training, mid-day
meal, stipend, health care etc. before being mainstreamed into formal education system.
 To ensure effective enforcement of the provisions of Child Labor act and smooth implementation of NCLP
scheme, a dedicated online portal PENCIL (Platform for Effective Enforcement for No Child Labor) is
made to make the NCLP successful through better monitoring and implementation ensuring timely
disposal of work with transparency.

Mains question
Despite several reforms, programmes and initiatives; child labour still remains a persistent problem in our
society. Analyze the reasons for this situation. Also, suggest some measures to surpass this challenge.
(250W|15M)

2. Education
2.1 Quality Education:
Steps taken to ensure quality of education in schools across the country by Government.
Highlights:
 Quality education is a holistic term which includes teachers, learners, teaching – learning process, learning
environment, curriculum, pedagogy, learning outcomes, assessment etc.
 To provide quality education to all, Government of India has launched SamagraShiksha Scheme – an
integrated scheme for education extending from pre-school to class XII.
 It aims to ensure inclusive and equitable quality education at all levels of school education.
 It subsumes erstwhile schemes of
o SarvaShikshaAbhiyan (SSA)
La Excellence IAS 22 Website: www.laex.in/testprep
9052 29 29 29 / 9052 49 29 29 (Hyderabad)
9121 41 29 29 / 9121 44 29 29 (Bangalore)
https://t.me/pdf4exams https://t.me/allupscmaterials

o RashtriyaMadhyamikShikshaAbhiyan (RMSA)
o Teacher Education (TE)
 It focusses on improvement in quality of education by providing support for different interventions like:
o In-service training of teachers and school heads
o Conduct of achievement surveys at state and national levels
o Composite school grant to every school for providing a conducive learning environment
o Grants for library, sports and physical activities
o Support for RashtriyaAvishkarAbhiyan, ICT and digital initiatives
o School leadership programmes
o Remedial teaching for academically weaker students
o Support for Padhe Bharat, Badhe Bharat.
 Draft National Educational Policy, 2019 is presently under consideration.
 National Council of Educational Research and Training (NCERT), has been advised to take appropriate
steps to review National Curriculum Framework-2005.
2.2 NISHTHA programme
Context: Recently, the Ministry of Human Resources Development (HRD) launched NISHTHA programme.
Highlights:
 Department of School Education and Literacy has launched a National Mission to improve learning
outcomes at the elementary level through an Integrated Teacher Training Programme called NISHTHA –
National Initiative for School Heads and Teachers Holistic Advancement under the Centrally Sponsored
Scheme of Samagra Shiksha in 2019-20.
 Aim of this training is to motivate and equip teachers to encourage and foster critical thinking in students,
handle diverse situations and act as 1st level counselors.
 They will be oriented on and develop their skill on various aspects related to Learning outcome,
Competency based learning and Testing, School Safety and Security ICT in teaching-learning including AI
etc.
 A mobile app and Learning Management System (LMS) have been developed by NCERT. LMS is being
used for registration of Resource persons and teachers, dissemination of resources, training gap and
impact analysis, monitoring, mentoring and measuring the progress online.
3. In-news:
3.1 Draft Social Security code:
Context: Centre gets responses to draft Social Security code which had asked for public suggestions.
Objectives of the draft security code:
1. To amalgamate a clutch of existing laws and proposes several new initiatives including universal social
security for unorganized sector workers and, insurance and health benefits for gig workers.
2. To Corporatize of existing organizations like EPFO and ESIC headed by people other than the labour
minister.
Key highlights in the draft Social Security code:
1) Insurance, PF, life cover for unorganized sector employees:
 The draft code says the “Central Government shall formulate and notify, from time to time,
suitable welfare schemes for unorganised workers on matter relating to life and disability cover;
health and maternity benefits; old age protection;
 Benefits including schemes relating to provident fund, employment injury benefit, housing,
educational scheme for their children, old age and funeral assistance.
2) Corporatization of EPFO and ESIC:
 The pension, insurance and retirement saving bodies including EPFO and ESIC will be body corporate. The
world body corporate has been added in the draft and may bring in a departure from the current
autonomous body status of such organization.

La Excellence IAS 23 Website: www.laex.in/testprep


9052 29 29 29 / 9052 49 29 29 (Hyderabad)
9121 41 29 29 / 9121 44 29 29 (Bangalore)
https://t.me/pdf4exams https://t.me/allupscmaterials

 Appointment of chief executive officers (CEOs) in the labour minister, labour secretary, the central PF
commissioner and Director General of ESIC may not be by default the head of such organizations.
 Financial Advisor and Chief Accounts Officer to assist the Chief Executive Officer in the discharge of his
duties
3) Benefits for Gig workers: Suitable social security schemes for gig workers and platform workers and such
schemes would encompass issues like “life and disability cover", “health and maternity benefits", “old age
protection" and “any other benefit as may be determined by the Central Government.
4) Maternity Benefit: Every woman shall be entitled to, and her employer shall be liable for, the payment of
maternity benefit at the rate of the average daily wage for the period of her actual absence.
5) Existing labour laws that the code will merge:
The Code on Social Security, 2019 once in place will merge eight exiting labour laws including Employees'
Compensation Act, 1923; Employees‘ State Insurance Act, 1948, Employees‘ Provident Funds and
Miscellaneous Provisions Act, 1952; Maternity Benefit Act, 1961; Payment of Gratuity Act, 1972; Cine Workers
Welfare Fund Act, 1981; Building and Other Construction Workers Cess Act, 1996 and Unorganized Workers‘
Social Security Act, 2008.
Mains Questions
What is Social Security Code? Critically Analyse its possible impacts on Unorganised sector. (150W|10M)

3.2 Maintenance and Welfare of Parents and Senior Citizens Amendment Bill
Context: Maintenance and Welfare of Parents and Senior Citizens Amendment Bill was recently cleared by
the union cabinet committee.
Highlights:
 The proposal ‘Maintenance and Welfare of Parents and Senior Citizens Amendment Bill’ has the following
major salient features:
 Definition of Children and Parents; maintenance and welfare has been expanded
 Mode of submission of application for maintenance has been enlarged
 Ceiling of Rs.10000 has been removed
 Registration of Senior Citizens Care Homes/Homecare Service Agencies etc., have been included.
 Minimum standards for senior citizen care homes have been included
 Appointment of Nodal Police Officers for Senior Citizens in every Police Station and District level
Special Police Unit for Senior Citizens has been included.
Maintenance and Welfare of Parents and Senior Citizens Act: The legislation was enacted in 2007,
initiated by Ministry of Social Justice and Empowerment, Government of India.
Features:
 To provide more effective provision for maintenance and welfare of parents and senior citizens
 It makes a legal obligation for children and heirs to provide maintenance to senior citizens and parents,
by monthly allowance.
 It also provides simple, speedy and inexpensive mechanism for the protection of life and liberty of older
persons.

Mains question
‘Ageing in India is exponentially increasing; this will also lead to increasing social challenges associated
with elderly care’. Discuss (250W|15M)

La Excellence IAS 24 Website: www.laex.in/testprep


9052 29 29 29 / 9052 49 29 29 (Hyderabad)
9121 41 29 29 / 9121 44 29 29 (Bangalore)
https://t.me/pdf4exams https://t.me/allupscmaterials

3.3 Hunar Haat and employment opportunities for minorities


Context: Hunar Haat has proved to be an “Empowerment Exchange” for Master Artisans and craftsmen
Highlights:
 Ministry of Minority Affairs has been working on war-footing to establish “Hunar Hubs”
 Hunar Haat has become “Hub to encourage and promote” traditional master artisans and craftsmen from
across the country.
 More than 2 lakh 50 thousand master artisans, craftsmen and culinary experts have been provided
employment and employment opportunities in last 3 years which includes large number of women
artisans.
About Hunar Haat: These exhibitions are organized for showcasing very exquisite pieces of Handicrafts &
Handloom work prepared master artisans and craftsmen including women artisans from all most all the
states of the country

3.4 Aadi Mahotsav


It is a joint initiative of Union Ministry of Tribal Affairs and Tribal Cooperative Marketing Development
Federation of India(TRIFED).
 Van Dhan Scheme:
 It is an initiative by Ministry of Tribal Affairs and TRIFED.
 It seeks to improve tribal incomes through value addition of tribal products
 It will be implemented through Ministry of Tribal Affairs as Nodal Department at the Central Level and
TRIFED as Nodal agency at the National Level.
 Under this scheme, 10 SHGs of 30 tribal gatherers is constituted.
 The Van DhanVikas Kendra is for providing skill upgradation and capacity building training and setting up
of primary processing and value addition facility.
 Eklavya Model Residential Schools:
 The objective of EMRS is to provide quality middle and high level education to Scheduled Tribe(ST)
students in remote areas
 It is to enable them to avail of reservation in high and professional educational courses and as jobs in
government and public and private sectors
 At least one EMRS is to be setup in each integral Tribal Development Agency (ITDA)/Integrated Tribal
Development Project (ITDP) having 50% ST population in the area.

La Excellence IAS 25 Website: www.laex.in/testprep


9052 29 29 29 / 9052 49 29 29 (Hyderabad)
9121 41 29 29 / 9121 44 29 29 (Bangalore)
https://t.me/pdf4exams https://t.me/allupscmaterials

C. ECONOMY
1. Economic Development
1.1 Joblessness rises to 3-year high
The rate for October is at the highest level since August 2016, says CMIE
 The urban unemployment rate for October 2019 stood at 8.9%, slightly higher than the rural
unemployment rate of 8.3%.
 Among States, Tripura and Haryana saw unemployment levels of more than 20% (the highest).
 While the unemployment was the lowest in Tamil Nadu at 1.1%.
 Rajasthan saw its unemployment rate double between September and October 2019.
Reasons:
 Automobile sector: Has been facing intense heat due to the mandated diesel phasing out and push for
electric vehicles.
 Export sector: This has been mired by challenges both structural as well as cyclical. The structural issues
being more difficulty in doing business via cash, focus on gig economy. The cyclical issues being lack of
monetary policy transmission and global slowdown.
 NPA Problem: The banks are unable to lend to the industries which has led to drying up of funds for them.
 Agriculture is a Seasonal Occupation: Agriculture is underdeveloped in India. It provides seasonal
employment. Large part of population is dependent on agriculture. But agriculture being seasonal
provides work for a few months. So this gives rise to unemployment.
 Fall of Cottage and Small industries: The industrial development had adverse effect on cottage and small
industries. The production of cottage industries began to fall and many artisans became unemployed.
 Slow Growth of Industrialization: The rate of industrial growth is slow. Though emphasis is laid on
industrialization yet the avenues of employment created by industrialization are very few.
 Less Savings and Investment: There is inadequate capital in India. Above all, this capital has been
judiciously invested. Investment depends on savings. Savings are inadequate. Due to shortage of savings
and investment, opportunities of employment have not been created.
 Housing Sector: The housing finance companies have had an asset liability mismatch. Firms have
borrowed short-term when their revenue streams are longer-term. This is the reason behind ILFS fiasco.
 Construction sector : There has been a stagnation on the type of people who have steady predictable
income which has affected EMI based housing demand.
 So, these above issues has created multiplier effect and led to reduction in the jobs in the country.
Steps needs to be taken
 Give auto sector incentives to invest and shift to electric vehicles.
 Incentives to auto sector employees to upskill on electric vehicles.
 Change GST collection to quarterly for companies below Rs 1 crore.
 Reduce the GST slab rates.
 Adopt the Direct Tax Code, cut income tax for the bottom slab.
 Improve credit flow to both consumer and industry.
 Reduce real interest rates by 135 basis points as cost of capital has to come down.
 Change the credit culture in public sector banks.
 Stimulus should drive investment, up-skilling for displaced employees.
 Factor market reforms, including bringing the cost of land down.

La Excellence IAS 26 Website: www.laex.in/testprep


9052 29 29 29 / 9052 49 29 29 (Hyderabad)
9121 41 29 29 / 9121 44 29 29 (Bangalore)
https://t.me/pdf4exams https://t.me/allupscmaterials

Conclusion
Both merger and governance reforms were important but are obviously not sufficient from
the slowdown point of view. To kick-start the consumption cycle money has to go into the common man's
pocket. This can happen by reducing income tax for the lowest slab, as recommended by the Direct Tax Code
report. It can be done by making GST filing quarterly for MSMEs with less than Rs 10 crore turn-over to ensure
they survive the slowdown. The GST Council can look at reducing rate slabs and reduce the overall burden on
corporate.

Mains Question
The galloping unemployment rate in India is a serious concern for policymakers in our country. Analyze
the reasons for the rise in recent rise in unemployment in India. Discuss some of the measures that are
needed to address this situation. (250W|15M)

1.2 Real Estate reforms


Context:
At recent capital market event, Finance minister told that the Government would address the ‘sluggishness’
being faced by the real estate sector with the help of Reserve Bank of India (RBI) to look at possible
amendments in the laws.
Highlights:
 Analysts are calling for an urgent booster dose for the real estate sector.
 A slump in the housing market has left builders struggling to repay loans to Non-Finance Banking
Companies (NBFCs).
 As per Fitch Rating's Indian division, with about $10 billion of development loans coming up for
repayment in the first half of 2020, the fallout could spread to mainstream banks.
 The focus on real estate is part of Govt.’s broader plan to kick-start economic growth, which has
slowed to a six-year low.
What is the concern?
 The Real Estate industry has failed to recover from the twin shocks of the Demonetization and the
Goods and Services Tax (GST).
 Sales of housing units fell 10 per cent across India's top nine cities to 52,885 units in the third quarter
of 2019 versus the year-ago quarter.
 The slowdown in housing sales is largely due to fewer new launches, lack of buyer confidence in older
under-construction projects and liquidity crunch in the market, the firm noted.
 The present NPA position of banks providing alternative route for raising debt by entrepreneurs.
What are the measures to be taken to infuse liquidity into the sector?
 If there is zero GST implemented for real estate projects at least for six months, it would make a
marked difference.
 There is an urgent need for active lenders in real estate, with existing banks not lending enough.
 Deepening the debt market and the stock exchanges can play an active role in attracting more retail
investors in the segment.
 Development of the bond market required much more attention and a unified approach by all
regulators.
 Stock exchanges are for-profit commercial entities; they needed to devote sufficient resources for
regulatory functions and should abstain from misusing their oligopolistic position by having exorbitant
and unreasonable fee structure.

La Excellence IAS 27 Website: www.laex.in/testprep


9052 29 29 29 / 9052 49 29 29 (Hyderabad)
9121 41 29 29 / 9121 44 29 29 (Bangalore)
https://t.me/pdf4exams https://t.me/allupscmaterials

Do you know?
The house price-to-income ratio, which measures the cost of housing versus the change in income levels,
rose from 56.1 in March 2015 to 61.5 in March 2019, indicating home purchases have become less
affordable.
Conclusion
The housing sales fare better during the festive quarter than the other quarters of the year. The reduction in
corporate tax will incentivize corporate to pump up investments. Also, the additional discount on the interest
rates while taking a home loan of less than Rs 45 lakh till March 31, 2020 will help in increasing the demand
during this period.

Mains question
Real estate sector can play a significant part in reviving the economic situation that our country is
facing. Comment. (150W|10M)

1.3 Moody’s Investors Service.


Context:
The global Rating Agency has changed its outlook for India from 'stable' to negative'.
Highlights:
 Moody’s has downgraded outlook on companies belonging to information technology, infrastructure
as well as oil and gas sectors.
 Moody has retained its 'Baa2' rating for the country's foreign and local currency.
 It's following in the footsteps of other rating and monetary agencies that have downgraded India's
GDP growth forecast.
 Other financial institutions that have seen a downgrade are Exim Bank, Hero FinCorp, Hudco and
Indian Railway Finance Corporation (IRFC).
How does credit rating work?

What did Moody’s refer to?


 In its report, Moody’s cited increasing risks that India’s economic growth will remain materially
lower, partly due to lower government and policy effectiveness.
 India is undergoing a significant slowdown — its economic growth hit a six-year low in the April-to-June
quarter.
 Tax revenue growth is nowhere near budgeted levels and with the slowdown extending into the third
quarter, it is clear that tax revenues will undershoot by a wide margin.
La Excellence IAS 28 Website: www.laex.in/testprep
9052 29 29 29 / 9052 49 29 29 (Hyderabad)
9121 41 29 29 / 9121 44 29 29 (Bangalore)
https://t.me/pdf4exams https://t.me/allupscmaterials

 The government in October 2019 gave away Corporate Tax Concessions amounting to a whopping Rs.1.45
lakh crores and it now appears certain that the government will miss the fiscal deficit target of 3.3% of
GDP.
 Moody’s has projected that the deficit will slip to 3.7% of GDP this fiscal. Ratings agencies are ultra-
sensitive to fiscal deficit overruns.
 India’s growth outlook has deteriorated sharply with a crunch that started out in the non-banking financial
institutions (NBFls) spreading to retail businesses, car makers, home sales and heavy industries.
 Moody’s said India’s potential Gross Domestic Product (GDP) growth and job creation will remain
constrained unless reforms are advanced to directly reduce restrictions on the productivity of labour and
land, stimulate private sector investment, and sustainably strengthen the financial sector.
Impact:
 The rupee and equities markets came under pressure after rating agency Moody’s revised the
country’s rating outlook to negative. While revising the outlook, Moody’s has affirmed India’s
sovereign rating at Baa2.
How the government responded?
 The Government said that the fundamentals of the economy remain quite robust and series of
reforms undertaken recently would stimulate investments.
 The Union finance ministry in a statement said that India’s potential growth rate remains unchanged,
and “India’s relative standing remains unaffected” as evident from the assessment by the
International Monetary Fund (IMF) and other multilateral organizations that continue to hold a
positive outlook on India.
 Government of India has also proactively taken policy decisions in response to the global slowdown.
Conclusion
The government has undertaken series of financial sector and other reforms to strengthen the economy as a
whole. The Government has also proactively taken policy decisions in response to the global slowdown. These
measures would lead to a positive outlook on India and would attract capital flows and stimulate investments.

Mains Question
The current economic slowdown point to a systemic change that is required in our economic
system. Do you agree? Justify your opinion. (150W|10M)

1.4 Retail inflation rise:


Context: India’s retail inflation unexpectedly quickened to a 16-month high at 4.62%, exceeding the central
bank’s medium-term target for the first time since July 2018.
Highlights:

La Excellence IAS 29 Website: www.laex.in/testprep


9052 29 29 29 / 9052 49 29 29 (Hyderabad)
9121 41 29 29 / 9121 44 29 29 (Bangalore)
https://t.me/pdf4exams https://t.me/allupscmaterials

 Core retail inflation, excluding food and fuel prices, moderated to its lowest in 94 months at 3.47%.
 India’s factory output shrank at 4.3 in September 2019, recording its worst show since at least April
2012.
 The surge in food prices is unlikely to deter the RBI from cutting policy rates, with a set of macro-
economic data pointing towards sharper than expected deceleration of the Indian economy.
What is Consumer Price Index (CPI)
 It measures price changes from the perspective of a retail buyer.
 It measures changes over time in the level of retail prices of selected goods and services on which
consumers of a defined group spend their incomes.
 Four types of CPI are as follows:
 CPI for Industrial Workers (IW)
 CPI for Agricultural Labourer (AL)
 CPI for Rural Labourer (RL)
 CPI (Rural/Urban/Combined)
 Of these, the first three are compiled by the Labour Bureau in the Ministry of Labour and Employment.
Fourth is compiled by the Central Statistical Organisation (CSO) in the Ministry of Statistics and
Programme Implementation.
 Base Year for CPI is 2012.

Where does inflation stand in terms of what RBI wants?


CPI inflation has broadly been within the Reserve Bank of India's comfort zone. The central bank
has retained its consumer price inflation forecast for the second half of 2019-20 at a range
between 3.5 and 3.7%.

1.5 Arcelor Mittal’s takeover of Essar cleared by SC


The Supreme Court accepted Arcelor Mittal’s offer to pay an aggregate Rs 42,000 crore as an upfront amount
to the secured financial creditors of bankrupt Essar Steel.
Description:
 This Supreme Court order paved the way for Arcelor Mittal to take over Essar and enter the world’s
second biggest steel market.
 The court set aside a judgment of the National Company Law Appellate Tribunal (NCLAT), which held that
the amount ought to be shared equally between financial creditors and operational creditors.
 Supreme Court told that the equality principle cannot be stretched to treating unequal equally. That will
destroy the very objective of the Insolvency and Bankruptcy Code (IBC) — to resolve stressed assets.
 Equitable treatment is to be accorded to each creditor depending upon the class to which it belongs:
secured or unsecured, financial or operational.

La Excellence IAS 30 Website: www.laex.in/testprep


9052 29 29 29 / 9052 49 29 29 (Hyderabad)
9121 41 29 29 / 9121 44 29 29 (Bangalore)
https://t.me/pdf4exams https://t.me/allupscmaterials

National Company Law Tribunal (NCLT)


 The Central Government has constituted National Company Law Tribunal (NCLT) under section 408 of
the Companies Act, 2013.
National Company Law Appellate Tribunal (NCLAT)
 It was constituted under the Companies Act, 2013 for hearing appeals against the orders of National
Company Law Tribunal
 NCLAT is also the Appellate Tribunal for hearing appeals against the orders passed by NCLT(s) under
Section 61 of the Insolvency and Bankruptcy Code, 2016 (IBC) & against IBBI, CCI.
What is Insolvency and Bankruptcy code (IBC)? The Insolvency and Bankruptcy Code, 2016 (IBC) is the
bankruptcy law of India which seeks to consolidate the existing framework by creating a single law for
insolvency and bankruptcy. Some of the features of the code are:
1. Resolution of Insolvency: The Insolvency and Bankruptcy Code, 2016 lays down the separate insolvency
resolving procedures for companies, individuals as well as partnership companies. It is possible to initiate
the procedure either by the creditors or the debtors.
2. Regulator of Insolvency: The Code lays down that the Insolvency and Bankruptcy Board of India shall
oversee the proceedings related to insolvency in the nation and also regulate all the organizations that
have been registered by the board.
3. Licensed Insolvency Professionals: The management of insolvency procedure shall be done by licensed
insolvency professionals. They would also exercise control on the debtor’s assets at the time of the
insolvency procedure.
4. Insolvency and Bankruptcy Adjudicator: The Code has introduced two distinct tribunals for overseeing
the procedure resolving insolvency, for companies and individuals. These are (i) the National Company
Law Tribunal for organizations and Limited Liability Partnership companies; as well as (ii) the Debt
Recovery Tribunal for overseeing insolvency resolution for individuals as well as partnership firms.

Mains question
1. What is Insolvency and Bankruptcy code (IBC)? Discuss the importance of this code for a country like
India. (250W|15M)
2. Discuss the reasons for the crisis in telecom and banking sector. How will this crisis impact our
economy? What are some of the steps the government has taken recently to alleviate this situation?
(250W|15M)

2. Industries
2.1 Core sector output falls 5.2% in September
Data reflects severity of the slowdown
 Output of eight core infrastructure industries contracted by 5.2% in September, indicating the severity
of the economic slowdown.
 As many as seven of eight core industries saw a contraction in output in September. Coal production
fell steeply by 20.5%, crude oil by 5.4%, and natural gas by 4.9%. Output of refinery products (-6.7%),
cement (-2.1%), steel (-0.3%), and electricity (-3.7%) too declined.
 The only segment to post growth in September was fertilizers, where production increased by 5.4%.
Reasons for Core Output Fall
 Challenging market conditions.
 The orders from the US-China trade commissions have impacted the Indian export market which also
contributed to a significant slower rate in the market.
La Excellence IAS 31 Website: www.laex.in/testprep
9052 29 29 29 / 9052 49 29 29 (Hyderabad)
9121 41 29 29 / 9121 44 29 29 (Bangalore)
https://t.me/pdf4exams https://t.me/allupscmaterials

 Low Inflation rate which simultaneously reflects the weak demand for both consumptions and
investment goods.
 Introduction of demonetization.
 The poor implementation of GST led to a whooping rise in unemployment, which made the wages
flow directly into particular rich individuals without actually making into a better economic
improvement.
 Core industry can be defined as the main industry. In most countries, there is a particular industry that
seems to be the backbone of all other industries and it qualifies to be the core industry.
 In India, there are eight core sectors comprising of

 These eight Core Industries comprise nearly 40.27% of the weight of items included in the Index of
Industrial Production (IIP), which measures factory output.
 Index of Eight Core Industries is released by Ministry of Commerce and Industry.

2.2 Index of Industrial Production (IIP):


Context:
IIP shrinks by 4.3% to lowest in 8 years in all 3 major sectors saw contraction.
Key facts:
 This is the first time after 2012 that all three broad-based sectors have contracted and the lowest
monthly growth in the 2011-12 base year series.
 In the old (2004-05) base, IIP in October 2011 contracted by 5%.
 Contraction observed are:
 The capital goods sector - 20.7%.
 The mining sector contracted by- 8.5%.
 The manufacturing sector contracted- 3.9%.
 The electricity sector -2.6%.

La Excellence IAS 32 Website: www.laex.in/testprep


9052 29 29 29 / 9052 49 29 29 (Hyderabad)
9121 41 29 29 / 9121 44 29 29 (Bangalore)
https://t.me/pdf4exams https://t.me/allupscmaterials

What is Index of Industrial Production (IIP)?


IIP is an index that tracks manufacturing activity in different sectors of an economy.
What is IIP data?
 The IIP number measures the industrial production for the period under review, usually a month, as
against the reference period.
 IIP is a key economic indicator of the manufacturing sector of the economy.
 There is a lag of six weeks in the publication of the IIP index data after the reference month ends.
 IIP index is currently calculated using 2011-2012 as the base year.
IIP Index Components:
 Electricity, crude oil, coal, cement, steel, refinery products, natural gas, and fertilisers are the eight core
industries that comprise about 40 % of the weight of items included in the IIP.
 Mining, manufacturing, and electricity are the three broad sectors in which IIP constituents fall.
Who releases Index of Industrial Production or IIP data?
 IIP data is compiled and published by Central Statistical Organisation (CSO) every month.
 CSO operates under the Ministry of Statistics and Programme Implementation (MoSPI).
 The IIP index data, once released, is also available on the PIB website.
Who uses IIP data?
 The factory production data (IIP) is used by various government agencies such as the Ministry of Finance,
the Reserve Bank of India, private firms and analysts, among others for analytical purposes.
 The data is also used to compile the Gross Value Added (GVA) of the manufacturing sector in the Gross
Domestic Product (GDP) on a quarterly basis.
Where is IIP data sourced from?
 The CSO uses secondary data to reach the monthly IIP number. The data is sourced from various
agencies in different ministries or departments of the government.
 The Department of Industrial Policy and Promotion (DIPP) is the source for the major chunk of data for
the calculation.
IIP vs ASI:
 While the IIP is a monthly indicator, the Annual Survey of Industries (ASI) is the prime source of long-term
industrial statistics.
 The ASI is used to track the health of the industrial activity in the economy over a longer period. The index
is compiled out of a much larger sample of industries compared to IIP.
Mains question
Discuss some of the factors that have led to consistent decline in our Index of Industrial production (IIP) in
recent few quarters. (150W|10M)

2.3 Cabinet approves Industrial Relations Code Bill, 2019.


Highlights:
 Union Cabinet has given its approval for introduction of Industrial Relations Code, 2019 in Parliament.
 The draft code on Industrial Relations has been prepared after amalgamating, simplifying and rationalizing
the provisions of
o Trade Unions Act, 1926
o Industrial Employment (Standing Orders) Act, 1946
o Industrial Disputes Act, 1947.
Benefits:
o Setting up of 2-member tribunal for adjudication of cases
o Imparting flexibility to the exit provisions
o Re-skilling fund, to be utilized for crediting to workers in the manner prescribed.
o Definition of fixed term employment and that it would not lead to any notice period and payment
of compensation on retrenchment excluded.
o Vesting of powers with government officers for adjudication of disputes involving penalty as fines
therefore lessening the burden on tribunal.
La Excellence IAS 33 Website: www.laex.in/testprep
9052 29 29 29 / 9052 49 29 29 (Hyderabad)
9121 41 29 29 / 9121 44 29 29 (Bangalore)
https://t.me/pdf4exams https://t.me/allupscmaterials

3. Infrastructure
3.1 Rs. 25,000-cr. fund to help housing sector.
Context:
The Union Cabinet has approved the creation of an Alternative Investment Fund (AIF) of Rs. 25,000 crores to
provide last-mile funding for stalled affordable and middle-income housing projects across the country.
Highlights:
 Rs 25,000 core fund to help 1,600 stalled housing projects including ones that have been declared bad
loans or admitted for insolvency proceedings.
 AIF will act as a ‘special window’ to provide loans.
 The AIF funds will be released in stages through an escrow account and will be contingent upon
completion of approved phase.
 4.58 lakh housing units are being targeted to be completed with a view to generate employment as revive
demand of cement, iron and steel industries.
 Only RERA- registered project with positive net worth will be provided funds.
Who are the contributors?
AIF will comprise Rs 10,000 crores from the Govt. and the remaining being provided by state insurer LIC and
the country’s largest lender SBI.
Why there is a crisis?
1. Unattractive/Negative ROI:
The returns on investments in residential real estate have dropped from two or even three-digit values to low
single-digit .The ROI from housing currently clocks in at a meager 2-3% even in the most favorable markets
across Indian cities.
2. The Economic Slowdown :
This has a direct correlation to employment creation and job security in India. Cash-conservation is the order
of the day in a country where citizens are unsure of getting jobs, or job continuity.
3. Lack of Faith in under-construction Properties:
The massive burden of heavily delayed and terminally stuck housing projects on the market is both a cause
and effect when it comes to low homebuyer sentiment.
4. The Anticipation of a significant Price Correction:
Neither end-users nor investors are interested in putting money into a depreciating asset.
5. Unfavorable Loan-to-value Ratio:
RBI has laid down stricter norms and guidelines for banks dispensing housing loans. In recent times, the loan-
to-value (LTV) ratio is now restricted to 70%, whereas it previously ranged between 80% and even 90% of the
property value.
To add to this, NBFCs/HFCs have also slowed down their lending to individual homebuyers.
6. High Taxation on under-construction Homes:
GST has replaced the multiple levels of taxation previously applicable on a home purchase, but the increased
simplicity has not resulted in better cost-efficiency (GST on Real Estate: 5% on Under Construction & 1% on
Affordable Housing).
7. Stagnant Job Market:
The problem of low job creation looms large in India, even though the economic growth rate is predicted to be
the highest. The Periodic Labour Force Survey (PLFS) by the National Sample Survey Office (NSSO) maintained
that the unemployment rate in both urban and rural India combined stood at 6.1% in FY18.

La Excellence IAS 34 Website: www.laex.in/testprep


9052 29 29 29 / 9052 49 29 29 (Hyderabad)
9121 41 29 29 / 9121 44 29 29 (Bangalore)
https://t.me/pdf4exams https://t.me/allupscmaterials

8. Demonetization:
The possibility of involving large cash components was a big factor that drove housing sales to investors.
Demonetization has not eliminated this practice as intended.
9. Growing Awareness of other Investment Options:
In the ‘golden years’ of India’s housing market, property was the default go-to option for big-ticket
investment. With real estate's fading allure, investors began exploring other options and found them to quite
rewarding.
What is an Alternate Investment Fund ("AIF")?
 Alternative Investment Fund or AIF means any fund established or incorporated in India which is a
privately pooled investment vehicle which collects funds from sophisticated investors, whether Indian or
foreign, for investing it in accordance with a defined investment policy for the benefit of its investors.
 AIF does not include funds covered under the SEBI (Mutual Funds) Regulations, 1996, SEBI (Collective
Investment Schemes) Regulations, 1999 or any other regulations of the Board to regulate fund
management activities.

What is the RERA (Real Estate Regulatory Act)?


 The Real Estate (Regulation and Development) Act, 2016 (RERA) is an Act passed by the Indian
Parliament.
 The RERA seeks to protect the interests of home buyers and also boost investments in the real
estate sector.
 The Rajya Sabha passed the RERA bill on March 10, 2016, followed by the Lok Sabha on March 15,
2016 and it came into force from May 1, 2016.
 Under the Act, the Central and State governments, are required to notify their own rules under the
Act, six months, on the basis of the model rules framed under the central Act.

Do You Know?
India needs to spend USD 200 billion annually on its infrastructure, but has been able to spend only USD
100-110 billion annually. As per the current trend, India can raise around USD 3.9 trillion.

Mains questions
A convincing and sustainable revival for the Indian housing sector depends on numerous factors getting
resolved. Elucidate. (150W|10M)

3.2 Ministry of Road Transport and Highways releases ‘Road Accidents in India-2018’
Highlights:
 Ministry of Road, Transport and Highways has released the Road Accidents in India, 2018.
 It is annual publication brought by Transport Research Wing of the Ministry of Road, Transport and
Highways.
 The Transport Research Wing of the Ministry of Road, Transport and Highways reports in accidents,
related deaths and injuries, calendar year-wise, based on information supplied by Police Departments of
States and UTs.
Findings of the Report:
 Road accidents have increased marginally by 0.46% in 2018.
 The reports also brings out that while accidents, deaths and injuries registered a steep rise till 2010,
somewhat stabilized after 2010 with only marginal year to year fluctuations.

La Excellence IAS 35 Website: www.laex.in/testprep


9052 29 29 29 / 9052 49 29 29 (Hyderabad)
9121 41 29 29 / 9121 44 29 29 (Bangalore)
https://t.me/pdf4exams https://t.me/allupscmaterials

 National Highways comprise 1.94% of total road network but accounted for 30% accidents and 36% of
deaths in 2018.
 State highways account for 3% of total road length but accounted for 25% of accidents and 27% of
deaths.
 Other roads constitute 95% of total roads and accounted for 45% of accidents and 38% of deaths.
 Road accidents are multi-casual and are often the result of an interplay of various factors:
 Human error
 Road environment
 Vehicular condition
 Under the category of Traffic Rule Violations, over-speeding is a major killer accounting for 64.4% of the
persons killed.
 A major initiative of the Ministry in the field of Road Safety is the passing of Motor Vehicle Amendment
Bill, 2019.
 This Act is hoped to bring in greater discipline and responsible attitude amongst road users and will mark a
turning point in road accidents and related deaths to which the country is seriously committed.

Mains question
Poor performing policies and programmes have made our Indian roads dangerous. Elaborate. What steps
has the government taken in recent times to prevent increasing road accidents in India? (250W|15M)

4. Agriculture
4.1 Seed treaty
Highlights:
 Union Agriculture Minister attended the 8th session of Governing Body of International Treaty of Plant
Genetic Resources for Food and Agriculture (ITPGRFA) in Rome, Italy.
The International Treaty on Plant Genetic Resources for Food and Agriculture
(ITPGRFA):
 It is also known as the Seed Treaty.
 It was signed in 2001 and effective from 2004.
 It is a comprehensive international agreement for ensuring food security through conservation,
exchange and sustainable use of world’s plant genetic resources for food and agriculture (PGRFA), as
well as the fair and equitable benefit sharing arising from its use.
 It recognizes farmer’s rights, subject to national laws.
The Protection of Plant Varieties and Farmers' Rights Act (PPV&FR Act):
 It is the world’s only IPR legislation which grants Intellectual Property Rights not only to the plant
breeders but also to the farmers by protecting new, extant and farmer’s varieties.
 According to the legislation, a farmer is entitled to save, use, sow, re-sow, exchange, share or sell his
farm produce including seed of a variety protected under PPV&FR Act, 2001.
 Under the provisions of this act, 138 farmers/farming communities have been awarded with Plant
Genome Saviour Awards.
 It was formulated by Indian Legislature to be in line with Convention on International Union for the
Protection of New Varieties of Plants (UPOV).
International Union for the Protection of New Varieties of Plants (UPOV):
 It is an Intergovernmental organization to provide and promote an effective system of plant variety
protection with aim of encouraging development of new varieties of plants for the benefit of
society.
 Protection to farmers: The UPOV Convention provides the basis for members to encourage plant
breeding by granting breeders of new plant varieties an IPR: the breeder’s right.

La Excellence IAS 36 Website: www.laex.in/testprep


9052 29 29 29 / 9052 49 29 29 (Hyderabad)
9121 41 29 29 / 9121 44 29 29 (Bangalore)
https://t.me/pdf4exams https://t.me/allupscmaterials

4.2 Zero Budget Natural Farming


Highlights:
 Government has been promoting Organic farming under the dedicated scheme of Paramparagat Krishi
Vikas Yojana.
 PKVY encourages all kinds of chemical free farming systems including ZBNF.
 Indian Council of Agriculture Research through ICAR-Indian Institute of Farming Systems Research has
initiated a study on Evaluation of Zero Budget Natural Farming practices in Basmati/ coarse rice-wheat
system from Rabi Season 2017 at 4 locations namely Modipuram, Pantnagar, Ludhiyana, Kurukshetra .
 Government has been implementing various schemes to meet the target of DFI (Doubling Farmers
Income) by 2022. The major schemes include:
 National Food Security Mission – for improved production and productivity of food grains and
oilseeds.
 Mission for Integrated Development of Horticulture (MIDH) – to enhance production and
productivity of horticultural crops.
 Paramparagat Krishi Vikas Yojana and Mission Organic Value Chain Development for North East
Region – to reduce cost of cultivation and ensure chemical free quality products that can fetch
premium prices to farmers.
 Soil Health Cards – Aims at production of judicious use of fertilizers through soil test based
recommendations thereby reducing excess use of fertilizers.
 PMKSY-PDMC (Pradhan Mantri Krishi Sinchayee Yojana – Per Drop More Crop) - to promote
water conservation and reduction in use of inputs through drip and sprinkler irrigation.
 Bamboo Mission and Agro-forestry - To harness the potential of bamboo crop. It is the sub
scheme of 100% Centrally Sponsored Scheme called Mission for Integrated Development of
Horticulture (MIDH).
 e-NAM– e-NAM or the e-trading platform (online trading portal) for the National Agriculture
Market (NAM) was launched on April 2016 for transparent and competitive markets for improved
remuneration to the farmers.
 Kisan Credit Card (KCC) – The Kisan Credit Card (KCC) scheme was announced in the Budget speech of
1998-99 to fulfill the financial requirements of the farmers at various stages of farming through
institutional credit. The model scheme was prepared by the National Bank for Agriculture and Rural
Development (NABARD) on the recommendation of V Gupta committee.
 Minimum Support Price - It is a form of market intervention by the Government of India to insure
agricultural producers against any sharp fall in farm prices. The prices are a guarantee price for their
produce from the Government.
 Pradhan Mantri Kisan Samman Nidhi (PM-KISAN) – In this scheme, all small and marginal farmers
will get up to Rs. 6,000 per year as minimum income support. It was launched in 2019.
Do You Know?
Krishi ka Rishi’ is the title farming communities across the country have bestowed on Subhash Palekar. This
agriculturist is the creator of the ‘Zero Budget Natural Farming’ model.
In 2016, in recognition of his work and the impact he was creating, the Government of India conferred
Palekar with the prestigious Padamashri Award. Palekar also made history for being the first active farmer
to receive the award.

Mains question
Indian agriculture needs nothing short of structural changes if we have to double the farmer’s income by
2022. Explain (250W|15M)

La Excellence IAS 37 Website: www.laex.in/testprep


9052 29 29 29 / 9052 49 29 29 (Hyderabad)
9121 41 29 29 / 9121 44 29 29 (Bangalore)
https://t.me/pdf4exams https://t.me/allupscmaterials

4.3 Skill development programmes for Farmers and Women


Highlights:
 Ministry of Skill Development and Entrepreneurship, under Skill India Mission is implementing schemes
for imparting employable skills to youth through short term and long term program on pan-India basis in
rural and urban areas.
 Pradhan Mantri Kaushal Vikas Yojana (PMKVY) 2.0 implemented by Ministry of Skill Development and
Entrepreneurship (MSDE) provides short term training to youth across the country including farmers and
women in rural areas.
 Three kinds of training routes under the scheme:
o Short term Training for fresh skilling of school / college drop outs and unemployed youth.
o Recognition of Prior Learning to recognize the existing skills.
o Special Projects to address skilling requirements of groups which are underrepresented or
require special initiatives and efforts.
 Directorate General of Training (DGT), Ministry of Skill Development and Entrepreneurship (MSDE) is
entrusted with the responsibility of long term vocational training in the Country.
 One of the schemes is ‘Craftsmen Training Scheme’ being implemented through network of Industrial
Training Institutes located all over the country.
 The objective is to provide skilled work force to the industry.
5. Government Initiative
5.1 Three years of DeMO
Context: Three years since Demonetisation, the level of cash with the public has grown faster than the GDP
growth of the country.
Highlights:
 Digital payments via UPI, debit cards, mobile banking, and prepaid instruments has seen robust growth
over this three-year period.
 RBI data shows that the public held 20.49 lakh crores in cash as of September 2019, which is 13.3% more
than that of 2018.
 The data shows that the cash held by the public made up 96% of the money in circulation, with most of
the rest deposited in banks.
What is demonetization?
Demonetization is the act of stripping a currency unit of its status as legal tender. It occurs whenever there is a
change of a National currency. The current form or forms of money is pulled from circulation and retired,
often to be replaced with new notes or coins. Sometimes, a country completely replaces the old currency with
new currency.
Why was demonetization done?
(As per the reasons given by the Government)
 To tackle the menace of Black money/Parallel economy/Shadow economy.
 The cash circulation in India is directly connected to corruption hence we want to reduce the cash
transactions and also control corruption and thereby move towards cashless transactions.
 To counter the menace of counterfeit currency.
 To prevent the cash being used for terrorist activities/terror funding.
Cash is the king
In India majority of the transactions are done in the form of cash.
 As per RBI, 87% of the transactions in India are cash transactions.

La Excellence IAS 38 Website: www.laex.in/testprep


9052 29 29 29 / 9052 49 29 29 (Hyderabad)
9121 41 29 29 / 9121 44 29 29 (Bangalore)
https://t.me/pdf4exams https://t.me/allupscmaterials

 As per RBI report, debit cards at ATMs account for 88% and 94% (by volume and value respectively) of the
debit card transactions and 12% and 6% account for Point of Sale (POS) transactions.
 The infrastructure growth is slow – The POS machines and ATMs are 1.2 million (and there are around 14
million merchants in India, in essence more than 90% of the merchants are not using the POS machines)
and 0.19 million respectively (From 2013 to 2015, ATMs increased by 43% and POS machines by 28%).
Reasons
 ATMs and POS machines are concentrated in urban areas.
 Penetration in non-urban areas is very poor and there is also connectivity issues.
 Even if the POS machines are installed, low value transactions are discouraged by the merchants.

Pros of Demonetization
 The menace of black money controlled to some extent.
 Terror financing, using black money for illegal activities etc. took a hit.
 The counterfeit currencies which have an impact on the real economy, was rooted out to some extent.
 It’s a major step by the government towards forming a cashless economy.
 Many shell companies have been unearthed.
Cons of Demonetization:
 For one all the black money is not stored in the form of cash only and secondly the measure takes care of
result but not the cause-black money is generated mainly because of corruption and tax evasion. This
measure controls the usage of black money but cannot control the causes.
 Panic amongst the common public (already we have seen the case wherein people have looted fair price
shop in MP, Cash Carrying companies seeking higher insurance etc) led to people hoarding currencies
which has further reduced the liquidity in the market.
 The small trade/shopkeepers are facing difficulties.
 Black marketing of the new notes/currencies is on the rise.
 The establishments such as banks, hospitals etc. are under lot of stress.
 Another area which is a cause of worry is the likely drop in the rural demand as the cash usage will
become restricted. Apart from this the experts are also expecting an impact on SME sector, agricultural
production (the economy was expected to perform well as there was an expectation of a good Rabi crop
after two bad monsoons but a prominent economist, Pronab Sen has said that demonetization is akin to
third bad monsoon year as it will have an impact on agricultural production, but the more dangerous
situation is this having a spillover effect on to fertilizer, tractor sectors).

La Excellence IAS 39 Website: www.laex.in/testprep


9052 29 29 29 / 9052 49 29 29 (Hyderabad)
9121 41 29 29 / 9121 44 29 29 (Bangalore)
https://t.me/pdf4exams https://t.me/allupscmaterials

Challenges
 The coverage of the banking sector-
 Only 27% of the villages have a bank within 5 kms (as per Economic Survey 2015-16)
 In spite of record-breaking implementation of PM Jan Dhan Yojana(JDY), the banking penetration is low-
on an average 46% in all the states (as per Economic Survey 2015-16).
 Another challenge in implementing and eradicating black money would be presence of an informal
economy. It accounts for 45% of GDP and 80% of employment hence this move had a greater impact on
informal economy.
 Logistics and cost challenges of replacing all the Rs 500 and Rs 1000 notes – as per the RBI documents
this measure costs at least Rs. 12000 crores as it has to replace over 2300 crore pieces of these
currencies.
 The decision to issue Rs 2000 denomination currency and withdrawal of Rs 500 and Rs 1000 currency led
to huge challenge as most of the day to day transactions in India are centered around Rs 500 note (more
than 47% of the value of notes in circulation is in Rs 500 note form).
 The availability of Rs 500 and Rs 1000 notes was the biggest challenge as both of them covered over 85%
in terms of value of total currencies issued.

Do You Know?
Demonetisation has been implemented twice before 2016 –in 1946 and 1978.

Mains question
All said and done, when dust has settled, demonetization has not achieved the objectives which was first
intended for it. Critically analyze (250W|15M)

5.2 Sabka Vishwas – Legacy Dispute Resolution Scheme


What is Sabka Vishwas (Legacy Dispute Resolution) Scheme, 2019?
The Sabka Vishwas Scheme, 2019 is a scheme proposed in the Union Budget, 2019, and introduced to resolve
all disputes relating to the erstwhile Service Tax and Central Excise Acts, which are now subsumed under GST,
as well as 26 other Indirect Tax enactments (as listed below). The scheme will be for taxpayers who wish to
close their pending disputes, with a substantial relief provided by the government.
From when will this scheme become operational?
The scheme has been notified by the CBIC to come into force on the 1st of September, 2019, and shall be
operational until the 31st of December, 2019.
There will be certain cases excluded under this scheme-
 Cases in respect of excisable goods set forth in the Fourth Schedule to the Central Excise Act, 1944 (this
includes tobacco and specified petroleum products)
 Cases for which the taxpayer has been convicted under the Central Excise Act, 1944 or the Finance Act,
1944
 Cases involving erroneous refunds
 Cases pending before the Settlement Commission
What are the benefits of this scheme?
This scheme offers several lucrative resolution benefits to taxpayers, such as-
1. Taxpayers can pay the outstanding tax amounts due and be free from any other consequences under
the Law.
2. Taxpayers will get substantial relief in the form of full waivers of interest, penalties and fines.
3. There will be complete amnesty from prosecution proceedings.
La Excellence IAS 40 Website: www.laex.in/testprep
9052 29 29 29 / 9052 49 29 29 (Hyderabad)
9121 41 29 29 / 9121 44 29 29 (Bangalore)
https://t.me/pdf4exams https://t.me/allupscmaterials

What is the relief provided under this scheme?


This scheme provides for a substantial relief margin on all Duty demands, ranging from 40% to 70% of the
demand, except in the case of voluntary disclosure. The relief will be applicable as follows-

For Cases of Confirmed Duty For Cases of


Relief Available from For Cases Pending
Demands (Where No Appeal is Voluntary
the Duty Demand Adjudication or Appeal*
Pending) Disclosure

Duty demands up to 70% 60%


Rs.50 lakh The full amount of
duty disclosed
Duty demands >Rs.50 50% 40%
lakh

6. In news
6.1 Harmonised system of Code
The Ministry of Commerce and Industry allocated a separate Harmonised System (HS) code for
Khadi. The move is expected to boost Khadi exports in the coming years. Earlier, Khadi did not have
its exclusive HS code.
ABOUT HS CODE
 The Harmonized Commodity Description and Coding System generally referred to as “Harmonized
System” or simply “HS” is a multipurpose international product nomenclature developed by the World
Customs Organization (WCO).
 It is called the “universal economic language” for goods, it is a multipurpose international product
nomenclature.
 It is a unique six-digit code has numbers arranged in a legal and logical structure, with well-defined rules
to achieve uniform classification.
 Of the six digits, the first two denote the HS Chapter, the next two give the HS heading, and the last two
give the HS subheading.
Significance
 Over 200 countries use the system as a basis for their customs tariffs, gathering international trade
statistics, making trade policies, and for monitoring goods.
 The system helps in harmonizing of customs and trade procedures, thus reducing costs in international
trade.

La Excellence IAS 41 Website: www.laex.in/testprep


9052 29 29 29 / 9052 49 29 29 (Hyderabad)
9121 41 29 29 / 9121 44 29 29 (Bangalore)
https://t.me/pdf4exams https://t.me/allupscmaterials

D. ENVIRONMENT
1. Ecosystem-Biodiversity
1.1 Sunderbans mangroves
Context: scientists, wildlife experts and local NGOs have been highlighting the constant degradation of the
mangrove forest in the Sunderbans, particularly in areas that are inhabited.
About sunderbans:
 The Sundarbans comprises hundreds of islands and a network of rivers, tributaries and creeks in the delta
of the Ganga and the Brahmaputra at the mouth of the Bay of Bengal in India and Bangladesh.
 The Indian Sundarbans constitutes over 60% of the country’s total mangrove forest area.
 It is the 27th Ramsar Site in India, and is now the largest protected wetland in the country.

Significance:
 Home to the Royal Bengal Tiger. What is a "mangrove" forest?
 Home to a large number of “rare and globally threatened species,  Mangroves are a group of trees
such as the critically endangered northern river terrapin (Batagur and shrubs that live in
baska), the coastal intertidal zone.
 The endangered Irrawaddy dolphin (Orcaella brevirostris), and the  There are about 80 different
vulnerable fishing cat (Prionailurus viverrinus). species of mangrove trees. All of
 Due to dense foliage and the close proximity of trees, the these trees grow in areas with
mangrove roots hold soil and the vegetation becomes a shield to low-oxygen soil, where slow-
protect from cyclones. moving waters allow fine
Environmental issues: sediments to accumulate.
 Over four million people live on its northern and northwestern  Mangrove forests only grow at
periphery, putting pressure on the ecosystem. tropical and subtropical
 Concerns have been raised about natural ecosystems being latitudes near the equator
changed for cultivation of shrimp, crab, molluscs and fish. because they cannot withstand
 The Ramsar Information Sheet lists fishing and harvesting of freezing temperatures.
aquatic resources as a “high impact” actual threat to the
wetland.
 The other threats are from dredging, oil and gas drilling, logging and wood harvesting, hunting and
collecting terrestrial animals.
 Salinity has been categorised as a medium and tourism as a low impact actual threat in the region.
 Along with anthropogenic pressures, it is also vulnerable to climate change and requires better
management and conservation practices.
Rapid destruction:
 A satellite image from the ISRO pointed to a loss of 3.71% mangrove and non-mangrove forest cover
along with massive erosion of the archipelago’s landmass.
La Excellence IAS 42 Website: www.laex.in/testprep
9052 29 29 29 / 9052 49 29 29 (Hyderabad)
9121 41 29 29 / 9121 44 29 29 (Bangalore)
https://t.me/pdf4exams https://t.me/allupscmaterials

 The analysis, based on satellite data of February 2003 and February 2014, shows that while a 9,990-
hectare landmass has been eroded, there has been an accretion of 216-hectare landmass in the
Sunderbans during the period.
 Mangroves have been cut not only for aquaculture, and for fisheries, but also for building embankments
and for human settlements.
 The State govt. had allocated houses under the ‘Banglar Abas’ scheme by clearing acres of mangrove
forest on Sagar island.

Do You Know?
Sundarban Jungle has been named after the large mangrove trees Sundari (Heritiera littoralis).
Sundarban has unique Tidal Phenomenon twice a day – there are high tides when water level seems to rise
around 6-10 feet and low tides when one can see the huge mud land area lying flat.
Mains question
What is a ‘Mangrove forest’? Discuss the factors that are leading to their destruction in India. List some
measures undertaken to prevent this degradation both nationally and internationally (250W|15M)

1.2 Butterfly survey in Western Ghats:


Context:
Three-day survey in Western Ghats finds 191 species of the insect, 12 of which are endemic to the
biodiversity-rich region
Highlights of the survey:
 The survey was done jointly by the Forest and Wildlife Department in association with the Ferns Nature
Conservation Society (FNCS).
 The degradation of the riparian forest in many part of the region may adversely affect the butterfly habitat
in the near future.
Key findings:
 The first-time sighting of Silver forget me not, Common three ring, and Brown onyx was recorded. The
sighting of Silver forget me not was reported only from the Chinnar Wildlife Sanctuary in Idukki district of
the State.
Survey objective:
 As the study on butterflies was also the study on nature and climate change, the survey report would help
the Forest Department to prepare a forest management plan in the region in the coming years.
Significance of butterflies
 Butterflies are great bio-indicators of an ecosystem as they are highly sensitive to environmental
conditions such as temperature, sunlight, humidity and rainfall patterns.
 Their presence, patterns and migration assist in mapping the climatic health of a region and are they are
perhaps the most studied insect group across the world.

Do You Know?
Western Ghats has an exceptionally high level of biological diversity and endemism and is recognized as
one of the world’s eight ‘hottest hotspots’ of biological diversity.
The forests of the site include some of the best representatives of non-equatorial tropical evergreen
forests anywhere and are home to at least 325 globally threatened flora, fauna, bird, amphibian, reptile
and fish species.

La Excellence IAS 43 Website: www.laex.in/testprep


9052 29 29 29 / 9052 49 29 29 (Hyderabad)
9121 41 29 29 / 9121 44 29 29 (Bangalore)
https://t.me/pdf4exams https://t.me/allupscmaterials

1.3 Avian Botulism


Context:
The Indian Veterinary Research Institute (IVRI), Bareilly, confirmed avian botulism is the reason for mass
mortality of birds, including migratory species from Northern Asia, at Sambhar Lake in Rajasthan.
What is the issue?
 Thousands of migratory birds have been found dead at Sambhar Lake, Rajasthan and officials have buried
over 18,000 carcasses so far.
 While there is no clarity yet on what has caused the deaths, investigations so far suggest avian botulism.
What is avian botulism?
 Avian botulism is a neuromuscular illness of waterfowl caused by a bacterial infection.
 This deadly bacterial infection paralyses the birds, making them unable to walk, swim or fly. The birds
are unable to hold up their necks, which droop.
 The infection releases a toxin in the body that finally kills the bird. When an infected bird dies, the
maggots that feed off it become infected. These maggots are in turn consumed by additional birds,
causing a massive outbreak. The cycle continues.
About Sambhar Lake:
 Located 80 km south-west of Jaipur, Sambhar Lake is India's largest inland saline water body
 It has been designated as a Ramsar Site and a wetland of international importance, attracting
thousands of migratory birds during winter.
 The ecosystem of the lake is facing grave threats
following government apathy and over-exploitation
of groundwater by commercial salt manufacturers.
 As a result, there has been a substantial decline in
the inflow of water to the lake, exacerbated
by low rainfall and construction of dams and check-
dams.
 The 4th Strategic Plan 2016-2024 of the Ramsar
Convention visualizes that the wetlands are
conserved, wisely used, restored and their benefits
are recognized and valued by all.
 It may be time for the Ramsar Advisory Mission to
get its act together and intervene for Sambhar as it
did for the Keoladeo wetland in 1988.

1.4 PLIOSAUR
Pliosaur bones have been found recently in Poland
 pliosaurs are the predatory reptiles that
lived in seas over 150 million years ago.
 They are one of the largest aquatic
carnivorous reptiles that have ever lived and
are often referred to as sea monsters.
 They had powerful large skulls and massive
jaws with large sharp teeth
 Scientifically, they are placed in the suborder
Pliosauroidea, whose members are called
pliosaur.

La Excellence IAS 44 Website: www.laex.in/testprep


9052 29 29 29 / 9052 49 29 29 (Hyderabad)
9121 41 29 29 / 9121 44 29 29 (Bangalore)
https://t.me/pdf4exams https://t.me/allupscmaterials

1.5 Idris Elba


Idris Elba is a wasp species which is named after the actor.
Description:
 Now, scientists have given the name Idris elba to a species of wasp that is again a protector of crops.
Idris Elba
 The wasp, recently discovered in Mexico, was found living as a parasite in the eggs of another insect,
known as the bagrada bug, which is a major pest of cruciferous vegetables.
 While other species of the Idris genus were known to only parasitise spider eggs, specimens of Idris elba
have now been found to emerge from eggs of the bagrada bug.

1.6 Paris bans wild animals from circuses


Amid ongoing concerns of animal cruelty, Paris outlawed the use of wild animals in circuses in the city.
Description:
 France, meanwhile, is still considering enforcing a nation-wide ban on the use of wild animals to perform
under the big top.
 According to proposals that have been set in motion and will be enacted from 2020, operating permits
within the city of Paris will be withheld from circuses that continue to use wild animals in violation of the
ban.
 According to data by the Animal Defenders International (ADI), an animal rights group that monitors the
use of animals for human entertainment, most European countries have nationwide bans on wild
animals in circuses.
 According to the ADI, France, Germany, Spain, the UK, the US, Canada, Argentina, Brazil & Australia are
the nations that presently have only local bans.
India’s stance on wild animals in circuses
 Under section 38 of the Prevention of Cruelty to Animals Act, 1960 (59 of 1960) MoEFCC proposed the
‘Performing Animals (Registration) (Amendment) Rules, 2018’.
 This issued a “prohibition on exhibiting and training of animals for specified performances.”
 The draft rules added that under these proposed provisions, “no animals shall be used for any
performances or exhibition at any circus or mobile entertainment facility.”
 In 2016, the Central Zoo Authority (a national government body that oversees the conditions of animals
used in circuses and entertainment) cancelled recognition it had granted to 21 circuses across the
country following reports of rampant abuse of animals.

2. Pollution
2.1 Delhi Pollution
Context: The Supreme Court sought to know how effective the Delhi government’s odd-even scheme which
ended recently.
Highlights:
 This is the third time the scheme has been implemented — the first two times were in 2016 — and all
three times, its effectiveness has been hotly debated.
 The average AQI in Delhi and the three NCR cities was calculated for the 12 days before odd-even was
implemented, and the 12-day period it was in place
 Delhi is the most closely monitored city in the country with the AQI value calculated based on 28-36
stations daily, as compared to 10 in Mumbai, which is the second highest.

La Excellence IAS 45 Website: www.laex.in/testprep


9052 29 29 29 / 9052 49 29 29 (Hyderabad)
9121 41 29 29 / 9121 44 29 29 (Bangalore)
https://t.me/pdf4exams https://t.me/allupscmaterials

Reasons for Deteriorating Air Quality of Delhi


Stubble Burning During the month of October-November is crop burning by the farmers in Haryana,
Punjab and UP.
It is estimated that approximately 35 million tonnes of crop are set afire by these
states. The wind carries all the pollutants and dust particles, which have got locked
in the air.

Vehicular Emission Delhi has more than 9 million registered vehicles. The Central Pollution Control
Board (CPCB) and the National Environmental Engineering Research Institute
(NEERI) have declared vehicular emission as a major contributor to Delhi’s
increasing air pollution.

Weather During the winter season, dust particles and pollutants in the air become unable to
move. Due to stagnant winds, these pollutants get locked in the air, resulting in
smog.

High Density With more than 11000 people per square kilometer, Delhi is among the most
densely populated cities in the world. Over-population adds up to the various
types of pollution.

Lack of Infrastructure In India, investment in public transport and infrastructure is low which leads to
congested roads, and hence air pollution.

Construction Activities Large-scale construction in Delhi-NCR is another culprit that is increasing dust and
and Open waste pollution in the air. Delhi also has landfill sites for dumping of waste, burning of
burning waste.

Thermal Power Plant Industrial pollution and garbage dumps are also increasing air pollution and
and Industries building-up smog in the air.

Firecrackers Despite the ban on cracker sales in 2017, firecrackers were a common sight in
Diwali. It is the major reason for smog in Delhi after Diwali.

Diesel generators A large number of housing societies and businesses resort to using diesel
generators as alternate for power supply during cuts. Diesel generators contribute
approximately 15 percent to the city’s air pollution.

Dust Storm from Gulf The dust storm from Gulf countries was also the reason which enhanced already
countries worse condition.

What is GRAP?
 The GRAP was formulated in 2016 and approved by the Supreme Court in the same year.
 It was notified in 2017 by the Centre, and draws its authority from this notification.
 It was planned after several meetings of EPCA (Environment Pollution (Prevention and Control) Authority)
with state government officials and experts.
 GRAP institutionalized the measures to be taken when air quality deteriorates.
 It aims to roll out progressively tougher actions as pollution levels rise, without waiting for an emergency
to impose strict measures.

La Excellence IAS 46 Website: www.laex.in/testprep


9052 29 29 29 / 9052 49 29 29 (Hyderabad)
9121 41 29 29 / 9121 44 29 29 (Bangalore)
https://t.me/pdf4exams https://t.me/allupscmaterials

How does GRAP work?


 The plan is incremental in nature.
 It does not include action by various state governments to be taken throughout the year to tackle
emissions.
 The plan requires action and coordination among 13 different agencies in Delhi, Uttar Pradesh, Haryana
and Rajasthan (NCR areas).
 At the top is the EPCA, mandated by the Supreme Court. Before the imposition of any measure, EPCA
holds a meeting with representatives from all NCR states.
 A call is taken on which actions have to be made applicable in which town.
 When the air quality shifts from poor to very poor, the measures listed have to be followed.
Ambient Particulate Matter Measures
Category
(PM) Concentration

 Enforce pollution control in


 PM 2.5 between 61-120 thermal power plants
µg/m3  Mechanized sweeping on
Moderate to Poor
 PM10 between 101-350 roads
µg/m3  Ban on firecrackers
 Stop garbage burning

 Stop use of diesel generator


sets
 PM2.5 between 121-
 Increase bus and metro
250µg/m3
Very Poor services and increasing
 PM10 between 351-430
frequency of metro service
µg/m3
 Stop use of coal/firewood
in hotels and open eateries

 Increase frequency of
mechanized sweeping of
road and sprinkling of
water on roads
 PM2.5 more than  Close brick kilns, Hot Mix
Severe 250µg/m3 plants, Stone Crushers
 PM10 more than 430µg/m3  Shut down Badarpur power
plant
 Introduce concessional
rates to encourage off-peak
travel in public transport.

La Excellence IAS 47 Website: www.laex.in/testprep


9052 29 29 29 / 9052 49 29 29 (Hyderabad)
9121 41 29 29 / 9121 44 29 29 (Bangalore)
https://t.me/pdf4exams https://t.me/allupscmaterials

 Stop entry of diesel trucks


 PM2.5 of or more than into Delhi (except essential
300µg/m3 commodities)
Severe+ or Emergency  PM10 of or 500µg/m3  Stop construction activities
 (persist for 48 hours or  Introduce odd and even
more) scheme
 Shutting of schools

Environment Pollution (Prevention and Control) Authority.


 A statutory body constituted by the central government in 1998 at the behest of the Supreme Court.
 EPCA is Supreme Court mandated body tasked with taking various measures to tackle air pollution in
the National Capital Region. It was notified in 1998 by Environment Ministry under Environment
Protection Act, 1986.
 One of the initial and significant achievements of this authority was introducing compressed natural gas
as a fuel for vehicles in Delhi after an order of the Supreme Court.
 The EPCA was constituted with the objective of ‘protecting and improving’ the quality of the
environment and ‘controlling environmental pollution’ in the National Capital Region.
 The EPCA also assists the apex court in various environment-related matters in the region.
 To enforce Graded Response Action Plan (GRAP) in NCR as per the pollution levels.

Do You Know?
Delhi Pollution claims the lives of more than 10000 people every year.
Living in Delhi is said to be equal to smoking twenty plus cigarettes every single day.

Mains question
The deteriorating Air pollution in New Delhi poses various health hazards. Elucidate. Analyze the
reasons why the government has not been unable to confront this challenge. (150W|10M)

2.2 Health emergency declared in the Capital


Hospitals report rise in respiratory and eye problems as air quality enters ‘severe plus’ category
 The Environment Pollution (Prevention and Control) Authority declared a public health emergency in the
Capital as pollution levels entered the ‘severe plus’ category.
 According to the official data provided by the Central Pollution Control Board (CPCB), the overall AQI score
of Delhi was 504.
Steps were taken by the government to tackle pollution
 To tackle Stubble burning
 The government is giving subsidy to farmers for buying Turbo Happy Seeder (THS) which is a machine
mounted on a tractor that cuts and uproots the stubble and also drill wheat seeds.
 Punjab and Haryana procure stubble and straw for biofuel plants and in the waste-to-energy power
generation plants.
Graded Response Action Plan (GRAP):
 The government has implemented GRAP to tackle with the rising pollution in Capital. It includes the
measures like shutting down Badarpur Thermal Power Plant and a ban on construction activities.
 Recently, the government has also launched a pollution forecast system for New Delhi.
La Excellence IAS 48 Website: www.laex.in/testprep
9052 29 29 29 / 9052 49 29 29 (Hyderabad)
9121 41 29 29 / 9121 44 29 29 (Bangalore)
https://t.me/pdf4exams https://t.me/allupscmaterials

 The central government released the National Air Quality Index (AQI) for public information under the
aegis of the Central Pollution Control Board.
 AQI has been developed for eight pollutants— PM2.5, PM10, Ammonia, Lead, nitrogen oxides, sulfur
dioxide, ozone, and carbon monoxide.
 The April 2020 deadline for the country to implement BS-VI (Bharat Stage-VI is the Indian equivalent to
Euro VI) grade fuels was advanced for Delhi to April 1, 2018, because of the extremely high levels of air
pollution in the city.
 Construction of Eastern and Western Expressways for by-passing non-destined traffic to Delhi.
 A ban on burning of leaves/ biomass in Delhi.
 Prohibition on the entry of overloaded and non-destined trucks in Delhi and imposition of ‘Green Tax’.
 Out of 2800 major industries, 920 industries have installed on-line continuous (24x7) air pollution
monitoring devices; others are in process of installation.
 The System of Air Quality and Weather Forecasting And Research (SAFAR)
 The System of Air Quality and Weather Forecasting And Research (SAFAR) is a national initiative
introduced by the Ministry of Earth Sciences (MoES) to measure the air quality of a metropolitan city,
by measuring the overall pollution level and the location-specific air quality of the city.
 The system is indigenously developed by the Indian Institute of Tropical Meteorology (IITM), Pune and
is operationalized by the India Meteorological Department (IMD).
 It has a giant true color LED display that gives out real-time air quality index on a 24  7 basis
with color-coding (along with 72 hours advance forecast).
 The ultimate objective of the project is to increase awareness among the general public regarding the
air quality in their city so that appropriate mitigation measures and systematic action can be taken up.
o It organizes awareness drive by educating the public (prompting self-mitigation), and
o It also helps the policy-makers to develop mitigation strategies keeping in mind the
nation’s economic development.
 SAFAR is an integral part of India’s first Air Quality Early Warning System operational in Delhi.
o It monitors all weather parameters like temperature, rainfall, humidity, wind speed, and
wind direction, UV radiation, and solar radiation.
o Pollutants monitored: PM2.5, PM10, Ozone, Carbon Monoxide (CO), Nitrogen Oxides
(NOx), Sulfur Dioxide (SO2), Benzene, Toluene, Xylene, and Mercury.

2.3 ICAR report on Stubble Burning


Context:
On 4th November, 2019, the ICAR’s Creams Laboratory bulletin stated that there has been reduction of 12% in
incidents of Stubble Burning as compared to the same period in 2018.
Highlights of the report:
 There were 31,402 burning events in the three states Punjab, Uttar Pradesh and Haryana as on October 1,
2019.
 The state wise reduction in stubble burning as compared to 2018 is as follows
 Uttar Pradesh-48.2%
 Haryana-11.7%
 Punjab-8.7%
 The report says that the reduction is mainly due to the launch of Central Scheme “Promotional of
Agricultural Mechanization of In-Situ Management of Crop Residue in Punjab, UP and Haryana and NCT of
Delhi”.
 The main purpose of the scheme is to subsidize machinery required.
Impact on national capital:
La Excellence IAS 49 Website: www.laex.in/testprep
9052 29 29 29 / 9052 49 29 29 (Hyderabad)
9121 41 29 29 / 9121 44 29 29 (Bangalore)
https://t.me/pdf4exams https://t.me/allupscmaterials

 New Delhi is worst affected due to stubble burning in the three states. This year (2019) the Air quality
Index of Delhi reached 999.
 This is the highest recorded AQI of Delhi in its history. According to WHO norms the AQI should be
under 100.
 Also, the PM 2.5 (Particulate Matter) concentration was more than 80% as against 50% of the normal
course.
Role of the Western Disturbances:
 The Stubble burning smoke reaching Delhi is greatly influenced by the Western Disturbances.
 When these Cyclonic circulations are strong and persistent the smoke deposited in the city decreases.
 The disturbances are capable of carrying the smoke far and spread them wide when they are strong.
 The weakening of these disturbances lately due to global warming is also one of the reasons for the
increased stubble smoke in Delhi.
Supreme court order:
 Supreme Court sets three-month deadline for Centre to prepare comprehensive national scheme, to
address the stubble burning issue.
 Comprehensive national scheme, in consultation with States, to wean small and marginal farmers away
from stubble burning, which has been identified as a major source of air pollution choking the national
capital.
 Punishing poor farmers for burning stubble is not a solution.
 States to distribute crop residue management tools and machines to small and marginal farmers free of
cost.
 The States need to use their own funds to finance the scheme.
 The court would take a decision on the sharing of financial liability between the Centre and the States.
 An incentive of Rs. 100 per quintal for small and marginal farmers who engage in the management of the
residue of their non-Basmati variety rice crop in Punjab, Haryana and Uttar Pradesh.
 The Delhi government has to submit an action plan to deal with issues of garbage dumping and burning,
pock-marked roads and the state of traffic congestion in the capital city.
What are the Solutions to the burning problem?
 In 2014, the Union government released the National Policy for Management of Crop Residue. Since then,
crop residue management has helped make the soil more fertile, thereby resulting in savings of Rs
2,000/hectare from the farmer’s manure cost.
 Farmers can also manage crop residues effectively by employing agricultural machines like:
 Happy Seeder (used for sowing of crop in standing stubble)
 Rotavator (used for land preparation and incorporation of crop stubble in the soil)
 Zero till seed drill (used for land preparations directly sowing of seeds in the previous crop
stubble)
 Baler (used for collection of straw and making bales of the paddy stubble)
 Paddy Straw Chopper (cutting of paddy stubble for easily mixing with the soil)
 Reaper Binder (used for harvesting paddy stubble and making into bundles)
 On other hand, these machines are too costly and the state governments should come forward and
provide better subsidy so that farmer can afford these machines
 A provision of Rs 1,151.80 core for two years has been made under this scheme for states like
Punjab,Haryana, Uttar Pradesh and the National Capital Region.

Mains question
Stubble burning is not just a pollution problem but also an agricultural problem. Comment.
(150W|10M)

La Excellence IAS 50 Website: www.laex.in/testprep


9052 29 29 29 / 9052 49 29 29 (Hyderabad)
9121 41 29 29 / 9121 44 29 29 (Bangalore)
https://t.me/pdf4exams https://t.me/allupscmaterials

2.4 Indian Air Quality Interactive Repository (IndAIR)


Context:
National Environmental Engineering Research Institute (NEERI) along with the Council of Scientific and
Industrial Research (CSIR) has launched India’s first web repository named IndAIR documenting air quality
studies done in the last 60 years.
About IndAIR
 The aim of the project is to make air quality research available to everyone.
 IndAIR has archived approximately 700 scanned documents from pre-Internet times (1950-
1999), 1,215 research articles, 170 reports and case studies, 100 cases and over 2,000 statutes to provide
the history of air pollution research and legislation in the country.
 This includes all major legislation in the country dating back to 1905.
 The repository will be the first comprehensive effort to inventorise surviving Indian research and analysis
on air pollution, its causes and effects and present these studies in an easily accessible web format for the
media, researchers and academics.

Significance
 IndAIR was initiated with the intent to document important milestones in the country and make them
available to the public.
 The repository will help the academicians understand the issues better and also enable policy makers to
frame legislation that encourages development.
CSIR-NEERI:
 The CSIR-National Environmental Engineering Research Institute (CSIR-NEERI) is a research institute
which was established in Nagpur in 1958 as Central Public Health Engineering Research Institute
(CPHERI) with focus on water supply, sewage disposal, communicable diseases and to some extent on
industrial pollution and occupational diseases found common in post-independent India.
 NEERI falls under the Ministry of Science and Technology of the Central government.

La Excellence IAS 51 Website: www.laex.in/testprep


9052 29 29 29 / 9052 49 29 29 (Hyderabad)
9121 41 29 29 / 9121 44 29 29 (Bangalore)
https://t.me/pdf4exams https://t.me/allupscmaterials

3. Geographical phenomenon
3.1 Cyclone Maha batters Lakshadweep and Kerala coast
The Lakshadweep Islands and, to a relatively lesser extent, the coastal districts of Kerala on Thursday bore
the brunt of Cyclone Maha
A depression that developed in the Comorin Sea (near the southernmost end of India) has intensified into a
cyclonic storm, named ‘Maha’.
 The name has been given by Oman.
 It lies centered over Lakshadweep and the adjoining southeast Arabian Sea and the Maldives area.
 Cyclone storm ‘Maha’ is the second in the 2019 northeast monsoon season, following close on the heels
of super cyclone Kyarr which took its way towards the Arabian Peninsula.

Mains question
There has been a growing tendency of increase in the occurrence of cyclones in the Arabian Sea.
Discuss the factors contributing to this increase. (150W|10M)

3.2 Cyclone ‘Bulbul’


Context:
IMD has warned that the cyclonic storm ‘Bulbul’ over the east-central Bay of Bengal likely to bring heavy rain
to Odisha.
Highlights:
 Even as Cyclone Maha weakens in the Arabian Sea, another cyclonic storm is building in the Bay of Bengal.
 The IMD said that a Depression in the BoB has already intensified into a Deep Depression and is expected
to turn into a cyclonic strom.
 The system is on course to move towards the coasts of West Bengal, Odisha and Bangladesh.
What is Cyclone?
In meteorology, the term cyclone can be defined as the rapid inward circulation of air masses about a low
pressure centre which is circling counter-clockwise in the northern hemisphere and clockwise in the southern
hemisphere.
There are factors responsible for the formation of cyclone:
 Sufficient warm temperature at sea surface
 atmospheric instability
 impact area of Coriolis force so that low pressure can be develop
 high humidity in the lower to middle levels of the troposphere
 a pre-existing low-level focus or disturbance
 low vertical wind shear.

La Excellence IAS 52 Website: www.laex.in/testprep


9052 29 29 29 / 9052 49 29 29 (Hyderabad)
9121 41 29 29 / 9121 44 29 29 (Bangalore)
https://t.me/pdf4exams https://t.me/allupscmaterials

How does a cyclone formed?


 Cyclones are formed in low pressure area. The topography and the intensity as well as frequency of
cyclones that could strike a coast decide the vulnerability of the place.
 The temperature difference between the warm, rising and the cooler environment led to the rise of air
to become buoyant and then moves to upward.
 Then the high pressure area fills the air in the low pressure area. This cycle continues as warm air rises
and low pressure area filled with cool air.
 They build up over a period of time. The warm, moist air rises and cools the water in the air and forms
clouds.
 The whole system of clouds and wind spins and grows, fed by the ocean’s heat and water evaporating
from the ocean surface.

Do you know?
Cyclones are named as per guidelines decided by the World Meteorological Organisation (WMO). The
WMO says that countries in the affected region should name the cyclones.
 In the north Indian Ocean region, eight countries decide the names of cyclonic storms. These countries
include India, Bangladesh, Maldives, Myanmar, Oman, Pakistan, Sri Lanka and Thailand.
 While the names are suggested by the countries, the final identification is done by New Delhi-based
Regional Specialised Meteorological Centre (RSMC).
 The first cyclone to be named as per the system was ‘Onil’, which originated in the Arabian Sea between
September and October 2004. The cyclone made landfall on the Gujarat coast.

3.3 DANKALI DEPRESSION


A recent study stated that an active and naturally occurring life cannot be sustained at Danakil, Ethiopia.
DESCRIPTION
 Microbes are known to survive almost anywhere. Scientists now believe that Danakil depression in
Ethiopia is an exception
 New research has pointed out that bubbling pools of water and mounds of salt covering its landscape —
that is too daunting even for these microorganisms.
LOCATON:
 The Danakil Depression is the northern part of the Afar Triangle or Afar Depression in Ethiopia, a
geological depression that has resulted from the divergence of three tectonic plates in the Horn of Africa.
 It is about 125 m (410 ft) below sea level and is bordered to the west by the Ethiopian Plateau and to the
east by the Danakil Alps, beyond which is the Red Sea.
 The area is often referred to as the cradle of humanity
 It is the hottest place on Earth in terms of year-round average temperatures

La Excellence IAS 53 Website: www.laex.in/testprep


9052 29 29 29 / 9052 49 29 29 (Hyderabad)
9121 41 29 29 / 9121 44 29 29 (Bangalore)
https://t.me/pdf4exams https://t.me/allupscmaterials

3.4 Acqua alta


High tides in the Adriatic Sea have caused floods in the historic city of Venice.
Description:
Late autumn and winter are the season for high tides or Acqua alta in Venice. Recently, over 75% of the
lagoon city went under water after high tides and stormy weather led to an increase in the water levels of the
canals.
 Background: Since 1991, Venice has been awaiting a multi-billion dollar flood barrier system called MOSE
(acronym for Experimental Electromechanical Module) that has been under construction since 2003, and
was scheduled to be completed in 2014.
Acqua alta
 “Acqua alta” is the name given to exceptionally high tides in the Adriatic Sea, which reached heights of
1.87 metres (well over 6 feet) — only a little short of the 1.91-metre record that was set during the
“great flood” of 1966.
 Adriatic Sea: The Adriatic Sea is part of the Mediterranean Sea separating the Italian Peninsula from the
Balkan Peninsula.
 The sea's western coast runs the length of Italy, while the eastern coast forms the borders
of Croatia, Bosnia and Herzegovina, Montenegro, and Albania.

4. Government initiatives
4.1 Cloud seeding
Context:
Haryana Deputy Chief Minister has written to PM, requesting to “undertake cloud seeding plan to combat the
air pollution engulfing Delhi and NCR”.
What is cloud seeding?
 Cloud seeding is a kind of a weather modification technology to create artificial rainfall.
 It is also known by other terms such as man-made precipitation enhancement, artificial weather
modification, rainmaking and so on.
 It works only when there is enough pre-existing clouds in the atmosphere. Rain happens when moisture in
the air reaches levels at which it can no longer be held, and cloud seeding aims to facilitate and accelerate
that process by making available chemical ‘nuclei’ around which condensation can take place.
 These ‘seeds’ of rain can be the iodides of silver or potassium, dry ice (solid carbon dioxide), or liquid
propane.
 The seeds can be delivered by plane or simply by spraying from the ground.
Where all has it been tried earlier?
 Cloud seeding is not new to India and it has earlier been attempted in Karnataka, Andhra Pradesh and
Maharashtra to address drought.
La Excellence IAS 54 Website: www.laex.in/testprep
9052 29 29 29 / 9052 49 29 29 (Hyderabad)
9121 41 29 29 / 9121 44 29 29 (Bangalore)
https://t.me/pdf4exams https://t.me/allupscmaterials

 Similar experiments of cloud seeding had earlier been tried in Australia, America, Spain and France.
 In United Arab Emirates, the cloud seeding technique led to creation of 52 storms in Abu Dhabi.
 Till 2018, IMD had around 30 successful incidents of seeding.
How successful is the cloud seeding technology?
 The Pune-based Indian Institute of Tropical Meteorology has been carrying out cloud seeding
experiments for several years now. These experiments have been done in areas around Nagpur,
Solapur, Hyderabad, Ahmedabad, Jodhpur, and recently Varanasi.
 The success rate of these experiments in inducing rains is about 60 to 70 percent, depending on local
atmospheric conditions, the amount of moisture in the air and cloud characteristics.

Do You Know ?
The inventor of Cloud Seeding, Mr. Vincent Schaefer carried out the first successful experiment of his idea
in 1946, in lab conditions, managed to create a snowstorm.
The most impressive claim of recent years though has to come from China, who say that in 2008 during
the Beijing Olympics they used this process in reverse to actually stop rain forming and keep it dry over
the event!

Mains question
What is cloud seeding? Do you think cloud seeding will alleviate the conditions of farmers and other rural
citizens of our country? (150W|10M)

La Excellence IAS 55 Website: www.laex.in/testprep


9052 29 29 29 / 9052 49 29 29 (Hyderabad)
9121 41 29 29 / 9121 44 29 29 (Bangalore)
https://t.me/pdf4exams https://t.me/allupscmaterials

4.2 Water policy


Context: The Union Water Resources Ministry has finalized a committee to draft a new National Water Policy
(NWP).
Highlights:
The Centre plan to update the NWP and make key changes in water governance structure and regulatory
framework
The committee is expected to produce a report within six months.
Why new policy?
 NWP was formulated to govern the planning and development of water resources and their optimum
utilisation. The first NWP was adopted in September, 1987. It was reviewed and updated in 2002 and later
in 2012.
 NITI Aayog has said that India is facing its first water crisis and the demand for potable water may outstrip
supply by the year 2030 if precautionary steps are not taken.
Significance of new water policy:
 After 7 seven years of previous updation, there are a lot of changes which need to be addressed and
the prioritization of the water usage needs to be defined.
 Spring sets in Himalayas have been decreasing without any active step by the government.
 Revitalisation of rivers needs to be brought in focus because many of our rivers and rivulets are drying and
the policy parameters need to be set up accordingly.
 Technological innovations like censors, geographic information systems (GIS) and satellite imagery need to
be introduced to modulate the water and track the flow.
 Budgeting needs to be done in a way that it covers all levels from the basin to sub basin.
 NITI Aayog has sensitised in its Composite Water Management Index 2.0, 2018 that water usage in current
times are very high and inefficient.
 The water used in irrigation sector gives the efficiency of 30-38%, the water for drinking water supply and
sanitation in the urban area bears the losses around 40-45%. Villages on the other hand get very less
amount of water so the supply needs to be balanced.
 To overcome the natural and human caused challenges like-Adverse effects of climate change ,Extreme
rainfall, Water scarcity during summers ,Drying up of rivers , Degrading water quality and river pollution.
 In irrigation sector, command area development has not been done. Around 21 million hectares of land
has been created for irrigation but it is not reaching the farmers because the small channels have not been
constructed.

Mains question
A comprehensive and dynamic water policy is the need of the hour to address the water concerns in our
country. Discuss the statement in light of the draft new water policy released recently. (150W|10M)

4.3 Swachh – Nirmal Tat Abhiyan


Context: Ministry of Environment, Forests and Climate Change has started an awareness-cum-cleanliness
campaign of 50 identified beaches under Swachh-Niramal Tat Abhyaan.
Highlights:
Aim: To make our beaches clean and create awareness amongst citizens about importance of coastal
ecosystems
 Identified beaches are in 10 coastal states/Union Territories (UTs) namely Gujarat, Daman & Diu,
Maharshtra, Goa, Karnataka, Kerala, Tamil Nadu, Puducherry, Andhra Pradesh and Odisha.

La Excellence IAS 56 Website: www.laex.in/testprep


9052 29 29 29 / 9052 49 29 29 (Hyderabad)
9121 41 29 29 / 9121 44 29 29 (Bangalore)
https://t.me/pdf4exams https://t.me/allupscmaterials

 Cleaning drives in al beaches involves school/college students of Eco-clubs, district administration,


institutions, volunteers, local communities, and other stakeholders.
 Cleaning activities will be duration of 2 hours on a daily basis of 1km stretch of the beach identified.
 Collected waste will be processed as under Waste Management Rules, 2016.
 Environment Education Division of the Ministry and SICOM(Society of Integrated Coastal Management)
under Ministry will responsible for overall coordination of the drive.
 After completion of the drive, the best three beaches will be suitably awarded by Ministry and a
Certificate of Appreciation for all the participating Eco-clubs.
Plastic Waste Management Rules, 2016:
 Increase the minimum thickness of plastic carry bags and sheets from 40 to 50 microns.
 Expand the jurisdiction of applicability from municipal area to rural areas.
 Extended producer responsibility: to add responsibility on producers and generators of plastic,
individual and bulk generators(offices, commercial establishments, industries) are to segregate plastic
waste at source, handover segregate waste and pay user fee as per local body byelaws
 Persons organizing public events made responsible for management of plastic waste generated from
those events
 Introduce collect back system of plastic waste by producers/brand owners.
 Collection of plastic waste management fee in the form on pre-registration of producers,
 State Pollution Control Boards (SPCBs) will not grant/ renew registration of plastic bags, or multi-
layered packaging unless the producer proposes the action plan endorsed by the concerned authority.
 Retailers and street vendors to not sell or provide commodities to consumers in carry bags or plastic
sheet or multi-layered packaging.
 Manufacturing and use of non-recyclable multi-layered plastic to be phased in 2 years.
Plastic Waste Management Rules(amended), 2018:
 Phasing out of Multi-layered Plastics (MLP) no applicable to MLP which are non-recyclable or non-
energy recoverable or with no alternate use.
 Automated, Central registration system for registration of producer or importer or brand owner, to be
evolved by Central Pollution Control Board (CPCB).
SICOM(Society of Integrated Coastal Management):
 SICOM has been established under aegis of MOEFCC,
 Vision: for vibrant, healthy and resilient coastal and marine environment for continuous and enhanced
outflow of benefits to the country and coastal community.
 Objectives:
 Support implementation of ICZM (Integrated Coastal Zone Management) activities in India.
 To promote R&D (research and development) and stakeholder participation in management of
Coastal areas in India.
 To support to check violations to CRZ (Coastal Regulation Zone) through improved technology-
enabled enforcement, strengthened institutions and regulatory and legal reform.

4.4 Red Atlas Action Plan Map


Context: Recently, Vice-President unveiled the 'Red Atlas Action Plan Map,' and the 'Coastal Flood Warning
System App (CFLOWS-Chennai) to aid Tamil Nadu government in effective flood mitigation in Chennai .
Highlights:
Red Atlas Action plan map
 It is a first of its kind ready reckoner, prepared by the Ministry of Earth Sciences.
 The atlas, with probable scenarios for different rainfall periods, aims at flood mitigation,
preparedness, operations and management aspects.
 The manual provides information, on corporation wards that are likely to be affected due to flooding,
and the areas that may need evacuation in Chennai taking into account all historical datasets.
La Excellence IAS 57 Website: www.laex.in/testprep
9052 29 29 29 / 9052 49 29 29 (Hyderabad)
9121 41 29 29 / 9121 44 29 29 (Bangalore)
https://t.me/pdf4exams https://t.me/allupscmaterials

Coastal Flood Warning System app for Chennai (CFLOWS)


 Launched by NIOT, CFLOWS is India’s first integrated coastal flood warning system.
 It is an integrated GIS-based decision support system to provide forecast on potential inundation 10 days
in advance and involves coupling models of regional weather forecasts, storm surges and captures about
796 flood scenarios
 CFLOWS can simulate the scenario and predict what will happen in a particular area.
 It will be hosted and made operational at National Centre for Coastal Research (NCCR) with
meteorological data inputs from IMD, National Centre for Medium Range Weather Forecasting (NCMRWF)
and Indian National Centre for Ocean Information Services (INCOIS).
4.5 Geochemical Baseline Atlas of India.
Context:
Geochemical Baseline Atlas of India’ developed by CSIR-National Geophysical Research Institute (NGRI) for use
by policy makers to assess environmental damage.
Highlights:
 This is the third map among the series of maps published by NDRI. Earlier, The Gravity map of India and
Seismic map of India were released by the research institute
 The atlas consisting 45 maps of metals, oxides and elements present in top and bottom soils across India
will serve as a reference against which future generations of the country would be able to assess the
chemical compositional changes on Earth’s surface.
 The geochemical data presented in these maps will be a part of the Global Map to be prepared by the
International Union of Geological Sciences (IUGC).

La Excellence IAS 58 Website: www.laex.in/testprep


9052 29 29 29 / 9052 49 29 29 (Hyderabad)
9121 41 29 29 / 9121 44 29 29 (Bangalore)
https://t.me/pdf4exams https://t.me/allupscmaterials

Importance of geochemical baseline maps:


 Soil is a vulnerable geological medium which sustains the human activities. Hence, according to
experts, it is important to determine the present abundance and spacial distribution of the chemical
elements across the earth’s surface.
 “The baseline map of India will help to plan the land use in different parts of the country. For example,
a toothpaste manufacturing industry, which use high concentration of Strontium, cannot be in a place
where soil is already having high baseline concentration of Strontium,” said scientists from CSIR-
NGRA.
 The map will also support in choosing the right soil that’s rich in certain micronutrient elements
required for each plant growth.
 These maps will form the backbone for environment management. It will help in finding out future
contaminations due to industries across the country. Government and policymakers can leverage it in
planning the land use accordingly.
Council of Scientific and Industrial Research:
 Abbreviated as CSIR was established by the Government of India in September of 1942.
 It is an autonomous body that has emerged as the largest research and development organisation
in India.
 Although it is mainly funded by the Ministry of Science and Technology, it operates as an
autonomous body through the Societies Registration Act, 1860.

National Geophysical Research Institute (NGRI):


 It is a geoscientific research organization established in 1961 under the Council of Scientific and
Industrial Research (CSIR).
 It is India's largest Research and Development organization.

4.6 Indian Forest Act of 2019


The Union Environment Ministry withdrew a draft amendment that proposed updates to the Indian Forest
Act, 1927.
Description:
 The Indian Forest Act, 2019, was envisaged
as an amendment to the Indian Forest Act,
1927, and an attempt to address
contemporary challenges to the country’s
forests.
 The draft law had been sent to key forest
officers in the States for soliciting comments
and objections.
 The legislation also proposed a forest
development cess of up to 10% of the
assessed value of mining products removed
from forests, and water used for irrigation or
in industries.
 This amount would have been deposited in a
special fund and used “exclusively for
reforestation; forest protection and other
ancillary purposes connected with tree
planting, forest development and
conservation,” the draft document noted.
La Excellence IAS 59 Website: www.laex.in/testprep
9052 29 29 29 / 9052 49 29 29 (Hyderabad)
9121 41 29 29 / 9121 44 29 29 (Bangalore)
https://t.me/pdf4exams https://t.me/allupscmaterials

Indian Forest Act, 1927:


 The Indian Forest Act, 1927 was largely based on previous Indian Forest Acts implemented under the
British. The most famous one was the Indian Forest Act of 1878.
 Both the 1878 act and the 1927 one sought to consolidate and reserve the areas having forest cover, or
significant wildlife, to regulate movement and transit of forest produce, and duty leviable on timber and
other forest produce.
 It also defines the procedure to be followed for declaring an area to be a Reserved Forest, a Protected
Forest or a Village Forest.
 It defines what a forest offence is, what are the activities prohibited inside a Reserved Forest, and
penalties leviable on violation of the provisions of the Act.

Mains question
The amendments proposed to Indian Forest Act, 1927 was archaic and would have harmed the prospects of
the communities depending on forest resources. Do you agree? Justify your opinion. (150W|10M)

5. In news
5.1 ETHANOL PRODUCTION IN INDIA
The Ministry of Environment and Forests has announced that mills would not require separate environmental
clearance to produce additional ethanol from B-heavy molasses.
Benefits of the latest move
 Mills currently have all-time-high stocks of sugar, and they have been at loggerheads with farmers over
non-payment of dues. Mill owners insist that the reason behind their woes is excess production of sugar
and fall in its price.
 Under the circumstances, ethanol is the only real saviour — both for mills and cane growers.
ETHANOL:
 Ethanol, or ethyl alcohol, is a liquid biofuel made from molasses, a byproduct of sugar manufacturing.
 it has several uses. At 95% purity, it is called rectified spirit and is used as the intoxicating ingredient in
alcoholic beverages. At 99%-plus purity, ethanol is used for blending with petrol.

La Excellence IAS 60 Website: www.laex.in/testprep


9052 29 29 29 / 9052 49 29 29 (Hyderabad)
9121 41 29 29 / 9121 44 29 29 (Bangalore)
https://t.me/pdf4exams https://t.me/allupscmaterials

Need of Bio Fuels


 India is dependent on imports for about 82.1% of its crude oil requirement and to the extent of about
44.4% in case of natural gas.
 India is expected to need 10 billion litres of ethanol annually to meet the 20% blending target in 2030 if
petrol consumption continues to grow at the current pace.
Concerns and challenges:
 Consistent shortfall in supply of ethanol in the past, mainly on account of the cyclical nature of the
sugarcane harvests in the country.
 Lack of an integrated approach in the EBP across its value chain.
Way forward:
The National Policy on Bio-fuels has set a target of 20% blending of biofuels, both for bio-diesel and bio-
ethanol. This will require an integrated approach in the Ethanol Blending Programme (EBP). The time is ripe
for a cogent and consistent policy and administrative framework in the program implementation for the
success of EBP.

Mains questions
Ethanol is a premier, high performance fuel. It has tremendous environmental benefits and is a key
component to energy independence for our country. Critically comment (150W|10M)

5.2 The Lancet Countdown on Health and Climate Change


Context:
According to a major new report published in The Lancet, Climate change is already damaging the health of
the world’s children and is set to shape the well-being of an entire generation.
Highlights of the report
 The report notes that as temperatures rise, infants will bear the greatest burden of malnutrition and rising
food prices — average yield potential of maize and rice has declined almost 2% in India since the 1960s,
with malnutrition already responsible for two-thirds of under-5 deaths.
 Also, children will suffer most from the rise in infectious diseases — with climatic suitability for the Vibrio
bacteria that cause cholera rising 3% a year in India since the early 1980s, the study warns.
 Diarrhoeal infections, a major cause of child mortality, will spread into new areas, whilst deadly
heatwaves, similar to the one in 2015 that killed thousands of people in India, could soon become the
norm.
What are the Concerns?
 This report shows that the public health gains achieved over the past 50 years could soon be reversed by
the changing climate.
 If the world follows a business-as-usual pathway, with high carbon emissions and climate change
continuing at the current rate, a child born today will face a world on average over 4˚C warmer by their
71st birthday, threatening their health at every stage of their lives.
 Nothing short of a 7.4% year-on-year cut in fossil CO2 emissions from 2019 to 2050 will limit global
warming to the more ambitious goal of 1.5°C.

Mains question
Climate change will increasingly make it difficult for us to realize our true demographic potential. Analyze
the statement with respect to the impact of climate change on children of our world. (250W|15M)

La Excellence IAS 61 Website: www.laex.in/testprep


9052 29 29 29 / 9052 49 29 29 (Hyderabad)
9121 41 29 29 / 9121 44 29 29 (Bangalore)
https://t.me/pdf4exams https://t.me/allupscmaterials

E. INTERNATIONAL RELATIONS
1. Bilateral Relations
1.1 India-Uzbekistan
MoUs seek to enhance cooperation in military medicine, education and training
 India has offered a concessional line of credit of $40 million for procurement of goods and services by
Uzbekistan from India.
 The direct exchanges related to training, capacity building and education between the armed forces on
both sides
 The bilateral exercise will be conducted from November 4 to 13 at Chirchiq Training Area near Tashkent
and will be focused on counter-terrorism- “Dustlik 2019”.
 The first defence-industry Workshop organized in Tashkent in September 2019.

History of India-Uzbekistan Relations


 There are frequent references to Kamboja in Sanskrit and Pali literature, which is stated to include parts
of present-day Uzbekistan.
 Ancient trade route Uttarpath passed through Uzbekistan.
 The eminent Uzbek scholar Al-Beruni visited India in the 11th century.
 Babur, the founder of the Mughal Empire in India, is believed to have come to India from the Fergana
valley in Uzbekistan.
 Mirza Ghalib and Amir Khusro are notable Indians of Uzbek parentage. Indian movies have traditionally
been popular in Uzbekistan.
 India's first Prime Minister, Jawaharlal Nehru had visited Uzbekistan in 1955 and 1961 when it was part
of the erstwhile Soviet Union.
 India had close interaction with the Uzbek Soviet Republic during the Soviet times. Indian leaders often
visited Tashkent and other places. Prime Minister, Shri Lal Bahadur Shastri passed away in Tashkent on
11 January 1966 after signing the Tashkent declaration with Pakistan.
 Uzbekistan's independence in August 1991 has led to the expansion and strengthening of bilateral ties
between New Delhi and Tashkent in the political, economic and cultural spheres.
 India was the first country which was visited officially by President Karimov in August 1991. This was a
historic visit outlining the framework for mutually beneficial cooperation between the two countries.
India-Uzbekistan Relations
Political Relations:
 Following Uzbekistan’s independence, it was upgraded to the level of Embassy through the signing of a
Protocol on Diplomatic and Consular matters on 18th March 1992.
 Uzbekistan and India have signed Agreements/MOUs/ Protocols/Joint Statements in areas such as
trade, investment, education, civil aviation, tourism, science & technology, telecommunications,
agriculture and IT.
Economic Relations :
 Trade relations between India and Uzbekistan are governed by the Agreement on Trade and Economic
Cooperation signed in May 1993.
 This agreement provides for mutual Most Favored Nation (MFN) treatment, promotion of economic,
industrial, scientific and technical cooperation, including in the field of training of personnel, active
participation of small and medium sized enterprises in bilateral economic cooperation, etc.
 India and Uzbekistan signed an Agreement on Avoidance of Double Taxation in 1993, and for Bilateral
Investment Promotion and Protection in May 1999.
Bilateral Trade:

La Excellence IAS 62 Website: www.laex.in/testprep


9052 29 29 29 / 9052 49 29 29 (Hyderabad)
9121 41 29 29 / 9121 44 29 29 (Bangalore)
https://t.me/pdf4exams https://t.me/allupscmaterials

India’s export: Pharmaceutical products, mechanical equipments, vehicles, service, optical instruments and
equipment.
India’s import: Fruit and vegetable products, services, fertilizers, juice products, extracts and lubricants.
The two sides have agreed to scale up bilateral trade from $300 million to $1 billion by 2020.
Cultural Relations:
 Lal Bahadur Shastri Centre for Indian Culture working under Indian Council for Cultural Relations (ICCR),
was established in Tashkent in 1995
 Besides organizing seminar events relating to Indian culture, the Centre also organizes regular classes
for Kathak, Yoga, Tabla and Hindi language.
 Three Uzbek educational institutions, nationwide, promote studies of Hindi language, from primary to
post-graduate level.
 Uzbek Radio completed 50 years of Hindi broadcasting in 2012.
 A Protocol on cooperation in the field of mass media was signed in October 1992 and again in May
2000.
 The cities of Samarkand and Agra are named as sister cities.
Importance of India-Uzbekistan Relationship
Energy Needs
1. India’s need for energy can be fulfilled by gaining access to the natural resource of Uzbekistan, at the
same time India can also prove to be a viable market for Uzbek oil and gas resources.
2. India has signed a pact on the import of over 2,000 tonnes of uranium with Uzbekistan.
Military and Security Interest
1. India views itself as a stabilizer and security provider in Central Asia.
2. India has long wanted to play a larger role in the Shanghai Cooperation Organisation (SCO), Uzbekistan
being the largest military power among central Asian Countries, cooperating with it can help us achieve
our objective.
3. Military Cooperation with Uzbekistan is of utmost importance for our strategic interest and protection
of our assets in Afghanistan.
Afghanistan angle
1. Both Uzbekistan and India have a shared goal of peace and prosperity in Afghanistan as any turmoil in
Afghanistan can adversely impact both the countries.
2. India is also exploring with Uzbekistan the possibility of extending the Friendship Railway Bridge to
Herat in western Afghanistan.
3. The rail route to Herat, if extended to Kabul, would also link to India’s “air corridor”, allowing trade,
especially dry fruits, and agricultural produce to travel along the routes from India to Central Asia and
back in much shorter time.
Economic and Trade Relations
At the same time, growing economies of Central Asia are a big market for Indian exports like
pharmaceuticals with a lot of potential for improvement.

Mains question
Despite having deep civilizational relations since antiquity with the Central Asian Republics (CARs), India
is yet to have a well defined and well-articulated foreign policy towards the region. Comment
(250W|15M)

La Excellence IAS 63 Website: www.laex.in/testprep


9052 29 29 29 / 9052 49 29 29 (Hyderabad)
9121 41 29 29 / 9121 44 29 29 (Bangalore)
https://t.me/pdf4exams https://t.me/allupscmaterials

1.2 India, Russia to conclude mutual logistics agreement:


Context: India and Russia are expected to conclude a mutual logistics agreement and review the setting up of
joint ventures for manufacturing spares for Russian defence platforms in India during the visit of Defence
minister to Moscow.
What are Logistics agreements?
Logistics agreements are administrative arrangements facilitating access to military facilities for exchange of
fuel and provisions on mutual agreement when the Indian military is operating abroad.
Highlights:
Defence minister will co-chair the 19th India-Russia Inter-Governmental Commission on Military and Military
Technical Cooperation with his Russian counterpart.
 Agreement on Reciprocal Logistics Support (ARLS) is one of the main agendas during this meet. (The
signing of this agreement will facilitate access to military facilities for exchange of fuel and provisions on
mutual agreement when the Indian military is operating abroad.)
 The purchase of the S-400 air-defence missile systems, will be the another major issue on the agenda ,
over which the U.S. is continuing a tough stance with respect to waiver from sanctions under the
Countering America’s Adversaries Through Sanctions Act.
 The deliveries are expected to begin by 2020-end, 24 months after the signing of the contract, and that is
when the sanctions are expected to kick in.
Why are these agreements important for India?
These agreements will improve the relationship between India and Russia which has relatively suffered in
recent times due to India’s perceived shift towards American influence. These agreements are expected to
bridge the trust deficit which has developed in recent times. Apart from these, the agreements signed will
improve the strategic capability of India under emerging new threats.

Mains question
India-Russia relations despite many challenges is progressing and evolving according to the changing
situation. Discuss (250W|15M)

1.3 H-1B visa


Context: U.S. court refuses to ban work permits for H-1B holders’ spouses and has allowed spouses of H-1B
visa holders to work in America.
Background:
 A 2015 rule issued by then U.S. president ,allowed work permits for certain categories of H-4 visa holders,
primarily spouses of those having H-1B work visas waiting for their Green Card, to work in the U.S.
 Indians, in particular women, were the greatest beneficiary of this rule, which has been challenged by
several U.S. workers.
 However, the US citizenship and Immigration Services (USCIS) has denied several applications by the new
employer by citing that the new position does not constitute a 'specialty occupation’.
H-1B visa:
The US H-1B visa is a non-immigrant visa that allows US companies to employ graduate level workers in
specialty occupations.
Specialty occupations requires:
 Theoretical or technical expertise in specialized fields such as in IT, finance, accounting, architecture,
engineering, mathematics, science, medicine, etc.
 Any professional level job that usually requires you to have a bachelor’s degree or higher can come
under the H-1B visa for specialty occupations.

La Excellence IAS 64 Website: www.laex.in/testprep


9052 29 29 29 / 9052 49 29 29 (Hyderabad)
9121 41 29 29 / 9121 44 29 29 (Bangalore)
https://t.me/pdf4exams https://t.me/allupscmaterials

Changes introduced to H1B visa:


 US says H1B visas will be issued to only the most-skilled foreigners or highest-paid beneficiaries
 In July 2018, USCIS adjudicators granted right to reject H1B applications that do not provide the necessary
required information when submitted.
 In October 2018, The US initiates deportation of H1B holders with expired visas.
 In October 2018, The US proposes revision of “specialty occupations” definition for the H1B visa.
 In Jan 2019, The USCIS announces it will require petitioners seeking to file H1B cap-subject petitions to
first electronically register with USCIS.
What impact will this have on India?
Indians, in particular women, are the greatest beneficiary of this rule because majority of the H4 visa holders
are Indian Americans and Women.
1.4 Tiger Triumph
Context: The maiden India-U.S. joint tri-services Humanitarian Assistance and Disaster Relief (HADR) Exercise
named “Tiger Triumph” is scheduled on the Eastern seaboard.
Highlights:
 The exercise is aimed at developing interoperability for conducting HADR operations.
 Relief forces would be undertaken to the exercise scenario at the HADR area, Kakinada.
India – US strategic relationship:
 New- frame work for India-US Defense relations has led to increase of defence trade, joint military
exercises, co operation in the maritime security aspects etc.
 India had participated in Rim of Pacific (RIMPAC) exercise between July and August 2016.
Examples of enhanced relationship between the two countries in recent times:
 Logistics Exchange Memorandum of Association (LEMOA) agreement was also signed in recent times.
 India was recognized by the US as a “major defence partner in 2016.
 The year 2018 saw the inauguration of India US 2+2 Ministerial dialogue and signing of Communications
compatibility and security Agreement (COMCASA) to help share intelligence between the two countries.
 This year also saw the altering of nomenclature of the US-Pacific Command to the US-Pacific Command
and agreement of first tri-service Exercise.
1.5 U.S. trade negotiators to visit Delhi for more talks
Description:
Trade negotiators from the Office of the U.S. Trade Representative (USTR) are expected to visit New Delhi, to
continue discussions with India's Commerce Minister.
 Price caps or ‘trade margin rationalisation’ for knee implants and cardiac stents have been one of the key
differences between the two sides. Also under discussion for months has been the Generalised System of
Preferences.
 A limited trade package between the two countries may be finalized soon.
Background:
 India’s GSP benefits were revoked in June on the grounds that India had not assured the U.S. that it would
provide “equitable and reasonable” access to its markets, as per U.S. President Donald Trump’s
proclamation on the issue.
 He believes that the world trading system, and more broadly, the multilateral system, treats America
unfairly.
 India has wanted its GSP benefits restored but it is unclear whether any limited trade agreement reached
between the sides will result in a full or partial restoration of GSP benefits to India.

La Excellence IAS 65 Website: www.laex.in/testprep


9052 29 29 29 / 9052 49 29 29 (Hyderabad)
9121 41 29 29 / 9121 44 29 29 (Bangalore)
https://t.me/pdf4exams https://t.me/allupscmaterials

Generalised System of Preferences:


 U.S. trade preference programs such as the Generalized System of Preferences (GSP) provide
opportunities for many of the world’s poorest countries to use trade to grow their economies and climb
out of poverty.
 The GSP program provides additional benefits for products from least developed countries. The list of
products eligible for duty-free treatment when imported from GSP beneficiaries can be found here.

Impact:
 A limited trade package might be finalized.
 Though the limited trade package won’t be very big value-wise, but after roughly 18 months of negotiating
and almost six months since GSP was suspended, it would be a symbolic win for both sides.
 This might clear the initial hurdles in the path of a future full-fledged trade deal. Trade ties between the
two countries will only help strengthen the strategic relationship between the two.

Mains question
While the strategic partnership between the US and India remains robust, some analysts see the
relationship as becoming significantly strained in other areas in recent times. Analyse. (150W|10M)

2. International Institutions
2.1 ICJ
Highlights
 The ICJ had ruled that Pakistan must review the death sentence awarded to Jadhav.
 The bench ordered an "effective review and reconsideration of the conviction and sentence of Kulbhushan
Sudhir Jadhav.
 One of the issues that the UN Court had to examine was the question of whether the rights relating to
consular access, set out in Article 36 of the Vienna Convention, were in any manner to be excluded in a
situation where the individual concerned was accused of "carrying out acts of espionage".
 Court noted in that regard that there is no provision in the Vienna Convention containing a reference to
cases of alleged "espionage"; nor does the Article concerning consular access, Article 36, exclude from its
scope certain categories of persons, such as those suspected of espionage.
 The Court concluded that Article 36 of the Vienna Convention was applicable in full to the case at hand
 The World Court noted that Pakistan's making of the notification some three weeks after Jadhav's "arrest"
constituted a breach of its obligation to inform India's consular post without delay, as required by the
provisions of the Vienna Convention
About ICJ
 ICJ was established in 1945 by the United Nations charter and started working in April 1946.
 It is the principal judicial organ of the United Nations, situated at the Peace Palace in The Hague
(Netherlands).
 Unlike the six principal organs of the United Nations, it is the only one not located in New York (USA).
 It settles legal disputes between States and gives advisory opinions in accordance with international law,
on legal questions referred to it by authorized United Nations organs and specialized agencies.
 It has 193 state parties.
Vienna Convention on consular relations
 The Vienna Convention on Consular Relations of 1963 is an international treaty that defines a framework
for consular relations between independent states.

La Excellence IAS 66 Website: www.laex.in/testprep


9052 29 29 29 / 9052 49 29 29 (Hyderabad)
9121 41 29 29 / 9121 44 29 29 (Bangalore)
https://t.me/pdf4exams https://t.me/allupscmaterials

 A consul performs two functions: protecting in the host country the interests of their countrymen, and
furthering the relations between the two states.
 Consular and Diplomat work out of the same premises, and under this treaty they are afforded most of
the same privileges.
 Immunity given to consular is similar to diplomat.
 The treaty has been ratified by 179 states.
Vienna Convention on Diplomatic Relations
 It is a treaty that came into force in 1964
 It lays out the rules and regulations for diplomatic relations between countries as well as the various
privileges that diplomats and diplomatic missions enjoy.
 One of these privileges is legal immunity for diplomats so that they don’t have to face prosecution as per
their host country’s laws.

Mains question
What do you understand by term ‘Universal treaties’? Discuss the main features of Vienna Convention.
(150W|10M)

2.2 RCEP

Context:
Seven years after India joined negotiations for the 16-nation ASEAN (Association for South East Asian Nations)-
led RCEP or Free Trade Agreement India was dropping out of the agreement, citing its negative effects on
“farmers, MSMEs and the dairy sector”.
Highlights:
Why India pulled out RCEP at the last minute:
India stated in the RCEP summit in Bangkok that the present form of the RCEP agreement does not fully reflect
the basic spirit and it does not address satisfactorily India’s outstanding issues and concerns. Hence, it is not
possible to join the RCEP agreement.
Who signed the pact?
The RCEP brings together the 10 members association of southeast Asian nations (including India), China,
Japan, South Korea, Australia, and New Zealand.
The Regional Comprehensive Economic Partnership (RCEP) is a so-called mega-regional economic agreement
being negotiated since 2012 between the 10 ASEAN (Association of South-East Asian Nations) governments
and their six FTA partners: Australia, China, India, Japan, New Zealand and South Korea.
Goal: To boost economic growth and equitable economic development, advance economic cooperation and
broaden and deepen integration in the region through the RCEP.

La Excellence IAS 67 Website: www.laex.in/testprep


9052 29 29 29 / 9052 49 29 29 (Hyderabad)
9121 41 29 29 / 9121 44 29 29 (Bangalore)
https://t.me/pdf4exams https://t.me/allupscmaterials

Reasons for Not Joining


 Domestic industry and dairy farmers had strong reservations about the trade pact.
 India’s trade deficit with the RCEP nations is 105 billion dollars
 China alone accounts for 54 billion dollars
 The worry is over Chinese manufactured goods and dairy products from New Zealand flooding Indian
markets, hurting domestic interests.
 India had also sought 2014 base year for tariff reductions instead of 2013.
 “Inadequate protection” against surges in imports.
 India already has bilateral FTA with most RCEP nations, but it has recorded trade deficits with these
countries.
 The trade agreement was also seen as being detrimental to the government’s Make In India initiative
 India has not received any assurances on its demand for more market access.
India’s Trade Deficit India’s Trade Surplus
China-53.57 billion dollars Philippines-1.16 billion dollars
South Korea-12 billion dollars Laos
Indonesia-10.57 billion dollars Myanmar
Japan-7.91 billion dollars Cambodia
Australia-9.61 billion dollars
What Could Have Been Its Impact ?
 Impact on economy which is already under tremendous pressure after demonetization and GST.
 The worst hit manufacturers through RCEP could be –Steel, Aluminium, Copper, Pharmaceuticals and
Textiles.
 It could threaten farm livelihoods autonomy over seeds and also endangers the country’s self sufficient
dairy sector.
What Should India Do?
 India must enhance exports.
 Build manufacturing capabilities.
 Make high value goods at competitive prices.
 Focus on innovation and research.
 Improve services trade.
The China Factor
 India’s biggest fear is RCEP will allow China to dump its cheap products in India adversely affecting
domestic products. That will make trade deficit worst.
 India’s trade deficit with China has risen 13 fold in the past decade
 India tends to export primary materials such as ores minerals and cotton.
 Chinese exports to India include capital and manufactured goods.
India’s Previous trade Deals
 Utilisation rate of india’s FTA stands at 25%.
 India also runs a trade deficit with African countries.
 India has trade deficit with South American trade bloc MERCOSUR.
 Import duties forgne during the last three years were 10.3 billion dollars.
 With ASEAN countries the merchandise trade deficit has widened to 14.6 billion dollars.
CONCERNS with the decision
India's entry into RCEP will strengthen its strategic weight but it may act as a double-edged sword for India:
La Excellence IAS 68 Website: www.laex.in/testprep
9052 29 29 29 / 9052 49 29 29 (Hyderabad)
9121 41 29 29 / 9121 44 29 29 (Bangalore)
https://t.me/pdf4exams https://t.me/allupscmaterials

 Short term: The loss to the economy far exceeds the short term perceived benefits of staying out of the
pact.
 Protectionism: This action signals a shift towards protectionist stance.
 Strategic Loss: With this India has ceded space to China to have greater say in the region.
 Missed Supply Chains: Manufacturing today requires greater integration with global supply chains.
 Policy Dilemma: On the one hand, India wants to become a manufacturing hub.staying out of RCEP
reduces opportunities for trading with these countries, which account for roughly a third of globe trade.
 No Focus on Reforms: India should have used this opportunity to push through contentious but necessary
reforms that would boost competitiveness.
 Chinese Slowdown Advantage: Signing the agreement would have signalled an embrace of free trade. It
could have aided in the shift of companies out of China to India.
 Sector wise approach lacking: Indian side should have put greater effort into convincing other countries
for allowing a gradual phasing out of tariffs to ease domestic fears.
 India can be diplomatically pressurised from trade partners like ASEAN, Japan, South Korea, with whom
India already has FTAs.
 In the event of declining economic growth, India should not stay out of the largest free-trade bloc in the
world.
 Apart from this, RCEP provides a chance for India to bring in historic trade reforms, which in itself will
cement India's position as a major global economy and make Indian industry competitive.
Why Farmers Were Opposed To This?
Trade Tariffs: RCEP will permanently bring down import duties on most agricultural commodities to zero
which will lead to countries looking to dump their agricultural produce in India which would lead to a drastic
drop in prices.
Dairy Sector and Plantations and Plantations Sector: Going to hit very hard.
 New Zealand and Australia being part of RCEP will invariably lead to the dumping of their dairy products
into India.
 SE Asian Countries have larger and cheaper production of plantation crops like rubber, coconut, palm oil
as compared to India and opening up of markets will lead to a large inflow of these products given their
price competitiveness.
SEED Freedom:
 The IPR clauses are likely to seriously impinge on farmer’s seed freedom.
 Seed companies will get more powers to protect their intellectual property rights and farmers would be
criminalised when they save and exchange seeds.
Way Forward
India commands around 1.7 per cent share of the world’s total goods exports ranking 20th as per the WTO
2018 data. For achieving a 5 per cent share in world exports (the government targets $1 trillion exports out of
total global exports of $20 trillion), India must build its manufacturing capabilities, and the recent steps by the
government are in that direction. How India manoeuvres the geo-political space will determine how successful
it is in becoming an export behemoth (in its quest towards a $5 trillion economy)

Mains question
By walking out of Regional Comprehensive Economic Partnership (RCEP) trade negotiations; India has
effectively ceded the economic sphere in the Indo-pacific region to China. Critically analyze. (150W|10M)

La Excellence IAS 69 Website: www.laex.in/testprep


9052 29 29 29 / 9052 49 29 29 (Hyderabad)
9121 41 29 29 / 9121 44 29 29 (Bangalore)
https://t.me/pdf4exams https://t.me/allupscmaterials

2.3 BRICS
What is BRICS?
 BRICS is the acronym for an association of five major emerging national economies: Brazil, Russia, India,
China and South Africa.
 BRIC started as a formal grouping in 2006 on the margins of G8 outreach summit.
 Originally the first four were grouped as "BRIC" before the induction of South Africa in 2010.
 1st Summit was held in Russia in 2009. Moreover 1st, 5 member BRICS summit was held in 2011.
 Since 2009, the BRICS nations have met annually at formal summits. 8th summit in 2016 was held in Goa
and BIMSTEC countries were invited for a joint summit as per the the tradition. 9th summit will be held in
China
BRICS Summit: origin
On November 30, 2001, Jim O’Neill, a British economist who was then chairman of Goldman Sachs Asset
Management, coined the term ‘BRIC’ to describe the four emerging economies of Brazil, Russia, India, and
China.
Background:
 In 2001 and 2002, real GDP growth in large emerging market economies will exceed that of the G7. At
end-2000, GDP in US$ on a PPP basis in Brazil, Russia, India and China (BRIC) was about 23.3% of world
GDP.
 On a current GDP basis, BRIC share of world GDP is 8%. Over the next 10 years, the weight of the BRICs
and especially China in world GDP will grow, raising important issues about the global economic impact of
fiscal and monetary policy in the BRICs.
 Eighteen years later, India finds itself as one of the emerging economies in the grouping and beyond,
especially G20. BRICS now brings together five economies accounting for 42% of the world’s population,
23% of the global GDP and an around 17% share of world trade.
As a formal grouping, BRIC started after the meeting of the leaders of Russia, India and China in St
Petersburg on the margins of the G8-Outreach Summit in July 2006.
 The grouping was formalised during the first meeting of BRIC Foreign Ministers on the margins of the
UNGA in New York in September 2006. The first BRIC Summit was held in Yekaterinburg, Russia, on June
16, 2009.
 It was agreed to expand BRIC to BRICS with the inclusion of South Africa at the BRICS Foreign Ministers’
meeting in New York in September 2010.
 South Africa attended the third BRICS Summit in Sanya on April 14, 2011.In 2018,Leaders of the grouping
commemorated the 10th anniversary of BRICS in Johannesburg.
What is the Significance of BRICS?
 Starting essentially with economic issues of mutual interest, the agenda of BRICS meetings has
considerably widened over the years to encompass topical global issues.
 BRICS cooperation has two pillars – consultation on issues of mutual interest through meetings of Leaders
as well as of Ministers of Finance, Trade, Health, S&T, Education, Agriculture, Communication, Labour, etc.
and practical cooperation in a number of areas through meetings of Working Groups/Senior Officials.
 Focused on “greater people-to-people participation” during the BRICS events like BRICS Film Festival,
BRICS Wellness Forum, BRICS Youth Forum and BRICS Friendship Cities Conclave held throughout the year
across the country.
 The New Development Bank(NDB) and the Contingent Reserve Arrangement (CRA) are the financial
mechanism under BRICS.

La Excellence IAS 70 Website: www.laex.in/testprep


9052 29 29 29 / 9052 49 29 29 (Hyderabad)
9121 41 29 29 / 9121 44 29 29 (Bangalore)
https://t.me/pdf4exams https://t.me/allupscmaterials

 CRA proposes to provide short-term liquidity support to the members through currency swaps to help
mitigating BOP crisis situation, if such a situation arises.
 It would also contribute to strengthening the global financial safety net and complement existing
international arrangements (from IMF) as an additional line of defence.
 BRICS Credit Rating Agency may come in near future to challenge the monopoly of the West.
 The BRICS free trade agreement may come up against fears of Indian and Russian markets being swamped
by Chinese imports.
 In the current global political and economic scenario where protectionism and patriotism is on the rise,
BRICS can become the bulwark of new globalization and may create new world order driven by emerging
economies.
BRICS Business Council:
The BRICS Business Council created a road map to achieve 500 billion USD trade target among BRICS nations
by next summit. An agreement was signed between New Development Bank and BRICS Business Council to
achieve this.
India & the current BRICS Summit
 India holds bilateral talks with leaders of Brazil, Russia and China
a) India – Brazil.
 The Brazilian President accepted India’s invitation to be the chief guest at India’s Republic Day 2020.
 The two leaders agreed to comprehensively enhance the strategic partnership between the two nations.
India Outlined areas for potential investment from Brazil including areas such as agricultural equipment,
biofuels, post-harvest technologies and animal husbandry.
 Discussed other areas of cooperation such as defence and space sector. Further, the Indian Prime Minister
welcomed the Brazilian President's decision to grant visa-free travel to Indian citizens.
b) India – china:
 India received to China for the third informal summit between the two nations in 2020.
 The two leaders further agreed on the importance of maintaining close dialogue on matters such as trade
and investment.
 The leaders further reviewed the ongoing preparations for the celebration of the 70th anniversary of the
establishment of diplomatic relations between India and China in 2020.
 The leaders also reiterated the importance of maintaining peace and security in border areas and
exchanged views on many issues including RCEP, WTO and BRICS.
c) India – Russia:
 During the talks, the two leaders reviewed the progress made in the bilateral relationship between the
two nations.
 The two leaders noted that the USD 25 billion bilateral trade target by 2025 has already been achieved.
 They further decided to hold the 1st bilateral regional forum between Russian Provinces and the Indian
States in 2020 to do away with the trade barriers at the regional level.
 They noted the progress made in oil and natural gas import. Russian President highlighted the potential of
the Arctic region in natural gas and invited India to invest in the region.
 The leaders also reviewed the progress made in the Infrastructure sector especially railways including
raising the speed of the Nagpur-Secunderabad sector railway line.
 The leaders also noted cooperation between India and Russia in Defence sector and Civil Nuclear Energy,
share a common position on several international issues and agreed to continue close consultations.
 Prime Minister of Indian accepted the Russia’s invitation to participate in Russia’s Victory Day celebrations
in 2020.

La Excellence IAS 71 Website: www.laex.in/testprep


9052 29 29 29 / 9052 49 29 29 (Hyderabad)
9121 41 29 29 / 9121 44 29 29 (Bangalore)
https://t.me/pdf4exams https://t.me/allupscmaterials

2.4 BIMSTEC Ports:


Context:
 The first-ever ‘BIMSTEC Ports’ Conclave held at Visakhapatnam.
 The BIMSTEC member Countries will be part of this conclave and will present their country’s perspective
on BIMSTEC and regional ports.
About the BIMSTEC:
 The Bay of Bengal Initiative for Multi-Sectoral Technical and Economic Cooperation (BIMSTEC) is an
international organisation of seven nations of South Asia and South-East Asia, housing 1.5 billion people
and having a combined gross domestic product of $3.5 trillion (2018).
 Bangladesh, India, Myanmar, Sri Lanka, Thailand, Bhutan and Nepal. Delegates from the People’s
Republic of Bangladesh, the Kingdom of Bhutan, the Republic of India, and the Republic of the Union of
Myanmar, the Federal Democratic Republic of Nepal, the Democratic Socialist Republic of Sri Lanka and
the Kingdom of Thailand.
 The objective of BIMSTEC alliance was to harness trade and accelerated growth with mutual cooperation
in different areas of common interest by utilizing the regional resources and geographical advantage.
Agenda of the conclave:
Port led Industrial and Tourism Development
 It is expected to explore the possibility of increasing economic cooperation by furthering EXIM trade and
coastal shipping.
 The diverse historical and cultural heritage of the BIMSTEC nations makes it an ideal destination for
tourism.
 Ports plays a vital role for encouraging cruise tourism.
Emergent Roles of Ports in the Global Supply Chain:
 Emergent role of ports and terminals in the backdrop of the expanding supply chains and the solutions
available.
 Adoption of new technological solutions to maximize efficiency and productivity by optimizing the
available space, resources, time and energy is the need of the hour.
Safe and Secure Ports:
 Emerging threats and safeguards against security threats, as ports are not only of vital commercial
interest, but also a nation’s strategic assets.
Ports Services: Delivering Value:
 Provide a platform for discussion on various steps undertaken for improving the ease of doing business.
Green Port Operations
 Commitment to the environment and Paris Accord to mitigate climate change.
 The ports must adopt a sustainable operations model.
2.5 International Criminal Court
Context: Judges at the international criminal court (ICC) have authorized a full-scale investigation into
allegations of mass persecution and crimes against Rohingyas.
Highlights:
 Myanmar is accused of mass persecution and ‘ongoing genocide’ of Muslim minority.
 ICC exercises jurisdiction over crimes where part of the alleged criminal conduct – in this case mass
deportation – takes place on the territory of a state party, the ICC has extended its international law-
enforcement role.
 A similar argument has been presented at the ICC on behalf of Syrian refugees who have been forced to
flee to neighboring Jordan, which, like Bangladesh, is a signatory to the Rome statute.
La Excellence IAS 72 Website: www.laex.in/testprep
9052 29 29 29 / 9052 49 29 29 (Hyderabad)
9121 41 29 29 / 9121 44 29 29 (Bangalore)
https://t.me/pdf4exams https://t.me/allupscmaterials

Key facts:
 ICC said that it has jurisdiction over crimes partially committed in Bangladesh, which is a member state
of the court.
 Myanmar is not a member of the global court. It has been accused of committing widespread abuses in a
campaign against the Rohingya. Myanmar's military began a counterinsurgency campaign against the
Rohingya in August 2017 in response to an insurgent attack.
International criminal court (ICC):
 International Criminal Court (ICC) Judges have approved a request from prosecutors to open an
investigation into crimes committed against Myanmar's Rohingya Muslim minority.
 Mandate: it is an intergovernmental international tribunal for Criminal prosecution of individuals for four
main crimes namely –Genocide,War crimes,Crimes against humanity and Crime of aggression.
 Established in: 2002 by Rome statue.
 HQ: Hague, Netherlands. The Court's official seat is in The Hague, Netherlands, but its proceedings may
take place anywhere.
 Jurisdiction: It can exercise its jurisdiction only when national courts are unwilling or unable to
investigate or prosecute such crimes. The Court can generally exercise jurisdiction only in three cases,
viz.
 If the accused is a national of a state party,
 If the alleged crime took place on the territory of a state party or
 If a situation is referred to the Court by the United Nations Security Council.

Who are the Rohingya?


 The Rohingya are Muslims who live in majority-Buddhist Myanmar. They are often described as "the
world's most persecuted minority".
 Nearly all of Myanmar's 1.1 million Rohingya lived in the western coastal state of Rakhine. The
government does not recognise them as citizens, effectively rendering them stateless.
 Extremist nationalist movements insist the group are illegal immigrants from Bangladesh, although the
Rohingya say they are native to Rakhine state.
India’s stand on the Rohingyas
 The Indian government feels that its internal security threatened due to the large scale migration.
 There is a widespread suspicion that some of the Rohingyas could have militant background. There are
some specific intelligence report that they are found to be very active in Jammu, Delhi, Hyderabad and
Mewat. Indian government views the report seriously as a potential threat to the internal and national
security of India.
 India has also not been a signatory of the 1951 UN Convention or the 1967 Protocol - both relating to the
Status of Refugees and included in the UNHCR statute. According to the UNHCR, a refugee is a person
living in another country following persecution in his own on the grounds of "race, religion, nationality,
membership of a particular social group or political opinion."

Mains question
Examine India's approach towards the Rohingya crisis and explores ways for India to strengthen
its role in finding solutions (250W|15M)

La Excellence IAS 73 Website: www.laex.in/testprep


9052 29 29 29 / 9052 49 29 29 (Hyderabad)
9121 41 29 29 / 9121 44 29 29 (Bangalore)
https://t.me/pdf4exams https://t.me/allupscmaterials

3. International Events
3.1 Iran announces latest nuclear deal violations
Context:
Iranian official said the country has doubled the number of advanced centrifuges in operation in clear violation
of the 2015 nuclear deal.
The announcement comes as Iran commemorates the 40th anniversary of the 1979 US embassy hostage crisis
that lasted 444 days.
Highlights:
 TEHRAN says that it will operates twice as many advanced centrifuges banned by the 2015 accord and is
working on a prototype that’s 50 times faster than those allowed by the deal.
 Iran will be operating advanced centrifuges IR-6, which can produce enriched uranium 10 times as fast as
the first-generation IR-1s allowed under the accord.
 The nuclear deal limited Iran to using only 5,060 first-generation IR-1 centrifuges to enrich uranium by
rapidly spinning uranium hexafluoride gas.

Iran nuclear deal: Key details:


In 2015, Iran agreed a long-term deal on its nuclear programme with a group of world powers known as the
P5+1 - the US, UK, France, China, Russia and Germany.
 Under the accord, Iran agreed to limit its sensitive nuclear activities and allow in international inspectors
in return for the lifting of crippling economic sanctions.
 As a part of Joint Comprehensive Plan of Action(JCPOA) iran agreed to reduce its number of
centrifuges(tube shaped machines that help enrich uranium) by two thirds.

La Excellence IAS 74 Website: www.laex.in/testprep


9052 29 29 29 / 9052 49 29 29 (Hyderabad)
9121 41 29 29 / 9121 44 29 29 (Bangalore)
https://t.me/pdf4exams https://t.me/allupscmaterials

 It also agreed to restrictions that would allow it to have enough enriched uranium to maintain the
country’s energy needs without having the ability to build a nuclear bomb. (Reduce stockpile of enriched
uranium by 98% and limit uranium enrichment to 3.67%.)
 Iran agreed to give access to inspectors from the international atomic energy agency( IAEA) , the UN’s
nuclear watchdog agency, to its nuclear facilities, among other facilities. The IAEA has repeatedly found
Iran to be complying with the terms of pact.
 In January 2016, when the IAEA declared Iran was living up to its end of bargain, all nuclear-related
international sanctions against Iran were lifted.
The impact of USA and Iran’s decision on the global arena and on India
 Iran is India’s third largest oil supplier after Iraq and Saudi Arabia. Favourable terms of trade with Iran,
which comprises 60-day trade credit, discount on freight and insurance and payment in rupee terms of 45
percent of total supply from Iran, makes it an important strategic trading partner for India. The rupee
payment mechanism with Iran also helped India save its foreign exchange.
 India will have to consider alternatives to source crude oil for more than 18 percent of import
requirements, which would impact its import bill.
 It will be expensive to change the configuration of state-run refineries, currently equipped to process oil
from Iran, to other grades.
IAEA
The International Atomic Energy Agency (IAEA) is an intergovernmental forum for the scientific and technical
co-operation in the nuclear field world-wide. It was established as autonomous organisation on 29 July 1957.
The prime objective of the forum is to works for the safe, secure and peaceful utilization of nuclear science
and technology. So that science and technology contributes in the international peace and security and also in
the United Nations' Sustainable Development Goals.
There are One Hundred Sixty Eight (168) member states in the International Atomic Energy Agency.
The apex body of the IAEA is responsible for policy-making, programmes and budgets. It consists of
the General Conference of all Member States and the 35-member Board of Governors. The Board meets five
times per year in Vienna.

Mains question
The recent unilateral sanctions of USA and Iran’s action towards USA’s decision will make the Middle-
East region and the world much more volatile. Comment. Examine the options available for the world
and also for India in dealing with the likely fallout of these latest decision. (250W|15M)

3.2 KALAPANI DISPUTE


Nepal government has strongly objected to the inclusion of ‘Kalapani’ under the Indian Territory as per the
newly released political map of India.
Description:
 In the latest map, India included Kalapani into the Pithoragarh district of Uttarakhand. According to it, the
historic Kalapani region forms part of the state of Uttarakhand.
 However Nepal government says that Kalapani is an integral part of the country. Talks are going between
the two countries over this unresolved area.
La Excellence IAS 75 Website: www.laex.in/testprep
9052 29 29 29 / 9052 49 29 29 (Hyderabad)
9121 41 29 29 / 9121 44 29 29 (Bangalore)
https://t.me/pdf4exams https://t.me/allupscmaterials

KALAPANI:

 Kalapani is a valley that is administered by India as a part of the Pithoragarh district of Uttarakhand. It is
situated on the Kailash Mansarovar route.
 The Kali River in the Kalapani region demarcates the border between India and Nepal.
 The Treaty of Sugauli signed by the Kingdom of Nepal and British India (after Anglo-Nepalese War)
in 1816 located the Kali River as Nepal's western boundary with India. The discrepancy in locating the
source of the river led to boundary disputes between India and Nepal, with each country producing maps
supporting their own claims.
Do you know?
 Susta area is one of the disputed territories between India and Nepal
 Susta is located on the bank of the Gandak river
 The change of course by the Gandak river is the main reason for disputes in the Susta area.

Mains question
Discuss the strategic importance of Kalapani for India. (150W|10M)

3.3 Towards a Colombo reset


The election of the new president of Sri Lanka, Gotabaya Rajapaksa strengthened the narrative about
Colombo’s “tilt” towards China and against India. The headline misrepresents the complex power play
involving Beijing, Delhi and Colombo.
Description:
 The Great Game in the Subcontinent is not limited to just India and China. It is quite easy to forget the
considerable interests and influence of many other powers in the region, including the US, European
Union, Japan and Russia.
 The exclusive focus on major power rivalry masks the agency of South Asian political elites and their
capacity to manoeuvre among the major powers.
Background:
 Although the Rajapaksas had blamed India for their defeat in the 2015 elections, they have sought to
make up with Delhi in recent years.
 India has been engaging all the major political formations in Sri Lanka.
 The stage is ready for a reset in the bilateral relations between the two strong governments in Delhi and
Colombo.

La Excellence IAS 76 Website: www.laex.in/testprep


9052 29 29 29 / 9052 49 29 29 (Hyderabad)
9121 41 29 29 / 9121 44 29 29 (Bangalore)
https://t.me/pdf4exams https://t.me/allupscmaterials

India-China:
 The outgoing coalition led by Sirisena and Wickremesinghe proves the irrelevance of labelling
governments in Colombo as “pro-China” or “pro-India”.
 It came to power criticising the Chinese projects in Sri Lanka as financially unsustainable.
 Two years into power, the coalition extended full backing to the Chinese projects. So-called “pro-India”
regime offered China a 99-year lease on the Hambantota project.
 India can’t expect its neighbours to shut down economic and commercial engagement with China.
 There are questions about the terms of China’s assistance on projects, including those under the Belt and
Road Initiative.
 India can only ask Sri Lanka not to take steps with Beijing that threaten India’s security.
 Both need a clear understanding of mutual red lines relating to national security and a political comfort
level to discuss cases that fall within the orange zone.
 That should help prevent the recurrence of the controversy over Chinese submarines in Colombo port as
in 2014.
India-Srilanka:
 Rajapaksas are reported as saying that China is a “trade partner” while India is a “close relative”.
 Other terms used to describe the new policy include “neutrality” and “non-alignment” between major
powers.
 The world rediscovers the geopolitical value of Sri Lanka at the heart of the Indo-Pacific. It has huge
opportunities to leverage its location for national benefit.
 The second structural factor shaping India’s relations with Sri Lanka is the Tamil question. India’s
involvement in Sri Lanka’s tragic civil war has been consequential.
 Delhi has certainly learnt the dangers of being drawn too deep into the domestic conflicts of
neighbouring countries.
 If the new government in Sri Lanka can advance reconciliation with the Tamil minority, it will be easier
to strengthen ties.
 The Western powers have expressed deep concerns about the war crimes in the military campaign
against the LTTE and the need to bring those responsible to book.
Conclusion:
 India knows that too much heat from the West will automatically increase China’s leverage in Colombo.
 If the Tamil question continues to have a big impact on Sri Lanka’s foreign policy, Delhi needs to look
beyond old formulae to try and encourage reconciliation within Lanka and across the Palk Strait with Tamil
Nadu.

Mains Question
Examine the challenges that India faces its bilateral relationship with Sri Lanka. Also, discuss some
measure taken by India to address these challenges in recent times. (250W|15M)

La Excellence IAS 77 Website: www.laex.in/testprep


9052 29 29 29 / 9052 49 29 29 (Hyderabad)
9121 41 29 29 / 9121 44 29 29 (Bangalore)
https://t.me/pdf4exams https://t.me/allupscmaterials

F. SCIENCE AND TECHNOLOGY


1. Health-Nutrition-Diseases
1.1 Sex ratio improves in country
Total fertility rate in 12 States has fallen below two children per woman
According to the Central Bureau of Health Intelligence’s (CBHI) National Health Profile (NHP) 2019, India has
registered an improved sex ratio and a decline in birth and death rates with non-communicable diseases
dominating over communicable in the total disease burden of the country.
Observations:
 Sex ratio (number of females per 1,000 males) in the country has improved from 933 in 2001 to 943 in
2011
 In rural areas the sex ratio has increased from 946 to 949.
 Kerala has recorded the highest sex ratio in respect of total population (1,084), rural population (1,078)
and urban (1,091).
 The lowest sex ratio in rural areas has been recorded in Chandigarh (690)
 The estimated birth rate reduced from 25.8 in 2000 to 20.4 in 2016 while the death rate declined from
8.5 to 6.4 per 1,000 population over the same period.
 The natural growth rate declined from 17.3 in 2000 to 14 in 2016 as per the latest available information.
 The total fertility rate (average number of children that will be born to a woman during her lifetime) in
12 States has fallen below two children per woman and nine States have reached replacement levels of
2.1 and above.
 Delhi, Tamil Nadu and West Bengal have the lowest fertility rate among other States
 It was also observed that non-communicable diseases dominated over the communicable in the total
disease burden of the country.
National Health Profile (NHP) is an annual publication of the Central Bureau of Health Intelligence (CBHI).
The first release of the annual publication was in 2005. It covers all the major information on Demography,
Socio-Economic Status, Disease Morbidity & Mortality, Healthcare Finance, Human Resources in Health and
Healthcare Infrastructure. NHP is an initiative which is at par with international standards of data
publications.

Mains questions
Analyze the reasons responsible for India not being able to reduce the Total Fertility Rate (TFR) to its
desirable objective in some states in India. (150W|10M)

1.2 Indian lungs under extreme stress


Context:
Acute respiratory infections (ARI) accounted for 69.47% of morbidity last year which was the highest in the
communicable disease category, leading to 27.21% mortality.
Highlights:
 Andhra Pradesh, Gujarat, Karnataka, Kerala, Tamil Nadu, Uttar Pradesh and West Bengal reported a large
number of patients and fatalities due to ARI as per the National Health Profile-2019, which was recently
released by the Union Health Ministry.
 According to World Health Organization, acute respiratory infection is a serious ailment that prevents
normal breathing function and kills an estimated 2.6 million children annually
 Indians face the double burden of heavy air pollution in addition to the high rate of ARI which hits children
the hardest.

La Excellence IAS 78 Website: www.laex.in/testprep


9052 29 29 29 / 9052 49 29 29 (Hyderabad)
9121 41 29 29 / 9121 44 29 29 (Bangalore)
https://t.me/pdf4exams https://t.me/allupscmaterials

What is the issue?


 When you breathe in polluted air, particles and pollutants penetrate and inflame the linings of your
bronchial tubes and lungs.
 This leads to respiratory illness such as chronic bronchitis, emphysema, heart disease, asthma, wheezing,
coughing and difficulty in breathing.
Who are the main victims?
 Children seem to be most vulnerable to the harmful effects of air pollution.
 Children are particularly susceptible as they breathe through their mouths, bypassing the filtering
effects of the nasal passages and allowing pollutants to travel deeper into the lungs.
Impact on Pregnant Women:
 The current level of air pollution poses a high risk to pregnant women and the baby.
 The foetus receives oxygen from the mother, and if she is breathing polluted air, it can increase the health
risk of unborn babies.
 Pregnant women in the first trimester need to be more careful as risk increases and pollution can cause a
medical condition called intrauterine inflammation.
 Prenatal exposure to pollutants increases risk of preterm delivery and low birth weight, factors that can
lead to developmental disabilities later on.

Mains question
Air pollution in India is one of the serious threats that undermine our demographic dividend. Elaborate.
Examine how the recent development in science and technology aid in fighting this challenge.
(150W|10M)

1.3 Anaemia
Context:
A study titled ‘Anaemia among men in India revealed that Anaemia among men in India is an important public
health problem.
Objectives :
 This study aimed to determine the national prevalence of anaemia among men in India.
 How the prevalence of anaemia in men varies across India among states and districts and by socio
demographic characteristics.
 Whether the geographical and socio demographic variation in the prevalence of anaemia among men is
similar to that among women to inform whether anaemia reduction efforts for men should be coupled
with existing efforts for women.

La Excellence IAS 79 Website: www.laex.in/testprep


9052 29 29 29 / 9052 49 29 29 (Hyderabad)
9121 41 29 29 / 9121 44 29 29 (Bangalore)
https://t.me/pdf4exams https://t.me/allupscmaterials

What is anameia?
The World Health Organization defines anaemia as a condition in which the number of red blood cells or their
oxygen-carrying capacity is insufficient to meet physiological needs.
Anaemia in men can cause fatigue, lethargy, creates difficulty in concentrating, thereby reducing the quality of
life and decreasing economic productivity.
Do you know?
 An estimated 1.9 billion people had anaemia in 2013, which is 27% of the world’s population, and 93%
of these cases occur in low- and middle-income countries.
 Factors such as consuming smokeless tobacco, being underweight, level of urbanisation and household
wealth are associated with a higher probability of developing the disease.

Mains question
Anemia among both men and women is increasing in recent times. What are the impacts of this growing
anemia in our society? Discuss some of the measures taken to prevent this by the government
(150W|10M)

1.4 Pneumonia, diarrhoea still a big threat.


Context:
The 10th pneumonia and diarrhoea progress report card has found that Progress short on ensuring children
have access to prevention and treatment services.
Key findings of the report:

 This report analyses how effectively countries are delivering 10 key interventions, including breastfeeding,
vaccination, access to care, use of antibiotics, ORS, and zinc supplementation.
 Globally, pneumonia and diarrhoea led to nearly one of every four deaths in children under five years of
age in 2017.
 India accounts for a major portion of these deaths. Only half of the Indian children with diarrhoea receive
ORS (oral rehydration solution) and 20% receive zinc supplementation to help protect against, prevent and
treat pneumonia and diarrhoea.
La Excellence IAS 80 Website: www.laex.in/testprep
9052 29 29 29 / 9052 49 29 29 (Hyderabad)
9121 41 29 29 / 9121 44 29 29 (Bangalore)
https://t.me/pdf4exams https://t.me/allupscmaterials

Do you know?
Additional reports from organisations like Save the Children and UNICEF have noted that, in 2017, the
highest risk factors for child pneumonia death in India were: 53% caused by child wasting, 27% by outdoor
air pollution, and 22% caused by indoor air pollution from solid fuels.

World Pneumonia Day:


World Pneumonia Day is being marked every year since 2009. The day was hosted by the Global Coalition
against Child Pneumonia for the first time in 2009. The coalition includes several organizations across the
world like CARE, Save the Children, PATH, UNICEF, WHO, etc.
Theme for 2019: “Healthy Lungs for all”
What is Pneumonia?
Pneumonia is an infection that inflames the air sacs in one or both lungs. The air sacs may fill with fluid or
pus (purulent material), causing cough with phlegm or pus, fever, chills, and difficulty breathing. A variety of
organisms, including bacteria, viruses and fungi, can cause pneumonia.
Highlights:
 This year global organizations have come together and created an initiative “Stop Pneumonia”. It
includes WHO, UNICEF, ISGlobal, Save the Children, etc. The 2019 World Pneumonia Day is being
launched by this organization.
 The initiative will hold a conference in Barcelona, Spain between 29-31 January, 2020 to set
pathways for the governments to meet the SDGs on child survival and to achieve GAPPD.
Objective:
 To create awareness about the world’s leading killer of children under 5 (according to WHO)
 To create action plans to combat pneumonia
 To prevent and treat pneumonia
GAPPD:
The Global Action Plan for Prevention and Control of Pneumonia and Diarrhea was launched by WHO and
UNICEF. It aims to accelerate pneumonia control. The aim of the organization is to reduce deaths due to
pneumonia to less than 3 per 1000 live births by 2025.
India’s plans:
 The Ministry of Health and Family Welfare has launched programs under MAA, UIP and ICDS initiatives
to eradicate pneumonia.
 Also, the government has taken actions at the grass root levels to create awareness on preventing
pneumonia through ASHA and Anganwadi workers.
 According to WHO ,around 300,000 die due to Pneumonia In India.
 It is concentrated in few handful of countries all over the world. It includes Nigeria, China, Pakistan,
India, Ethiopia, Democratic Republic of Congo
 The United Nations celebrated the first World Pneumonia Day in 2009.

La Excellence IAS 81 Website: www.laex.in/testprep


9052 29 29 29 / 9052 49 29 29 (Hyderabad)
9121 41 29 29 / 9121 44 29 29 (Bangalore)
https://t.me/pdf4exams https://t.me/allupscmaterials

1.5 Parliamentary Panel Report of Cancer Patients


Context:
The Department-related Standing Committee on Science has expressed serious concern over the rising
number of cancer patients in the country.
Key Findings:
 The incidence, or the number of newly diagnosed cases of cancer annually, is about 16 lakh. The disease
kills 8 lakh people annually. Among these are 140,000 fresh cases of breast cancer, 100,000 cervical cancer
cases, and 45,000 cases of oral cancer among women.
 Among men, the top three cancers with the highest incidence are those in the oral cavity (1,38,000 cases),
cancer of the pharynx (90,000) and those of the gastro-intestinal tract (2,00,000).
Recommendations:
 Setting up of a High-Level Steering Committee to work out modalities for rolling out of a Hub and Spoke
Model in a time-bound manner. This approach is already practice in Punjab which has a network of
centers, or hubs, capable of treating complex forms of cancer.
 It urged the government to expand and upgrade cancer treatment infrastructure for affordable and
quality care by enlarging the network of the Mumbai-based Tata Memorial Centre (TMC).

What is the status of cancer in India?


 According to the Indian Council of Medical Research, annual cancer deaths, now at 7.3 lakh, are set to
increase by another 20 per cent to 8.8 lakh by 2020.
 The number of new cases each year is expected to touch 17.3 lakh by 2020.
 The National Cancer Registry Programme, started in the early 80s, maintains registries of 23 different
cancers, generating valuable data for collaborative research.
What are the challenges before India in treating cancer?
 Although medical science has been able to reduce mortality from cancers significantly, the beneficiaries
have largely hailed from the rich countries.
 In India, most cancer research is carried out in tertiary cancer centres and specialised institutions of
biomedical science, against well-developed cancer research networks in high-income countries.
 The rising burden of cancer in India acts as a major drain on research time, particularly for clinical staff.
 According to estimates, there are only 2,000 cancer specialists in India for 10 million patients.
 Besides, infrastructure to support cancer research has a long way to go.
 With the cost of treatment forbidding and most of India’s health spend not covered by health
insurance, cancer not only kills but also drives families into poverty and debt.

Mains questions
Indian lifestyle seriously raises more cancer risk in our society. Elucidate. Examine the challenges that
India faces in treating this challenge. (150W|10M)

La Excellence IAS 82 Website: www.laex.in/testprep


9052 29 29 29 / 9052 49 29 29 (Hyderabad)
9121 41 29 29 / 9121 44 29 29 (Bangalore)
https://t.me/pdf4exams https://t.me/allupscmaterials

1.6 India is home to 77 million diabetics


Context:
On International Diabetes Day, the International Diabetes Foundation (IDF) Diabetes Atlas makes it clear
that One in six people with diabetes in the world is from India.
Key findings of the report:
 India continues to be home to the second-largest
number of adults with diabetes worldwide.
 India was the largest contributor to diabetes mortality
with more than 1 million estimated deaths attributable
to diabetes and related complications, in the larger
South East Asian region.
 The number of people with diabetes is predicted to
rise to 578 million by 2030 and to 700 million by 2045.
 Diabetes was responsible for an estimated $760 billion
in health expenditure in 2019.
 Diabetes is among the top 10 causes of death, with
people under the age of 60 accounting for almost half
the deaths.
 One in six live births is affected by hyperglycaemia in
pregnancy.
Poor health expenditure:
 Low health expenditure – Diabetes, related health expenditure in the South East Asia, at $8.1 billion in
2019, was the lowest total of all IDF regions.
 The highest percentage was in Mauritius (16.9 per cent), and the lowest was in Nepal (4.2 per cent).
 In India, which accounts for 87.9 per cent of adults with diabetes in the Region, $92 was spent per person.
World diabetes day:
Observed on: November 14. It is because the day marks the birth anniversary of scientist and Nobel
laureate Frederick Banting who discovered insulin (in 1922).
Objective: To create awareness about the effects of diabetes globally.
Origin: It was started in 1991 by the International Diabetes Federation (IDF) and the World Health
Organization (WHO). The day became an official United Nations Day in 2006.
Diabetes in India:
Significant numbers of Indians already affected by the disease and it magnifying overall health and wellbeing
on factors ranging from heart disease to eye health. The disease is one of the fastest-growing conditions in the
country.
Footprint of diabetes is in India:
 Ten percent of Indian youth aged ten to nineteen being prediabetic.
 India will see 98.0 million cases of type-2 diabetes alone by 2030.
 Lifestyle factors amplifying the risk for diabetes are on the rise in India, obesity in particular.
 This is driven by the increased availability of sugary food and drinks and fast food enabling increasing
numbers of people to consume ‘westernised’ diets, coupled with more Indians living sedentary lifestyles
foregoing physical activity.
 The Government of India are taking steps to address these issues, encouraging Indians to ‘eat right’ and
exercise more through its ‘Fit India’ campaign.

Mains question
Diabetes continues to be a disease of epidemic proportions in India. Discuss the reasons for this rise in
diabetes prevalence and also suggest some measures taken to combat this challenge. (250W|15M)

La Excellence IAS 83 Website: www.laex.in/testprep


9052 29 29 29 / 9052 49 29 29 (Hyderabad)
9121 41 29 29 / 9121 44 29 29 (Bangalore)
https://t.me/pdf4exams https://t.me/allupscmaterials

1.7 National Health Profile 2019


Context:
The 14th edition of National Health Profile-2019 (NHP-2019) report was unveiled by Union Health Minister.
Highlights:
 A comprehensive report highlighting India’s current health status, trends in demography, disease profile
(communicable and non-communicable/ lifestyle diseases).
 The health finance section provides an overview of health insurance and expenditure on health, both
public and Out of Pocket Expenditure (OOP)etc.
 The section on human resources provides an overview of availability of manpower working in the health
sector, while health infrastructure section provides details of Medical and Dental Colleges, AYUSH
Institutes, Nursing Courses and Paramedical Courses etc.
Significance:
 The report would help policy makers to address the new challenges in our health system such as
epidemiological and demographic transitions and health-impacting environmental changes.
 The profile will be a major source of information various communicable and non-communicable diseases
that are not covered under any other major programs.
National Health Profile:
NHP is an annual publication of the Central Bureau of Health Intelligence (CBHI). The first release of the
annual publication was in 2005. It covers all the major information on Demography, Socio-Economic Status,
Disease Morbidity & Mortality, Healthcare Finance, Human Resources in Health and Healthcare
Infrastructure. NHP is an initiative which is at par with international standards of data publications.
Objectives:
 The objective of NHP is to create a versatile data base of health information of India and making it
available to all stakeholders in the healthcare sector.
 This data base of health information is comprehensive, up-to-date and easy to access.
 This publication takes into account recent trends in demography, disease profile (communicable and non
communicable/lifestyle diseases) and available health resources which define a country's health status.
 The disease profile has been presented following standard coding from Family of International
classification (FIC). This improves interoperability of the data internationally.
 The purpose is to provide relevant information for planning and decision making on an informed basis to
the planners, policy makers, health administrators, research workers and others engaged in raising the
health and socio-economic status of the community.
 This publication is useful for medical post graduates and trainees of medical and paramedical personnel.

1.8 NATIONAL HEALTH STACK (NHS) AND NATIONAL DIGITAL HEALTH BLUEPRINT (NDHB)
The NITI Aayog came out with a blueprint of the the National Health Stack (NHS), a shared digital healthcare
infrastructure, with a view to implement the Centre's flagship scheme Ayushman Bharat and other public
healthcare programmes in the country
WHAT IS NHS?
 The NHS is proposed to be the country's first futuristic nationally shared digital healthcare infrastructure
usable by both the Centre and states across public and private sectors
 There are five components of NHS
 National Health Electronic Registries: will create a single source of data for both beneficiaries and
provider, incorporating existing registries, and overcome data duplication and redundancy
 Coverage and Claims platform: Meant to support expansion of NHPS via claims engines and fraud
detection service
 Federated Personal Health Records (PHR) Framework: to make health profiles of individuals for own
access and for medical research

La Excellence IAS 84 Website: www.laex.in/testprep


9052 29 29 29 / 9052 49 29 29 (Hyderabad)
9121 41 29 29 / 9121 44 29 29 (Bangalore)
https://t.me/pdf4exams https://t.me/allupscmaterials

 National Health Analytics Platform: to provide anonymised and aggregated health data for targeted
policy-making, for instance, through improved predictive analytics
 Other horizontal Components: includes among other things creation of a unique Digital Health ID,
Health Data Dictionaries and Supply Chain Management for Drugs, payment gateways, etc
What is NDHB?
 NDHB is the architectural document for the implementation of NHS
 It’s vision is to create a national digital health ecosystem that supports universal health coverage
in an efficient, accessible, inclusive, affordable, timely and safe manner, and ensuring the
security, confidentiality and privacy of health related personal information.
Why THE NHS IS NECESSARY?
 Currently apart from Ayushman bharath-PMJAY, there are many secondary and tertiary schemes running
in the states. Hence there is a need for interoperability of these various health schemes
 Also there is an urgent need for integration of two arms of Ayushman bharath- Health and wellness
centres (primary arm) and PMJAY(secondary and tertiary care arm). with out integration PMJAY would
end up becoming a perpetual drain of resources
 NHS seeks to provide solution to both these challenges
BUT WHAT ARE THE CHALLEENGES WITH NHS?
 One of the biggest concern is regarding data security and privacy of patients
 More than year after justice srikrishna committee prepared a draft privacy law, there has been a little
meaningful movement on it
Way forward
 There is a need to implement a data privacy law.
 It is also important that the data captured is used and disclosed in a manner appropriate with law.
1.9 Jansankhya Sthirata Kosh
Highlights:
 Jansankhya Sthirtha Kosh, is an autonomous body under Ministry of Health and Family Welfare,
implemented the following schemes:
 Prerna Scheme (for delaying marriage, childbirth and spacing),
 Santushti scheme (PPP for sterilization services), and
 National Helpline (for information on family planning).
 Jansankhya Sthirtha Kosh has been discontinued on 08/02/2019
 Steps being taken by the Government to Control Population Growth:
 Mission Parivar Vikas:
 Aim: For substantially increasing access to contraceptives and family planning services in 146 high fertility
districts with total fertility rate (TFR) > 3 in 7 high focus states.
 New Contraceptive Choices: New contraceptive i.e., injectable contraceptive Centchroman have been
added to the existing basket of choices.
 A new method of IUCD insertion immediately after delivery i.e., post-partum IUCD(PPIUCD) has been
introduced.
 Redesigned contraceptive packaging – The packaging has now been improved and redesigned so as to
increase the demand for these commodities.
 Compensation Scheme for sterilization acceptors
La Excellence IAS 85 Website: www.laex.in/testprep
9052 29 29 29 / 9052 49 29 29 (Hyderabad)
9121 41 29 29 / 9121 44 29 29 (Bangalore)
https://t.me/pdf4exams https://t.me/allupscmaterials

 Clinical Outreach Teams (COT) scheme


 Scheme for Home delivery of Contraceptives by ASHAs at doorstep of beneficiaries
 Scheme for ASHAs to ensure spacing in births.
 Scheme for provision of Pregnancy Testing Kits in the drug kits of ASHAs for use in communities.
 Family Planning Logistic Management and Information System (FP-LMIS): A dedicated software to
ensure smooth forecasting, procurement and distribution of family planning commodities across all
the levels of health facilities.
 National Family Planning Indemnity Scheme (NFPIS) under which clients are insured in the
eventualities of death, complication and failure following sterilization.
 Improved Demand Generation activities through a 360o media campaign.
1.10 Mental Health Awareness Initiatives
Highlights:
 To address the burden of mental disorders, the Indian government has been implementing National
Mental Health Programme (NMHP) since 1982.
 DMHP(District Mental Health Programme) has been implemented under NMHP in 655 districts of country
for early detection, management and treatment of mental disorders/illnesses.
 To address the shortage of qualified mental health professions in the country, Government under NMHP is
implementing Manpower Development Schemes for establishment of Centres of Excellence and
strengthening/establishment of Post Graduate (PG) Departments in mental health specialities.
 The government has also enacted the Mental Healthcare Act, 2017.
o This Act aims at protection and promotion of rights of persons with mental illness during the
delivery of health care in institutions and in the community.
o This act also ensures health care, treatment and rehabilitation of persons with mental illness
within a rights based framework to achieve the greatest public health good.
 The government has also spending considerable amount of money in under Information, Education and
Communication activities to generate awareness among the masses about mental health illnesses.
 Every year 12lakhs have provided to each district for targeted intervention with respect to DMHP with the
following objectives:
o Providing class teachers with facilitative skill to promote among their students
o Providing class teachers with knowledge and skill to identify emotional conduct, scholastic and
substance use problems in their students
o Providing class teachers with a system of referral for students with psychological problems to the
District Mental Health Team for inputs and treatment.
o To involve other stakeholders like parents, community leaders to enhance development
of adolescents, etc.

Mains question
India is woefully and inadequately equipped to prevent ‘mental health epidemic’. Comment.
(150W|10M)

La Excellence IAS 86 Website: www.laex.in/testprep


9052 29 29 29 / 9052 49 29 29 (Hyderabad)
9121 41 29 29 / 9121 44 29 29 (Bangalore)
https://t.me/pdf4exams https://t.me/allupscmaterials

2. Space
2.1 Navigation in Indian Constellation
Context:
 The navigation system that Indians use on their mobile phones and cars could be set for a reboot.
 It has been reported that ISRO is in talks with processing chip manufacturers such as Qualcomm to
substitute the existing Global Positioning System (GPS) with the Indian version of satellite navigation.
Navic:
The Indian Regional Navigation Satellite System (IRNSS), with an operational name of NAVIC (acronym
for NAVigation with Indian Constellation) is an autonomous regional satellite navigation system that provides
accurate real-time positioning and timing services.

Keyfacts :
 The fully deployed NAVIC system consists of 3 satellites in GEO orbit and 4 satellites in GSO orbit,
approximately 36,000 km altitude above earth surface.
 The system consists of a constellation of seven satellites and a support ground segment. Three of the
satellites in the constellation are located in geostationary orbit (GEO) at 32.5° East, 83° East, and
131.5° East longitude.
 The other four are inclined geosynchronous orbit (GSO). Two of the GSOs cross the equator at 55° East
and two at 111.75° East.
 The regional navigation satellite system can provide accurate position information service to users in India
and the region, extending up to 1,500 km from its boundary, which is its Primary Service Area.
Other countries to have positioning system:
 The GPS is a satellite-based radio navigation system that is owned by the United States government and
operated by the United States Air Force.
 Apart from GPS, there is GLONASS of Russia, Galileo of the European Union and BeiDou Navigation
Satellite System (or BDS) of China.

Mains question
NAVIC does not only increase the standard and quality of life in India for its citizens but also increases the
strategic depth for our country. Elaborate. (150W|10M)

La Excellence IAS 87 Website: www.laex.in/testprep


9052 29 29 29 / 9052 49 29 29 (Hyderabad)
9121 41 29 29 / 9121 44 29 29 (Bangalore)
https://t.me/pdf4exams https://t.me/allupscmaterials

2.2 DWARF PLANETS


Scientist have claimed for a new dwarf planet named ‘Hygiea’
Description
 It lies in the asteroid belt between mars and Jupiter
 It has satisfied all the criteria set by ‘International Astronomical Union’
 If confirmed hygiea becomes the 6th and the smallest dwarf planet in the solar system
 The observations are made through European Space Organisation’s ‘SPHERE’ instrument at the VERY
LARGE TELESCOPE(VLT)
Criteria set by international astronomical union for a dwarf planet
Four criteria have been set
a. It orbits around the sun
b. It is not a moon
c. It has not cleared the neighborhood around it’s orbit
d. It has enough mass that its own gravity pulls it in to a roughly spherical shape

Do you know?
As of today there are five dwarf planets in our solar system. The most famous one Pluto, downgraded from
the status of planet in 2006.The other four in the order of size are Eris, Makemake, Haumea and ceres

2.3 Spicule in sun


Context:
An international team of researchers of Peking University, China; including one at Indian Institute of
Astrophysics, Bengaluru, has unraveled why the Sun’s atmosphere is hotter than its surface.
Background
 The temperature at the core of the Sun is nearly 15 million degrees Celsius, while that at its surface layer,
known as the photosphere, is merely 5,700 degrees C.
 The natural thing to expect is that still further outwards, in its atmosphere, known as the corona, the
temperatures would be comparable to that at the surface (photosphere).
 However, the temperature of the corona is much higher. It starts increasing outside the
photosphere, reaching a value of about one million degrees or more in the corona. This is called Corona’s
Heating Puzzle.
Spicules in the Sun:

La Excellence IAS 88 Website: www.laex.in/testprep


9052 29 29 29 / 9052 49 29 29 (Hyderabad)
9121 41 29 29 / 9121 44 29 29 (Bangalore)
https://t.me/pdf4exams https://t.me/allupscmaterials

 The key to the puzzle lies in geyser-like jets known as solar spicules that emanate from the interface of the
corona and the photosphere.
 While in a photograph these look like tiny hairlike projections They extend up to 10,000 km (6,000 miles)
and, although they fall back to the Sun, are thought to contribute to the solar wind by feeding material
into the corona.
 It has been suspected that these spicules act as conduits through which mass and energy from the lower
atmosphere bypass the photosphere and reach the corona.
 The present study, has deciphered how these spicules form and also shows that they act as conduits
through which hot plasma is carried into the corona region.
Do you know:
 About 100,000 spicules are active on the Sun’s surface at any given time. They rise from the lower
chromosphere at about 20 km (12 miles) per second to a height of several thousand kilometres, and
then within 10–15 minutes they disperse or collapse.
 Although they are invisible in white light, early observers could see them in the hydrogen alpha (Hα)
emission line with a spectrograph and compared them to a “burning prairie.”
 Spicules are not connected with sunspots or plages and are also absent from the channels where solar
prominences appear.

2.4 Arrokoth
The most distant space object ever seen up close has been recently named as ‘Arrokoth’.
Description:
 NASA’s New Horizons spacecraft flew by the snowman figured ice mass in December 2018, some 1.6
billion kilometers beyond Pluto. At that point it was provisionally called Ultima Thule.
 “NASA's New Horizons mission has broken all records of space exploration by exploring deep space
objects beyond Pluto, such as Ultima Thule.
Arrokoth
 The International Astronomical Union and Minor Planets Center, the global body for naming Kuiper Belt
objects, has given a name for the most distant world ever explored by a space mission. It is called
Arrokoth.
 The New Horizons team of NASA proposed the name to the International Astronomical Union and
Minor Planets Center. In the language of the Powhatan tribe, Arrokoth means “sky”.
New Horizons mission
 NASA launched the New Horizons mission in January 2006.
 After crossing by Pluto in 2015, in 2019 it flew by Arrokoth. This remains the “farthest flyby ever
conducted.”
Kuiper Belt
 The Kuiper Belt is a region of the Solar System that exists beyond the eight major planets, extending
from the orbit of Neptune (at 30 AU) to approximately 50 AU from the Sun.
 The Kuiper Belt has thousands of similar icy bodies. It is known as the third zone of the solar system, after
the zone hosting the gas planets in our solar system.
 It contains hundreds of millions of small icy bodies that are thought to be left over material from the
formation of the outer planets.
 Some of the solar system’s moons are thought to have originated there, such as Neptune’s Triton and
Saturn’s Phoebe.

La Excellence IAS 89 Website: www.laex.in/testprep


9052 29 29 29 / 9052 49 29 29 (Hyderabad)
9121 41 29 29 / 9121 44 29 29 (Bangalore)
https://t.me/pdf4exams https://t.me/allupscmaterials

2.5 Space Internet


Understanding space Internet
Description:
The SpaceX, the world’s leading private company in space technology fired a spray of 60 satellites into orbit. It
is intended to eventually evolve into a constellation of nearly 12,000 satellites aimed at providing low-cost and
reliable space-based Internet services to the world.
 Space internet services are mainly used to ensure that reliable and uninterrupted Internet services are
ensured across the world.
 Currently, about 4 billion people, more than half the world’s population, do not have access to reliable
Internet networks.
 And that is because the traditional ways to deliver the Internet — fibre-optic cables or wireless networks
— cannot take it everywhere on Earth.
 In many remote areas, or places with difficult terrain, it is not feasible or viable to set up cables or mobile
towers. Signals from satellites in space can overcome this obstacle easily.
Space Internet
 SpaceX announced the satellite Internet constellation in January 2015, and launched two test satellites
in February 2018.
 The Starlink network, as the project is called, is one of several ongoing efforts to start beaming data
signals from space, and also the most ambitious.
 This launch is the first operational batch of what is intended to eventually evolve into a constellation of
nearly 12,000 satellites.
 They are aimed at providing low-cost and reliable space-based Internet services to the world.
 Additionally, satellites in these orbits travel at more than double the speed of satellites in geostationary
orbit — about 27,000 km per hour — to balance the effects of gravity.
Placing satellites in lower orbits help:
 One big advantage of beaming signals from geostationary orbit is that the satellite can cover a very
large part of the Earth.
 Signals from one satellite can cover roughly a third of the planet — and three to four satellites would
be enough to cover the entire Earth. Also, because they appear to be stationary, it is easier to link to
them.
 But satellites in geostationary orbit also have a major disadvantage. The Internet is all about
transmission of data in (nearly) real time.
 However, there is a time lag — called latency — between a user seeking data, and the server sending
that data. And because data transfers cannot happen faster than the speed of light (in reality, they
take place at significantly lower speeds), the longer the distance that needs to be covered the greater
is the time lag, or latency.
Limitations
 Three issues have been flagged — increased space debris, increased risk of collisions, and the concern
of astronomers that these constellations of space Internet satellites will make it difficult to observe
other space objects, and to detect their signals.
 Astronomers and scientists have also complained about increased “light-pollution”, a reference to
light reflected from the man-made satellites that can be mistaken for light coming from other
heavenly bodies.
Conclusion:
 To put things in perspective, there are fewer than 2,000 operational satellites at present, and fewer
than 9,000 satellites have been launched into space since the beginning of the Space Age in 1957.
 Most of the operational satellites are located in the lower orbits. The European Space Agency (ESA)
this year had to perform, for the first time ever, a “collision avoidance manoeuvre” to protect one of
its live satellites from colliding with a “mega constellation”.

La Excellence IAS 90 Website: www.laex.in/testprep


9052 29 29 29 / 9052 49 29 29 (Hyderabad)
9121 41 29 29 / 9121 44 29 29 (Bangalore)
https://t.me/pdf4exams https://t.me/allupscmaterials

3. Techonolgy
3.1 Govt. questions WhatsApp after spyware is used for snooping
Those targeted include journalists, lawyers & Dalit and human rights activists
The Centre has sought an explanation from messaging platform WhatsApp after the Facebook-owned
company confirmed that some Indian users of its app came under surveillance using an Israeli spyware. Most
of those targeted in India were journalists, Dalit and human rights activists and lawyers.

Mains question
The use of technology under the garb of protecting fundamental rights and other democratic interest will
end up harming democracy more than anything else. Critically analyze (150W|10M)

3.2 Economic slowdown may lighten India’s carbon burden


Context:
According to an recent analysis published in Carbon Brief (tracks emission and carbon dioxide trends), Carbon
dioxide emissions are poised to grow at their slowest since 2001.
Highlights:
 The rise in C02 emissions from India sees wild swings — 7% in 2014 to 3.5% 2015 and then back to 7.8% in
2018.
 This is the first time that emissions are expected to grow below 3% from the previous year.
 Slower growth in coal-based power generation will also benefit the country’s air quality efforts, as
essentially all coal-fired power plants lack pollution controls commonly required in the EU and China.
Is there any rise in Renewables?
 Industrial coal use fell dramatically in 2017 because of a slowdown in the construction sector and bounced
back in 2018.
 Wind generation increased by 17% in the first six months of 2019 compared to the same period a year
earlier, with solar up 30% and hydro increasing by 22%.
 According to report by the International Energy Emissions Agency , India’s per capita emissions were
about 40% of the global average and contributed 7% to the global carbon dioxide burden compared to
USA - the largest emitter, contributed 14%.
What is India’s Commitment?
As per its commitments to the United Nations Framework Convention on Climate Change, India has promised
to reduce the emission intensity of its economy by 2030, compared to 2005 levels. It has also committed to
having 40% of its energy from renewable sources by 2030.

Mains question
What has been the progress that India has made in realizing its objectives under the Paris climate change
agreement? Examine the challenges which India is facing in realizing its true potential towards meeting
climate change targets. (250W|15M)

La Excellence IAS 91 Website: www.laex.in/testprep


9052 29 29 29 / 9052 49 29 29 (Hyderabad)
9121 41 29 29 / 9121 44 29 29 (Bangalore)
https://t.me/pdf4exams https://t.me/allupscmaterials

3.3 Internet penetration


Context: Kerala’s Finance Minister has announced Kerala on its way to achieve 100% Internet penetration.
Highlights :
 In a recent report by the Internet And Mobile Association of India (IAMAI), Kerala is already halfway to its
goal of 100% coverage.
 FM of kerala stated that the Kerala Fibre Optic Network project, would provide Internet to every
household, including free service to 20 lakh BPL households.
 According to the report ‘India Internet 2019,’ the kerala State’s Internet penetration rate is the second
highest in the country (54%) after Delhi NCR- 69%. the lowest : Odisha (25), Jharkhand (26) and Bihar (28).
 The report also notes that Kerala, Tamil Nadu and Delhi have the highest proportion of female Internet
users.
What is Internet Penetration?
 The relationship between the number of Internet users in each country and its demographic data.
 This means the portion of the population that has access to the Internet. It defines a portion of the digital
divide.
 Indicates the percentage of internet users in any country.
Do you know?
According to Google’s ‘Year in Search - India: Insights for Brands’ report, an estimated 40 million Indians are
joining the internet bandwagon annually, and this rate of expansion is among the fastest in the world.

Key facts:
 At 400 million active internet users, India is the second largest internet user market after China.
 The average mobile data usage per subscriber is pegged at about 8GB a month, which is at par with the
consumption seen in developed markets.
 9 out of 10 new internet users in India are likely to be Indian language users.
 There have been 270% growth year-on-year in voice searches on Google’s platform and consumers across
categories are fast adapting to this new search tool for seeking information and discovering content.
 Audience for online videos is poised to touch 500 million by 2020.

La Excellence IAS 92 Website: www.laex.in/testprep


9052 29 29 29 / 9052 49 29 29 (Hyderabad)
9121 41 29 29 / 9121 44 29 29 (Bangalore)
https://t.me/pdf4exams https://t.me/allupscmaterials

Where does Kerala's Internet access stand compared to other States?


 TRAI data - 2019 show that Kerala is well-placed in terms of Internet connections.
 It stands fourth among all telecom service areas in terms of Internet subscriptions per 100 population,
behind Delhi, Punjab and Himachal Pradesh.
 In urban areas, it does better — it ranks second behind Himachal Pradesh.
 Kerala has about 70 Internet subscriptions per 100 population across service areas
 Kerala is among the States with a huge gap between the number of rural and urban Internet connections.
This is also the case in Delhi and Himachal Pradesh.
Role of Internet Penetration in Economic Development:
 The information technology and mobile phone revolution is bringing much needed economic change to
emerging countries
 Increase in Internet penetration can create significant growth in important sectors such as education,
healthcare, energy, and governance in an economy.
 Helps in understanding the need for bridging the digital divide.
 More and more players across several digital platforms will adopt a regional content strategy to reach out
to the Indian masses.
 The attitudinal shift to online services will aid in digital banking, digital governance etc.

Mains question
The Kerala government’s plan to achieve 100% internet penetration is a step in right direction. Discuss the
benefits accrued to society and the individual from such a plan (150W|10M)

4. In-news
4.1 NuGen Mobility Summit 2019 in Manesar
Context: International Centre for Automotive Technology (ICAT) is organizing the NuGen Mobility Summit
2019
Syllabus (GS3): Science and Technology – developments and their applications and effects in everyday life.
Highlights:
 International Centre for Automotive Technology (ICAT) is organizing the NuGen Mobility Summit 2019
from 27-29th November 2019 in Manesar, Gurugram.
 Objective: To share new ideas, learnings, global experiences, innovations and future technologies trends
for the faster adoption, assimilation and development of advanced Automotive Technologies for a smarter
and greener future.
 It will help in building a platform for bringing together all stakeholders in the automotive industry to
understand global advancements in technologies.
 The New Delhi Director of ICAT while briefing media about event said that the event will be one the
largest automotive events in India.
 The summit will provide insights into the upcoming technologies in the area of smart and green mobility,
first hand feel of vehicle technology trends, interaction with industry experts and OEMs(original
equipment manufacturers) from India and abroad, understanding the associated testing regulations,
future design trends and solutions.

La Excellence IAS 93 Website: www.laex.in/testprep


9052 29 29 29 / 9052 49 29 29 (Hyderabad)
9121 41 29 29 / 9121 44 29 29 (Bangalore)
https://t.me/pdf4exams https://t.me/allupscmaterials

4.2 Global Bio-India Summit


Highlights:
 Union Minister for Science and Technology, Earth Sciences and Health & Family Welfare, will inaugurate
Global Bio-India Summit.
 This event has been organized by Department of Bio-Technology(DBT), Ministry of Science & Technology
and BIRAC(Biotechnology Industry Research Assistance Council).
 The Secretary, DBT & Chairperson, BIRAC has said that the Government has taken this initiative to hold
one the largest biotechnology stakeholders conglomerate in India for the first time.
 This summit will provide an opportunity to showcase the potential of India’s biotech sector to
international community, identify, create opportunities and deliberate on the key challenges in the areas
of Bio-pharma, Bio-Agri, Bio-industrial, Bio-Energy and Bio-services and allied sectors.
 The event has been structured to bring global bio-technology stakeholders including national and
international bodies, central and state ministries, regulatory bodies, SMEs, large industries, bio-clusters,
research institutes, investors, incubators, start-ups and others.
 This event will help harness synergies, discussions, brainstorming among stakeholders to identify an
actionable roadmap to catapult India’s biotech sector on the global map.
 It will also help in further aligning with the Government’s vision and mandate to boost translational
research, product development and commercialization, building new partnerships, long term
partnerships, long term collaborations and investment opportunities.

La Excellence IAS 94 Website: www.laex.in/testprep


9052 29 29 29 / 9052 49 29 29 (Hyderabad)
9121 41 29 29 / 9121 44 29 29 (Bangalore)
https://t.me/pdf4exams https://t.me/allupscmaterials

G. HISTORY
1. In - news
1.1 Berlin Wall
Context:
30 years of the fall of the Berlin wall.
Background:
 Until 1961 East German citizens were allowed to travel to West Berlin. Travelling to West Germany
became difficult after closing the border between East and West Germany in 1952.
 The Wall was erected in 1961 because more than 2.6 million East Germans escaped to West Berlin or
West Germany from 1949 to 1961 (total population of East Germany was about 17 million).
Key facts:
Do you know?
The Berlin Wall was a guarded concrete barrier that physically and ideologically divided Berlin from 1961
to 1989. Construction of the Wall was commenced by the German Democratic Republic on 13
August 1961.

 The Eastern Bloc portrayed the Wall as protecting its population from fascist elements conspiring to
prevent the "will of the people" in building a socialist state in East Germany.
 After 2nd world war : After World War II ended, Germany was divided into four zones, one zone for each
of the main Allied countries: France, the United Kingdom, the United States and the Soviet Union.
 In the British, French and American zones, the economic development continued on capitalist lines. The
two major parties in this zone, were the Christian Democratic Party and the Social Democratic Party.
 In 1948, Britain, France and the United States decided to merge the three zones under their control which
were in West Germany and form a separate government there. In September 1949 these zones were
united and a separate state in West Germany called the Federal Republic of Germany with its capital at
Bonn came into being.
 In East Germany which was under Soviet occupation, the policies pursued were different from those that
had been followed in the western zones. Lands were distributed among peasants and all the major
industries were taken over from private hands and made the property of the state. In 1946 the
Communist Party and the Social Democratic Party in the Soviet zone of Germany merged to form the
Socialist Unity Party of Germany. In October 1949, the Soviet zone became a separate state called
the German Democratic Republic.
 The life in the West was much better than in the East after 1948. West Germany including West Berlin had
got financial help through the Marshall plan from the USA. In East Germany a communist system was
established and many people had to suffer under repressions of the Communist party.
 The Berlin Wall was officially referred to as the “Anti-Fascist Protection Rampart” by GDR authorities,
implying that the NATO countries and West Germany in particular were “fascists.” The West Berlin city
government sometimes referred to it as the “Wall of Shame” while condemning the Wall’s restriction on
freedom of movement.
 Berlin Wall is a symbol of division between East and the West: Berlin Wall not only represented division
between West Germany and East Germany, but also division between the East and the West as a whole.
Events which led to the Fall of Berlin:
 The policies of restructuring and openness initiated by Soviet leader Mikhail Gorbachev paved the way
for the destruction of the symbolic physical barrier. Within two years, the mighty USSR too disintegrated.

La Excellence IAS 95 Website: www.laex.in/testprep


9052 29 29 29 / 9052 49 29 29 (Hyderabad)
9121 41 29 29 / 9121 44 29 29 (Bangalore)
https://t.me/pdf4exams https://t.me/allupscmaterials

 During Gorbachev visit to East Berlin, Gorbachev advised East German leader not to delay reforms. After
days of mass protests, the East German government on November 9th declared that citizens were free to
go the West.This led to thousands of East Germans moving into the West, and literally tearing off the wall.
 The process of ending communist rule in GDR and the reunification of Germany began in 1989 when the
Berlin Wall was opened and political parties which were outside the control of the communist party
(called the Socialist Unity Party) were allowed to function. In early 1990 elections were held and a new
government came to power. On 3 October 1990, the division of Germany was ended and a unified
Germany again emerged.
 In 1989, a series of revolutions in nearby Eastern Bloc countries—Poland and Hungary in particular—
caused a chain reaction in East Germany that ultimately resulted in the demise of the Wall.
 After several weeks of civil unrest, the East German government announced on 9 November 1989 that all
GDR citizens could visit West Germany and West Berlin.
 From November 9th to 11th, West Berliners clambered on to the Wall, leapt down onto East German soil.
East Berliners, men and women formed orderly queues at the Bernholmer Strasse checkpoint at
Prenzlauer Berg, clutching their passports issued by the East German government. The checkpoint was
opened before midnight on November 9, 1989 by the East German police, following which passports were
displayed, and entry gained into West Berlin.
Germany after 30 years of fall of the Berlin wall:
 Germany has become a critical factor not just within the EU, but in the world.
 The richest nation in the EU, fourth in the world, is plagued by its own internal politics. The two major
political parties, CDU (Christian Democratic Party) and SDP (Social Democratic Party) are increasingly
disconnected from the people.
 Germany is now the smoke in the European Union’s eye: The two Germanys have united to become the
EU bully. A large, unified nation, economically dominant, politically monopolistic, crooking its finger at
random EU nations, monitoring their every move, with no room for manoeuvre, paying lip service to the
“oneness of shared democratic values”.
 East Germans now openly question what they have gained from the fall of the Berlin Wall. In the past 30
years, no major new industry has come to East Germany. The government and large industrial companies
are not interested.
 Refugee crisis: Another serious issue has emerged since 2015 — 1.5 million refugees from Syria and the
Middle East. Taxes have risen, accommodation has become scarce, essential commodities cost five times
of what they used to. Water, electricity, gas, public transport, all cost three times as much now.

Mains Question
30 years after collapse of Berlin Wall, new barriers emerging in Europe. Elaborate (150W|10M)

1.2 Suranga Bawadi


Context:
‘Suranga Bawadi’, The water supply structure will enters World Monument Watch list.
Highlights :
 A New York-based NGO has included it in the World Monument Watch list for 2020 along with 24 other
monuments from across the world.
 The monument has been selected under the “Ancient Water System of the Deccan Plateau” by the World
Monuments Fund (the NGO), which monitors restoration of ancient monuments across the globe.

La Excellence IAS 96 Website: www.laex.in/testprep


9052 29 29 29 / 9052 49 29 29 (Hyderabad)
9121 41 29 29 / 9121 44 29 29 (Bangalore)
https://t.me/pdf4exams https://t.me/allupscmaterials

 The World Monuments Fund works in collaboration with the local stakeholders, including the district
administration, the Archaeological Survey of India and local explorers of ancient monuments.
 The Suranga Bawadi is expected to get funds for restoration within the next two years.
 The NGO would also coordinate with the authorities concerned for restoration and create awareness on
its importance.
Suranga Bawadi:
 Suranga Bawadi, is an integral part of the ancient Karez system of supplying water through subterranean
tunnels built during Adil Shahi era in Karnataka (Bijapur).
 The ancient water system ‘Karez’ is believed to the one of the best systems in the world.
 Though the Karez system was built in the 16th century by Ali Adil Shah–I, his successor, Ibrahim Adil Shah–
II, brought in several changes by adding more structures to strengthen it.
 According to historians, the Adil Shahis built the magnificent underground system to supply water to the
city, which had a population of nearly 12 lakh then.
1.3 Lutyens Delhi
Context:
The government has initiated a massive re-plan and redevelopment of New Delhi’s iconic central vista (the
stretch from the gates of Rashtrapati Bhawan to India Gate that was designed by Edwin Lutyens) along with
surrounding buildings spread over an area of approximately 4sq km.
Highlights:
 The mega plan will explore options that include constructing a new Parliament building close to the
present one or modernizing the existing building to meet future needs, constructing a common central
secretariat to accommodate most departments and upgrading the central vista to make it a major tourist
attraction.
The house of the Parliament of India
 The Sansad Bhavan is the house of the
Parliament of India contains the Lok
Sabha and the Rajya Sabha and is
located in New Delhi.
Background:
 It was designed by the British architect
Sir Edwin Lutyens and Sir Herbert
Baker in 1912-1913 so as to construct a
new administrative capital city for
British India.
 It is believed that the circular structure
of the 11th-century Chausath Yogini
Temple may also have inspired the
design of the building.
 Construction of the Parliament House
began in 1921 and it was completed in
1927.
 The opening ceremony of the
Parliament House was performed on
18 January 1927 by Lord Irwin, Viceroy
of India.
 The Parliament Museum which stands
next to the Parliament House in the
building of the Parliamentary Library
was opened in 2006
La Excellence IAS 97 Website: www.laex.in/testprep
9052 29 29 29 / 9052 49 29 29 (Hyderabad)
9121 41 29 29 / 9121 44 29 29 (Bangalore)
https://t.me/pdf4exams https://t.me/allupscmaterials

Features of the Building


 The shape of the building is based on the Ashoka Chakra.
 At the center of the building is the Central Chamber semicircular halls surrounding it were constructed for
the sessions of the Chamber of Princes (now used as the Library Hall), the State Council (now used for the
Rajya Sabha), and the Central Legislative Assembly (now used for the Lok Sabha).
 The building is surrounded by large gardens and the perimeter is covered by sandstone railings.
Proposal for a new building
 The new building is being considered to be remodeled or renovated on account of stability concerns.
 A committee to suggest few alternatives to the current building has been set up by the ex-Speaker, Meira
Kumar.
 The present building suffers from inadequacy of space to house members and their staff and also
structural issues.
 The building also needs to be protected because of its heritage tag.
1.4 Karim Shahi region
Context:
 The items collected from the Karim Shahi region revealed that humans occupied that region from Early
Iron Age to the Early Historic (3,100 – 2,300 years) period.
 This period was considered to be archaeologically silent and was often marked as ‘Dark Age’ as there was
no evidence of settlements
Highlights :
 Researchers from IIT Kharagpur during their geological explorations stumbled upon artefacts like pitchers,
jewellery, jars, figurines.
 Using modern luminescence and radiocarbon methods, the team tried to date them, which revealed that
these items were the earliest to be found in the presently arid Rann of Kutch and the Thar Desert.

Who were these people?


 The Karim Shahi region was found to be very close to sea-level and based on the artefacts collected it has
been pointed out that this region could have been a local trade center.
 The people are believed to have traded jewellery and pottery. Also, Chinese and Persian pottery were
found at the Vigakot site indicating that it was a hub for a long-distance trade both through sea and land
from China through India to West Asia.

La Excellence IAS 98 Website: www.laex.in/testprep


9052 29 29 29 / 9052 49 29 29 (Hyderabad)
9121 41 29 29 / 9121 44 29 29 (Bangalore)
https://t.me/pdf4exams https://t.me/allupscmaterials

1.5 Thiruvalluvar
WHO WAS THIRUVALLUVAR?
 Thiruvalluvar, commonly known as Valluvar, was a celebrated Tamil poet and philosopher
 He is believed to have lived at least in the town of Mylapore (a neighbourhood of the present-
day Chennai), and his floruit is dated variously from 4th century BCE to early 6th century CE.
 Little is known with certainty about his family background, religious affiliation, or birthplace.
 Valluvar has influenced a wide range of scholars down the ages since his time across the ethical, social,
political, economical, religious, philosophical, and spiritual spheres
LITERARY WORKS:
 Tirukkural is the primary work credited to Thiruvalluvar. It contains 1330 couplets, which are divided into
133 sections of 10 couplets each.
 The first 38 sections are on moral and cosmic order (Aram)
 the next 70 are about political and economic matters(porul)
 and the remaining 25 are about pleasure(kamam)
Sangam Literature
 The word ‘Sangam’ is the Tamil form of the Sanskrit word Sangha which means a group of persons or an
association.
 The Tamil Sangam was an academy of poets who flourished in three different periods and in different
places under the patronage of the Pandyan kings.
 The Sangam literature which was largely consolidated from the third Sangam, throws information on
conditions of life of people around the beginning of the Christian era.
 It deals with the secular matter relating to the public and social activities like government, war charity,
trade, worship, agriculture, etc.
 Sangam literature consists of the earliest Tamil works (such as the Tolkappiyam), the ten poems
(Pattupattu), the eight anthologies (Ettutogai) and the eighteen minor works (Padinenkilkanakku) and
the three epics.

1.6 Lala Lajpat Rai


November 17 was the centenary of the death anniversary of Lala Lajpat Rai, the firebrand Indian
nationalist leader affectionately called ‘Punjab Kesari’.
Description:
 Lala Lajpat Rai was born at Dhudike near Ludhiana in Punjab in 1865, Rai studied law at the Government
College, Lahore (now called GCU, Lahore), and had a legal practice in that city.
 Early in life, he became a follower of Dayanand Saraswati, the founder of the Arya Samaj, and went on to
become one of the society’s leaders.
 He died at Lahore in 1928 after he was attacked by police during a protest rally against the Simon
Commission.
 In 1881, he joined the Indian National Congress at the age of 16.
 In 1885, Rai established the Dayanand Anglo-Vedic School in Lahore and remained a committed
educationist throughout his life.
 In 1913, Rai set out for a lecture tour to Japan, England, and the United States, but was forced to stay put
abroad after World War I broke and remained overseas until 1920. During his travels, he met many
diaspora communities and founded the Indian Home Rule League of America in New York City in 1917.
 Rai was elected President of the Indian National Congress during its Special Session in Kolkata in 1920,
which saw the launch of Mahatma Gandhi’s Non-cooperation Movement.
La Excellence IAS 99 Website: www.laex.in/testprep
9052 29 29 29 / 9052 49 29 29 (Hyderabad)
9121 41 29 29 / 9121 44 29 29 (Bangalore)
https://t.me/pdf4exams https://t.me/allupscmaterials

Lala-lajpat Rai’s Participation in freedom struggle:


 Rai, Tilak, and Bipin Chandra Pal (called Lal-Bal-Pal) fervently advocated the use of Swadeshi goods and
mass agitation in the aftermath of the controversial Partition of Bengal in 1905 by Lord Curzon.
 In 1928, the Simon Commission, a British-appointed group of lawmakers arrived in India to study the
implementation of the Government of India Act, 1919 (the Montagu-Chelmsford Reforms). The group of
7 did not consist of a single Indian member, a fact that was heavily resented by the Congress.
 Rai was among the leaders of the movement opposing the Commission and was severely lathi-charged
during a protest in Lahore on October 30, 1928.
 It was after this that Rai famously said, “The blows struck at me today will be the last nails in the coffin
of British rule in India.”

Do you know?
Rai wrote extensively in English and Urdu. His important works include: ‘The Arya Samaj’, ‘Young India’,
‘England’s Debt to India’, ‘Evolution of Japan’, ‘India’s Will to Freedom’, ‘Message of the Bhagwad Gita’,
‘Political Future of India’, ‘Problem of National Education in India’, ‘The Depressed Classes’, and the
travelogue ‘United States of America’.

1.7 National Mission of Cultural Mapping


Context: National Mission on Cultural Mapping will compile data of artists, art forms & geo location with
inputs from Central Ministries, State Governments and art/culture bodies – Minister of Culture.
More about National Mission on Cultural Mapping
 Aim: to preserve and promote this diverse heritage, Ministry of Culture has decided to create a
consolidated database of our cultural assets and resources under its National Mission on Cultural Mapping
and Roadmap.
 This mission is included under the umbrella programme “Ek Bharat Shrestha Bharat.”
Important features:
 The mission involves the transfer of knowledge of our cultural arts and heritage through digital world. For
this, the culture ministry has decided to design web-portal to maintain a database of cultural assets and
resources.
 It involves cultural mapping – collecting, recording, analyzing and synthesizing the information of available
local cultural assets and resources with the help of local artists.
 It aims for statistical data collection of various art forms, about individual artists, communities, cultural
organizations and spaces, festivals and events, heritage properties, etc.
 It envisages creation of cultural knowledge centres as well as facilitating the structural and functional
changes in the existing zonal cultural centres.
 The mission also seeks to open a channel for direct communication between artists and the government.

La Excellence IAS 100 Website: www.laex.in/testprep


9052 29 29 29 / 9052 49 29 29 (Hyderabad)
9121 41 29 29 / 9121 44 29 29 (Bangalore)
https://t.me/pdf4exams https://t.me/allupscmaterials

H. TIT-BITS
1. Hyderabad figures in Unesco list
Selected as Creative City in the category of gastronomy
On the occasion of World Cities Day Hyderabad is selected as @UNESCO CREATIVE CITY in Gastronomy
category, on the occasion of World Cities Day today. It’s among the 2 cities from India in list of 66 cities, other
being Mumbai in Film category.
The UNESCO Creative Cities Network (UCCN) was created in 2004 to promote cooperation with and
among cities that have identified creativity as a strategic factor for sustainable urban development
 By joining the Network, cities commit to sharing their best practices and developing partnerships
involving the public and private sectors as well as civil society.
 The Network covers seven creative fields: Crafts and Folk Arts, Media Arts, Film, Design,
Gastronomy, Literature and Music.
 The Creative Cities Network is a privileged partner of UNESCO, not only as a platform for reflection
on the role of creativity as a lever for sustainable development but also as a breeding ground of
action and innovation, notably for the implementation of the 2030 Agenda for Sustainable
Development.

2. World Tsunami Awareness Day


World Tsunami Awareness Day is observed globally on 5 November 2019. The day was observed to raise
tsunami awareness and share innovative approaches to risk reduction.
The 2019 World Tsunami Awareness Day focuses on promoting targets on the Sendai Seven Campaign. It
focuses on reducing disaster damage to critical infrastructure and the disruption of basic services.
History:
In December 2015, the United Nations General Assembly (UNGA) marked 5 November as World Tsunami
Awareness Day. The observation of the day was initiated by Japan. Japan has experienced destruction because
of the Tsunami over the years. Japan has a major expertise in areas such as tsunami and has built up public
action, early warning, and building back better after a disaster to reduce future impacts.
Sendai Seven campaign:
In the year 2016, the United Nations Office for Disaster Risk Reduction (UNDRR) launched the Sendai Seven
campaign. It aims to promote all the sectors to reduce disaster risk and disaster losses. The campaign has set 7
targets and 38 indicators to measure progress on reducing disaster losses.

3. Vigyan Samagam
Vigyan Samagam, India’s first-ever global Mega-Science exhibition, was inaugurated at the Science City in
Kolkata, West Bengal, on 4 November 2019. The multi-venue Science Exhibition was successfully conducted
previously in Mumbai and Bengaluru. It is now open for the public in Kolkata.
Aim:
The Exhibition will highlight the contribution of India to the world’s science, technology, and researches in
areas like high energy physics, nuclear physics, astronomy and astrophysics, nuclear energy, etc. The
exhibition showcases that DAE not only works on nuclear technologies but also works on fundamental science
and research across the country.

La Excellence IAS 101 Website: www.laex.in/testprep


9052 29 29 29 / 9052 49 29 29 (Hyderabad)
9121 41 29 29 / 9121 44 29 29 (Bangalore)
https://t.me/pdf4exams https://t.me/allupscmaterials

Hosted by:
The partners of the exhibition are the Department of Atomic Energy (DAE), Department of Science &
Technology (DST), National Council of Science Museums (NCSM), Ministry of Culture, and other funding
agencies.
Vigyan Samagam will commence its final leg in New Delhi after Kolkata. It will be put up at the National
Science Centre from 21 January to 20 March 2020. It will remain a permanent exhibition in New Delhi. Vigyan
Samagam will be taken care-off by NCSM.

4. Chinese experts to help in restoration of Notre-Dame.


Context:
On 15 April 2019 the cathedral caught fire, destroying the spire and the "forest" of oak roof beams supporting
the lead roof.
About Notre-Dame:

 Notre-Dame, is a medieval Catholic cathedral on the Île de la Cité in the 4th arrondissement of Paris.
 The cathedral was consecrated to the Virgin Mary and considered to be one of the finest examples
of French Gothic architecture
 Its pioneering use of the rib vault and flying buttress, its enormous and colourful rose windows, as well as
the naturalism and abundance of its sculptural decoration set it apart from the earlier Romanesque style.
 Major components that make Notre Dame stand out include one of the world's largest organs and its
immense church bells.
 The cathedral was one of the most widely recognized symbols of the city of Paris and the French nation.
5. Mother Tongue
In our own words
Mother tongue must be the medium of instruction to preserve India’s cultural diversity, heritage.
Description:
 Every year, The United Nations Educational, Scientific and Cultural Organization (UNESCO) celebrates 21st
February as International Mother Language Day to promote mother tongue-based multilingual education.
 The day is also a reminder of how language connects us, empowers us and helps us to communicate our
feelings to others.
 India is a linguistic treasure-trove. India is widely acknowledged for its extraordinary linguistic and cultural
diversity.
 Language is a tool for intellectual and emotional expression.
 It is a vehicle of intergenerational transmission of culture, scientific knowledge, and a worldview.
 It is the vital, unseen thread that links the past with the present. It evolves with human evolution and is
nourished by constant use.
La Excellence IAS 102 Website: www.laex.in/testprep
9052 29 29 29 / 9052 49 29 29 (Hyderabad)
9121 41 29 29 / 9121 44 29 29 (Bangalore)
https://t.me/pdf4exams https://t.me/allupscmaterials

 Our languages permeate every facet of our day-to-day life and form the very basis of our civilisation. They
are the lifeblood of our identity, both individual and collective.
 They play a significant role in creating and strengthening bonds among people.
Facts:
 The world has over 7,000 languages whereas India alone has about 22 officially recognized
languages, 1635 mother tongues, and 234 identifiable mother tongues.
 More than 19,500 languages and dialects are spoken in India as mother tongues, according to the
Language Census.
 There are 121 languages which are spoken by 10,000 or more people in the country.
Challenges :
 196 languages in India are classified as endangered.
 We are not doing enough to preserve our rich native languages.
 Governments need to be doubly careful while adopting policies regarding the medium of
instruction, particularly at the primary and secondary school levels.
 The mother tongue lays a strong foundation for the expression of creativity.

Steps taken:
 The new draft National Education Policy puts forth a number of suggestions for supporting education in
home languages and mother tongues, tribal as well as sign languages.
 The United Nations has proclaimed 2019 as the International Year of Indigenous Languages to preserve,
revitalize and promote indigenous languages.
 In the Rajya Sabha, a provision has been made for its members to express themselves in any of the 22
scheduled languages.
 The Supreme Court has recently decided to make available its judgments in six Indian languages, to start
with.
 The finance ministry has decided to conduct the examinations for employment in Regional Rural Banks in
13 regional languages, in addition to English and Hindi.
 The Railways and Postal departments started conducting their exams in the states’ official languages.
 In 1999, UNESCO adopted a resolution on multilingual education and suggested the use of at least three
languages in education: The mother language(s), a regional or national language and an international
language.
Way forward:
 People can start using their native languages at home, in the community, in meetings, and in
administration. We must accord a sense of dignity and pride to those who speak, write and communicate
in these languages.
 We must encourage Indian language publications, journals and children’s books.
 Dialects and folk literature must be given adequate focus.
 Language promotion should be an integral part of good governance. Swami Vivekananda once said that
language is the chief means and index of a nation’s progress.
 Make mother tongue as the medium of instruction at the primary level.
 Take all steps to make it the language of administration, banking, and judicial proceedings.
La Excellence IAS 103 Website: www.laex.in/testprep
9052 29 29 29 / 9052 49 29 29 (Hyderabad)
9121 41 29 29 / 9121 44 29 29 (Bangalore)
https://t.me/pdf4exams https://t.me/allupscmaterials

6. MK 45 gun system
US to sell $1 billion worth defence equipment to India.
Description:
 The US State Department has approved the sale of naval guns and other equipment worth $1 billion to
India for use against warships, anti-aircraft and shore bombardment.
 The sale includes 13 MK 45 5 inch/62 caliber (MOD 4) naval guns and some other equipment that will be
manufactured by BAE Systems Land and Armaments.
MK 45 gun system:
 The MK 45 is a fully automatic naval gun system that is installed on ships and provides a Naval
Surface Fire Support (NSFS) range of more than 20 nautical miles (36 km) along with improved
propelling charge.
 This system of guns is currently in use by the US Navy on their fleet of Ticonderoga class cruisers
and Arleigh Burke class destroyers.
 MK 45 is an upgraded version with a 62 caliber barrel, strengthened gun and mount subsystems,
advanced control system enhancements, greater range and firepower, a reduced signature and low
maintenance gun shield.
MK 45 MOD 4 gun
 It is a light-weight version of the MK 42 5 inch/54 caliber gun mount meant to support
expeditionary operations and engage surface and air targets.
 The MOD 4 configuration gun mount is believed to boost the firing range by over 50 per cent,
increasing the speed and range of munitions.
 The principal contractor of the guns for this deal is Minneapolis-based BAE Systems Land and
Armaments with a gun manufacturing unit in Louisville, Kentucky.
 It is meant for both naval gunfire support and to destroy any hostile anti-ship weapons and air
defence systems.
 Other countries that have been sold the MOD 4 naval guns are Japan, Australia and South Korea.
Benefits
 The MK 45 Gun System will provide the capability to conduct anti-surface warfare and anti-air defence
missions while enhancing interoperability with US and other allied forces.
 India will use the enhanced capability as a deterrent to regional threats and to strengthen its homeland
defence.
REPORT/INDEX INSTITUTION
State of World’s Children Report UNICEF

The School Education Quality Index’ NITI Aayog

Global Competitiveness Index World Economic Forum

World Economic Outlook IMF


Global Wealth Report Credit Suisse Group, Switzerland
India Innovation Index NITI Aayog

Global Ease of Doing Business World Bank


World Urbanization Prospects Data United Nations Department of Economic and Social Affairs (UN DESA)

World Vision Report WHO

La Excellence IAS 104 Website: www.laex.in/testprep


9052 29 29 29 / 9052 49 29 29 (Hyderabad)
9121 41 29 29 / 9121 44 29 29 (Bangalore)
https://t.me/pdf4exams https://t.me/allupscmaterials

Joint Military Exercises-ARMY Countries


Dustlik India-Uzbekistan
Ekuverin India- Maldives
Dharma Guardian India-Japan
Yudh Abhyas India-USA
Nomadic Elephant India-Mongolia
Sampriti India-Bangladesh
IMBEX India-Myanmar
Bold Kurukshetra India-Singapore
INDRA India-Russia

Joint Military Exercises-NAVY Countries Joint Military Exercises- Countries


Indo Thai-CORPAT India-Thailand AIRFORCE
Za’ir-Al-Bahr India-Qatar GARUDA India-France
VARUNA India-France
Indradhanush India-UK
SLINEX India-Sri Lanka
Sahyog HOP TAC India-Vietnam AVIAINDRA India-Russia

KONKAN India-UK

Species IUCN Status


Ganges River Dolphin Endangered
Tasmanian Tiger Extinct
Bell Bird Least Concern
Snow Leopard Vulnerable
Asiatic Water Buffalo Endangered
Nilgiri Tahr Endangered
Indian Star Tortoise Vulnerable
Great Indian Bustard Critically Endangered
White Throated Tail Least Concern
Indian Rhino Vulnerable
Purple Frog Endangered
Grizzled Giant Squirrel Near Threatened
Kashmir Stag(Hangul) Critically Endangered
Asiatic Wild Dogs(Dhole) Endangered
Hump Backed Mahseer Critically Endangered

La Excellence IAS 105 Website: www.laex.in/testprep


9052 29 29 29 / 9052 49 29 29 (Hyderabad)
9121 41 29 29 / 9121 44 29 29 (Bangalore)
https://t.me/pdf4exams https://t.me/allupscmaterials

I. YOJANA NOVEMBER 2019-SANITATION FOR A HEALTHY SOCIETY-ESSENCE


SANITATION ECONOMY AND DIGNITY OF SANITATION WORKERS
What is sanitation economy?
Sanitation economy is not just about toilets but it also includes provision of clean drinking water, elimination
of waste and converting them into useful resources and digitised sanitation system that optimises data for
operating efficiencies, maintenance, consumer use and health information insights.
In 2017 Toilet Board Coalition report on India, market opportunities for the sanitation economy, which
showed such opportunities valued at $62 billion a year.
GOVERNMENT INITIATIVES TOWARDS SANITATION
 Swachh Bharat Mission for Urban Areas
 Swachh Bharat Mission (Gramin)
 Swachh Vidyalaya Abhiyan
 Rashtriya Swachhata Kosh
 GOBARdhan scheme
 Jal Jeevan Mission
PEOPLE ‘S POLICY
Strategic focus of Swachh Bharat Mission-“4Ps”
 POLITICAL LEADERSHIP
 PUBLIC FINANCING
 PARTNERSHIPS
 PEOPLE’S PARTICIPATION
Administrative focus of Swachh Bharat Mission
 ADMINISTRATIVE DISRUPTION: Administrative disruption led to efficient on-ground implementation.
It started with the Prime Minister setting a target, a sunset clause for the Mission- 2 October, 2019. A
sunset clause brought with it a sense of urgency and accountability. The deadline drove States to
prioritize SBM-G and led to innovative solutions.
 GROUND LEVEL TEAM: The next important step was building a team of people who believed that the
goal is achievable. Low-hanging fruits were targeted first-the districts with the highest sanitation
coverage-to become ODF on priority. This created a demonstration effect for others to learn from and
created belief in the system.
 PUBLICITY: The SBM-G made sanitation glamorous by engaging extensively with the media, leaverging
popular culture, and associating Bollywood stars, etc.
GRAM PANCHAYATS: BEYOND ODF
LESSONS THAT SHAPED SWACHH BHARAT
The approach to SBM-G Itself was structured to allow more freedom in execution and a few unique advances
included:
• Strong public and political willpower
• Adequate funding
• District-level flexibility in administering the necessary activities and campaigns to increase coverage
• Improving the ratio of financial investment in hardware with strong investment in software (i.e.
behaviour change communication) with the community-level outcomes (like-ODF status) - not single
households in mind.
La Excellence IAS 106 Website: www.laex.in/testprep
9052 29 29 29 / 9052 49 29 29 (Hyderabad)
9121 41 29 29 / 9121 44 29 29 (Bangalore)
https://t.me/pdf4exams https://t.me/allupscmaterials

• Utilising the Community Approaches to Sanitation (CAS) methodology.


• Women –headed households and Scheduled and Castes and Tribes prioritised in the programme,
• Concurrently, the Ministry of Panchayati Raj played a more visible role in strengthening Gram
Panchayats’ (GPs') ability to provide services, which included SBM-G targets.
• With the call to shift towards GP ownership, there have been efforts to strengthen the 3 Fs available
to GPs: Funds, Functionaries, and Functions.
• Through the national Gram Panchayat Develmopment Plans (GPDP) guidance of 2018, Ministry of
Panchayati Raj has made efforts to ensure that GDPDs are appropriately convergent in reflecting how
WASH investments and interventions can be mainstreamed into existing budgetary considerations.
SUSTAINING BEHAVIOURAL CHANGE
Challenges in sustaining a behavioural change
• It is found that the design of the toilet, availability of sanitation materials, access to water, and
political or social leadership account for a higher demand for construction and use of toilets.
• Many villages are not homogenous and are fragmented along the caste and religious lines. Collective
behaviour change in a village is easier when the whole village homogeneous but difficult when there
are more conflicts.
• Furthermore, caste-based notion of purity and pollution makes it difficult to construct pit latrine
which requires emptying it in future.
• The challenge of behaviour change is often compounded by the diversity in Indian society, and
therefore, would require more contextual understanding. As a matter of fact, without having local
knowledge into the fold, the sanitation campaign will lead to fruitless activities.
Recommendations
• The present programme, while widely appreciated leaves a scope of the new adoptees to get back
• To their original behaviour. To prevent this, the programme may include the provision of more than
one toilet for larger households. More emphasis may be given for information dissemination at the
ground level.
• Improvement of sanitation is linked with other indicators of living conditions. Hence, it is important to
have a better infrastructure at the household level as well as public service.
• At the same time, higher income of households with higher purchasing power for durable goods
would lead to better living standards of living and thus sanitation practice. Also, emphasis on female
literacy is imperative for better sanitation coverage.

MAINS PRACTICE QUESTION


The Swachh Bharat Mission (SBM), despite its intent, is less encouraging in terms of the outcome achieved
so far. Critically analyze

La Excellence IAS 107 Website: www.laex.in/testprep


9052 29 29 29 / 9052 49 29 29 (Hyderabad)
9121 41 29 29 / 9121 44 29 29 (Bangalore)
https://t.me/pdf4exams https://t.me/allupscmaterials

J. Editor’s Pick
1. Editorial: Substance across the Arabian Sea
Syllabus: GS2: Bilateral relations.
Context: India-Saudi Arabia relations in recent times have shown a positive trajectory of growth.
Highlights:
1. This trajectory of growth has been maintained in spite of some of the tensions in the region. Example:
India-Pakistan tensions; crises in Yemen, Syria and Afghanistan
2. One of the primary reasons for this growth has been frequent political visits by leaders of both the
countries.
3. In these meetings, both country leaders showed mutual respect and recognition of their country’s core
interests.
Example: Saudi Arabia showed an “understanding” of recent Indian actions in Jammu and Kashmir and
India “strongly condemned” the various attacks on Saudi civilian facilities.
4. Still, there are many areas where the relations between the two countries could improve. Example:
bilateral trade, investment.
5. Trade has drifted downwards largely due to lower crude prices. According to the latest Indian data, the
bilateral trade in the first nine months of 2019 stood at $22,416 million, having fallen by 9.2% over the
corresponding figure in 2018
6. The kingdom’s cumulative investments in India are only $229 million, or 0.05% of the total inbound FDI.
7. Opportunities that India could use to change this situation
 India should play a greater role in Saudi’s Vision 2030 by utilizing its expertise in human resources,
agriculture, research and development in science sector etc
 India could also fast-track some significant project in which Saudi Arabia is interested. Example:
Strategic Petroleum Reserves. If realized, these investments could total nearly $30 billion, catapulting
the kingdom to fourth position among countries investing in India.
8. Setting up of a bilateral Strategic Partnership Council (SPC) to be co-chaired by the Indian Prime Minister
and the Saudi Crown Prince would expedite the decision-making process
9. Other areas where both countries could concentrate to derive mutual benefit: Greater bilateral synergy
in Indian infrastructure, agriculture, start-ups, skilling and IT
Takeaway point:
Though the recent growth in relations between Saudi Arabia and India have been welcome. There still remain
many areas where both countries can work to improve their relations further.
2. Editorial: The future of work
Syllabus: GS2: Issues related to social security
Context: Karnataka government has announced that it would come up with a social security code for workers
in the ‘gig’ economy
Highlights
1. What is a gig economy? It is a labour market characterized by the prevalence of short-term contracts or
freelance work as opposed to permanent jobs. Example: People working in Uber, Ola, Zomato, Swiggy and
UrbanClap
2. Indian policymakers are yet to address the social security questions associated with the gig economy. In
this regard, the step taken by the karnataka government is welcome.
3. However, such steps should not be taken at the cost of affecting business
4. Significance of these digital companies.
 They have created jobs particularly when manufacturing sector in India is suffering

La Excellence IAS 108 Website: www.laex.in/testprep


9052 29 29 29 / 9052 49 29 29 (Hyderabad)
9121 41 29 29 / 9121 44 29 29 (Bangalore)
https://t.me/pdf4exams https://t.me/allupscmaterials

 They also promote skill development and entrepreneurship spirit in the economy
 They have also provided social security to some workers in partnership with the government.
Example: Uber partnered with Ayushman Bharat to facilitate free healthcare for drivers and delivery
partners.
5. The government looks at these platform companies as harbingers of employment while platform
companies like Uber, Ola rely on city markets for their workers to populate the platform and to hopefully
eventually turn a profit. There is no clarity on the role of labour unions in this situation
6. This is one of the areas the government should explore in its social security code along with other
questions relating to how these platform companies can be used for generating employment while at the
same time ensuring social security
Takeaway point: Social security at the expense of profits of business will hamper employment generation.
Profits at the expense of social security of the workers will hamper human development. This dichotomy
should be addressed by our policymakers urgently
3. Editorial: Anarchy in Bolivia; A Bolivian crisis
Syllabus: GS2: International news
Context: The forced resignation of Bolivian President Evo Morales has pushed the country into chaos which
could result in a humanitarian crisis in coming days.
Highlights
1. Who is Evo Morales? Mr Morales, a former coca farmer, was first elected to the office of President in
2006; he was the country's first leader from the indigenous community.
2. The pink-tide movement leader: This movement refers to a socialist turn in various South American
countries in the earlier part of 21st century. Evo Morales rose to power on his socialist credentials during
this movement.
3. What are the achievements of Evo Morales? His rule gave political stability to the poorest country in
South America. Apart from this, his presidency won applauds for fighting poverty and improving Bolivia's
economy. Example: 100% literacy rate was achieved; poverty reduced from 33% to 15% during his
presidency
4. However, he quoted controversy when he wanted to defy constitutional limits to run for a fourth term in
last month October's election.
5. Reason for this crisis
 His narrow win last month, was not accepted by most parties in the opposition. They alleged the
elections were rigged. His offer of holding new elections was also not accepted by these parties which
wanted him to resign immediately
 Eventually, the leader was forced to resign when his police force and armed forces also joined hands
in protesting the presidency of Evo Morales.
 USA has also been accused of interfering in the politics of Bolivia. Evo Morales’s policies in
nationalizing some of the sectors which were against the interests of the America. Furthermore,
America has the history of causing such coup.
6. Effect of this resignation
 Civil liberties of people will be affected in this situation
 The achievements made in the past in terms of reducing poverty, literacy rate will be affected if there
is a political crisis in the world.
 The possibility of violent clashes cannot be ruled out.
 Indigenous community from which Evo Morales hails can also be affected because of this.

La Excellence IAS 109 Website: www.laex.in/testprep


9052 29 29 29 / 9052 49 29 29 (Hyderabad)
9121 41 29 29 / 9121 44 29 29 (Bangalore)
https://t.me/pdf4exams https://t.me/allupscmaterials

Takeaway point: Holding of new elections in the country in the coming days in a transparent and non-
controversial manner is the need of the hour. Emphasis should also be placed on a peaceful transition of
power after the elections are held
4. Editorial: What we owe to the Mahatma
Syllabus: GS1: Indian leaders.
Context: Secularism in India is facing a stiff challenge from rising majoritarian ideology. Gandhian concept of
communal harmony can help us in facing such a challenge
Highlights
1. What is secularism? It is the belief that religion should not be part of the affairs of the state or part of
public education
2. Secularism in India: In India, secularism means a state that is neutral to all religious groups (unlike other
democracies which maintain a strict separation between state and religion). It is also one of the basic
features of our constitution
3. However, the rising majoritarian ideology in the society and in the government is harming the secular
credentials of our country. Example: Hate speeches; political leaders of some parties involving themselves
in communal riots etc
4. Gandhi’s ideas on secularism:
 Gandhi ji emphasized that there should be celebration of various ideas, customs, beliefs rather than
any over emphasis on one religion.
 Gandhi ji also laid emphasis on building trust and sociability among members of various religious
communities in a constructive and mutually beneficial manner.
 He also felt that the responsibility for maintaining communal harmony lies with communities
themselves.
 Gandhi ji believed that in order to promote a certain quality of sociability and fraternity between
communities, government should play a key role. This can only happen if the government is not
aligned towards any one particular religion.
5. Gandhi ji’s idea of communal harmony rests among those common people who are moved by moral and
ethical conviction to ensure a secular world rather than on those whose secular actions are distorted in
the name of power, wealth and fame. Example: Hate speeches made by the political leaders to gain
power.
Takeaway point: Secularism should not just be a word in the constitution but rather a core principle which
moves us towards the future; only then can we realize the conception of Ramrajya that Gandhi ji envisioned.
5. Editorial: The problem with re-basing GDP estimates.
Syllabus: GS3: Indian Economy
Context: Central Statistics Office (CSO) is considering the proposal to replace the gross domestic product
(GDP) series of 2011-12 base year with a new set of National Accounts using 2017-18 as the base-year.
Highlights
1. What is a base year?
 A base year is the first of a series of years in an economic or financial index.
 It is typically set to an arbitrary level of 100. New, up-to-date base years are periodically introduced to
keep data current in a particular index.
 Any year can serve as a base year, but analysts typically choose recent years.
2. Base year plays a crucial role in estimating our Gross Domestic Product (GDP).
3. However, in India, the recent GDP estimates have quoted controversy. Questions have been raised on
account of questionable methodologies and databases used in calculating GDP estimates

La Excellence IAS 110 Website: www.laex.in/testprep


9052 29 29 29 / 9052 49 29 29 (Hyderabad)
9121 41 29 29 / 9121 44 29 29 (Bangalore)
https://t.me/pdf4exams https://t.me/allupscmaterials

4. Various estimates show that GDP figures in India may be overestimated by 0.36 to 2.5 percentage points.
5. One of the reasons for the distrust in the current estimates of the GDP was changing the base year from
2004-05 to 2011-12 in 2015.
6. Such-rebasing are a matter of routine exercise. Re-basing usually results in a marginal rise in the growth
rate because of better capturing of domestic production using improved methods
7. However, the rebasing done recently was controversial. Since:
1. There was a drastic change in some of the key metrics that are important to the economy. Example:
manufacturing sector growth rate for 2013-14, swung from (-) 0.7% in the old series to (+) 5.3% in the
2011-12 series.
2. This high growth was not commensurate with other economic indicators such as bank credit growth,
industrial capacity utilization or fixed investment growth
3. Demonetization undoubtedly dented the growth prospects of India. However, GDP estimates do not
adequately reflect this situation
8. What is the cause of these concerns?
 Database used by CSO of Ministry of Corporate affairs has been cited by many economists of
being unreliable. Example: 42% of the sample companies used under his methodology were not
traceable
 There is no methodology available to capture the value added in the informal or unorganized
sector in the current series (sectors which are still reeling under the double distress of GST and
demonetization)
9. What should be done now?
 Removing the doubts concerning these figures should be the first priority. Trust needs to be brought
back again on growth estimates.
 In this regard, an independent commission of national and international experts to review the GDP
methodology can be setup
Takeaway point: The new rebasing should help in bringing up the credibility that our growth estimates have
lost in recent times. Credible data will help our policymakers to better achieve their objectives.
6. Editorial: Gloom deepens
Syllabus: GS3: Indian Economy
Context: The latest Index of industrial production (IIP) estimates confirms that India is yet to witness revival
of growth from the economic slowdown we are facing.
Highlights
1. The National Statistical Office (NSO) released IIP estimates shows that output shrank by 4.3% in the month
of September
2. What is IIP?
 The Index of Industrial Production (IIP) is an index which shows the growth rates in different industry
groups of the economy in a stipulated period of time.
 IIP is a key economic indicator of the manufacturing sector of the economy.
 There is a lag of six weeks in the publication of the IIP index data after the reference month ends.
 Electricity, crude oil, coal, cement, steel, refinery products, natural gas, and fertilizers are the eight
core industries that comprise about 40 percent of the weight of items included in the Index of
Industrial Production
 Mining, manufacturing, and electricity are the three broad sectors in which IIP constituents fall.
3. Observations made in the latest report:
 Contractions were observed in all manufacturing, mining and electricity sectors

La Excellence IAS 111 Website: www.laex.in/testprep


9052 29 29 29 / 9052 49 29 29 (Hyderabad)
9121 41 29 29 / 9121 44 29 29 (Bangalore)
https://t.me/pdf4exams https://t.me/allupscmaterials

 Investment in the economy has also not picked up. This could be seen from the fall in output of capital
goods.
 Consumer durables also posted a fourth straight contraction. This shows that the consumer demand
has also not revived in the economy
 Infrastructure and construction goods also showed a decline in the recent report released.
4. Effect of this on the Indian economy
 Persisting weakening demand will hurt new orders and business sentiment, thereby by delaying the
economic revival in spite of the steps taken by the government and RBI in recent times. Example:
Corporate tax cut, increase in government spending, reducing interest rates etc.
5. What should be done? An economic slowdown in the economy gives the government an excellent
opportunity to bring in structural reforms. Example: Labour reforms (this was also suggested by IMF),
taxation reforms etc
Takeaway point: Both monetary policy and fiscal policy should be oriented towards changing the present
scenario of Indian economy. Emphasis should be given to structural reforms in both these policies.
7. Editorial: Open, all the same.
Syllabus: GS2: Judiciary
Context: The Supreme Court of India in one of its recent judgments brought the office of Chief Justice of India
(CJI) under the ambit of Right to Information (RTI)
Highlights
1. What is Right to Information (RTI) act?
 Right to Information (RTI) Act 2005 provides timely response to the citizens on their requests for
government information.
 Objective: to promote transparency, empower the citizens and accountability in the working
condition of the Government.
2. Significance of the RTI: The RTI act has empowered ordinary citizenry in holding public authorities
accountable and also gather more information about their representatives. Example: Assets of Ministers
and their representatives.
3. However, judiciary was not hitherto brought under the ambit of the act.
4. The latest verdict by declaring CJI office as a public authority changes the earlier situation.
5. The bench which delivered the verdict made some observations also in this case. Such as:
 The Bench unanimously argued that the right to know under the RTI Act was not absolute and this
had to be balanced with the right of privacy of judges
 The judgment also that disclosure of details of serving judges’ personal assets was not a violation of
their right to privacy.
 The bench also observed there needs to be a proper calibration of transparency under the RTI in light
of the importance of judicial independence. Example: Dealing with cases with respect to judicial
appointments
6. Impact of this judgment:
 Will bring in more transparency and accountability on Indian judiciary
 Trust of the people in the judicial process will increase.
 It will promote citizen activism and also participatory democracy in our country
7. What more is needed to empower RTI act further? There has been a growing consensus on bringing the
political parties also under the ambit of Right to Information (RTI). This would bring in more transparency
with regards to electoral funding

La Excellence IAS 112 Website: www.laex.in/testprep


9052 29 29 29 / 9052 49 29 29 (Hyderabad)
9121 41 29 29 / 9121 44 29 29 (Bangalore)
https://t.me/pdf4exams https://t.me/allupscmaterials

Takeaway point: The recent judgment of the Supreme Court of India has made the Right to Information (RTI)
much more potent tool to bring in participatory democracy in our country
8. Editorial: Back from the brink
Syllabus: GS2: Issues related to Health
Context: USA was able to fight the challenge of measles outbreak it witnessed last year.
Highlights
1. About Measles:
 Measles is a highly contagious viral disease. It remains an important cause of death among young
children globally, despite the availability of a safe and effective vaccine.
 Measles is transmitted via droplets from the nose, mouth or throat of infected persons
 Severe measles is more likely among poorly nourished young children, especially those with
insufficient vitamin A, or whose immune systems have been weakened by HIV/AIDS or other diseases.
 The most serious complications include blindness, encephalitis (an infection that causes brain
swelling), severe diarrhea and related dehydration, and severe respiratory infections such as
pneumonia.
2. What was the reason for the outbreak?
 Inequities in vaccine coverage, or gaps in vaccine coverage between communities, age groups and
geographic areas were the primary reasons for the outbreak of measles
3. Lessons for India from this case:
 India has various pockets of under-immunized children in the country. This can lead to a measles
outbreak just like USA
 The immunisation coverage with two doses at the national level is also far below the World Health
Organization level of 95% needed for protection and elimination
4. In the October 2018-2019 period, India reported 71,834 cases, the third highest number in the world,
according to the WHO.
5. Indian government plans to eliminate measles by 2020
6. What should India do to achieve this objective?
 Improve its national coverage of vaccination
 Improve vaccination coverage for infants in our country
 Improve awareness among the citizens about the measles
 Identify the districts which are suffering from Measles and come up with targeted measures to address
them
Takeaway point: India should ensure that the target it has set for elimination of measles is duly achieved. In
this regard, it can borrow lessons from the US’s effort to eliminate measles
9. Editorial: No light at the end of the Brexit deadlock
Syllabus: GS2: International news
Context: Ever since the Brexit referendum; the UK society has been witnessing turmoil
Highlights
1. What is Brexit?
 It represents Britain’s split from the European Union after a referendum was conducted on whether to
stay or leave from the block in 2016
 Eventually, people voted to leave the EU by a slender margin vote difference in this referendum
 Some of the issues which dominated the campaign to leave the EU were migration, refugee crisis,
monetary policies of the EU etc

La Excellence IAS 113 Website: www.laex.in/testprep


9052 29 29 29 / 9052 49 29 29 (Hyderabad)
9121 41 29 29 / 9121 44 29 29 (Bangalore)
https://t.me/pdf4exams https://t.me/allupscmaterials

2. Ever since the referendum result, the UK government has been unable to negotiate a proper deal with the
European Union which is acceptable to all
3. This inability to negotiate a deal has caused political turmoil in UK in recent times. Also, it has caused
divisions in the society.
4. What are the prevalent opinions in the UK society on Britain’s issue now?
 There are still many in UK who want a second referendum to be conducted on Brexit issue
 while some prefer a deal which would safeguard the interests of the UK when it leaves the European
Union
 while the UK Prime minister prefers to leave the European Union with or without the deal at the
earliest
5. Impact of UK Prime Minister’s position: Leaving EU without a deal or with a poorly negotiated deal would
jeopardize the economic future of the UK according to many reports published
6. UK Prime Minister Boris Johnson has called for a general election to shore up support for his position
7. Other main concern with respect to Brexit
 Brexit has created deep division in the UK society. Example: Brexit has reignited Scottish nationalism
which is demanding another independence referendum
Takeaway point: There has to be an effort by the political parties to jointly come up with a plan to face this
deadlock rather than fuel more divisions in the society.
10. Editorial: The broken promise of decent and fair wages
Syllabus: GS2: Government acts and policies
Context: The draft rules that the government is proposing to the labour Code on Wages Act 2019 will hamper
the welfare of workers in India
Highlights
1. About Code on Wages act, 2019
 It is an Act of the Parliament of India that consolidates the provisions of four labour laws concerning
wage and bonus payments and makes universal the provisions for minimum wages and timely
payment of wages for all workers in India.
2. Significance of this act:
 The passage of this act was of particular significance to lives of workers in the informal sector who
make up 93% of the total working population and contribute to over 60% of India’s GDP
 The law proposes to increase their income capacity and the purchasing power of the informal
workers.
3. Some economists had criticized the bill because they felt that the increase in wages would further hamper
the revival of economic slowdown. However, the bill passed earlier had created enthusiasm among minds
of many informal workers in India since there was expectation that their wages would increase
4. The centre proposed to adopt the framework of ‘Floor wages’ to determine the new wages
5. What are floor wages? According to the Code, the central government will fix a floor wage, taking into
account living standards of workers. The minimum wages decided by the central or state governments
must be higher than the floor wage
6. Concerns raised against proposed draft rules to the bill:
 A detailed study of the proposed draft rules to the bill shows that the government’s concept of floor
wages will not significantly increase the income of informal workers in the society. Thereby, by
defeating the whole purpose of the bill.
 The floor wage concept is against the constitutional right of citizens in India as well as the direction
given by the Supreme Court of India in ensuring a ‘Need-Based Minimum Wage’ (which covers
nutrition, health care, education, housing and provisions for old age as well).
La Excellence IAS 114 Website: www.laex.in/testprep
9052 29 29 29 / 9052 49 29 29 (Hyderabad)
9121 41 29 29 / 9121 44 29 29 (Bangalore)
https://t.me/pdf4exams https://t.me/allupscmaterials

7. What will be the impact of this proposed draft rules to the code?
 There will be a continued exploitation of workers in India. The proposed draft rules would not prevent
the workers in India from working below the minimum wages
8. What needs to be done?
 A rethink on the draft proposals are required.
 Centre and other states could follow the example of governments of Delhi and Kerala in fixing the
basic wages required to lead a dignified life.
Takeaway point: Ensuring the well-being of workers in India is a fundamental duty on the part of the
government. In this regard, it would be better if the government rethinks the draft rules proposed for the
code.
11. Editorial: Hong Kong’s protest vote
Syllabus: GS2: International news.
Context: The recent polls conducted in the Hong Kong show that the general will of the people and their
preference for democratic reforms in the country.
Highlights
1. In the recently concluded elections to the district council of Hong Kong, candidates from establishment
party (pro-china party) suffered a huge setback
2. This has been seen as an outright rejection of the handling of the Hong Kong protest by the present
administration
3. Hong Kong- China relationship: Until 1997, Hong Kong was ruled by Britain as a colony but then returned
to China. Under the "one country, two systems" arrangement, it has more autonomy than the mainland,
and its people more rights
4. Why the protests start? Hong Kong's protests started in June against plans to allow extradition to
mainland China. This bill was targeted since many believed it was enacted to target the dissidents who
hold contrary opinions to that of the mainland China administration
5. The protests that started with the demanding of the scrapping of the bill but later turned violent. The
protesters also called for more democratic reforms. The administration did not pay heed to such calls.
6. What needs to be done now? The administration should respect this mandate. Measures to address the
issues raised by the protesters should be also given priority.
Takeaway point: Both the administration and protesters should now seek common ground to end violence
and restore order in the city.
12. Editorial: Not as you say, but as you do
Syllabus: GS2: India-Sri Lanka bilateral relations
Context: The Foreign Minister of India paid an official visit to Sri Lanka after Gotabaya Rajapaksa was elected
as the new President
Highlights
1. This visit by the Foreign minister is in line with the ‘Neighborhood first’ policy of the present government.
2. What is ‘Neighborhood first’ policy?
 This policy accords primacy to nations in India’s periphery has always been the priority of Indian
foreign policymakers.
 The policy also includes in its ambit the BIMSTEC—a grouping that includes almost all countries of
South Asia and some in South-East Asia.
3. However, there might be problems for India in engaging with new regime in Sri Lanka due to some
concerns. Such as:

La Excellence IAS 115 Website: www.laex.in/testprep


9052 29 29 29 / 9052 49 29 29 (Hyderabad)
9121 41 29 29 / 9121 44 29 29 (Bangalore)
https://t.me/pdf4exams https://t.me/allupscmaterials

1) The past: India shares a bitter history with Rajapaksa regime. (Sri Lanka-China ties that caused
strategic concerns for India, treatment of Tamils in North Sri Lanka)
2) The ever-growing delay in project completion: India has significantly invested in the infrastructure
projects in Sri Lanka and has plans to invest more. However, the current ongoing projects initiated by
India are suffering from cost-overruns and undue delay
4. What should be done to ensure smooth relationship between the two countries?
 Frequent diplomatic visits between the two countries might help in minimizing the concerns between
the two countries.
 Increased intelligence sharing between the two countries can improve the strategic trust between the
two countries
 India should explore taking new projects in Sri Lanka. But, at the same time, emphasis should be given
to completing the ongoing projects in Sri Lanka (Trincomalee port and other such projects) in a timely
manner.
 India-Japan need to fast track and explore infrastructure options to counter Chinese investments in
the country
 India should also be wary that some of its internal issues do not affect the foreign policy initiatives
with other countries. Example: India’s position on Article 370, mob lynching, National Register of
Citizens (NRC), rights of Tamils in Sri Lanka etc
Takeaway point: India can lead the region much more powerfully if it upholds high ideals of democracy itself
in the region.
13. Editorial: India’s food basket must be enlarged.
Syllabus: GS2: Issues related to Hunger and poverty
Context: Increasing the size of India’s food basket might help in improving the hunger situation in India
Highlights
1. About Global Hunger Index (GHI)
 The Global Hunger Index (GHI) is a tool that measures and tracks hunger globally, by region, and by
country. The GHI is calculated annually, and its results appear in a report issued in October each year
 Created in 2006, the GHI was initially published by the US based International Food Policy Research
Institute (IFPRI) and Germany based Welthungerhilfe. In 2007, the Irish NGO Concern Worldwide also
became a co-publisher.
 In 2018, IFPRI stepped aside from its involvement in the project and the GHI became a joint project of
Welthungerhilfe and Concern Worldwide
2. How is Hunger defined in this index? Hunger is defined by caloric deprivation; protein hunger; hidden
hunger by deficiency of micronutrients.
3. India’s rank in this index:
 India is ranked 102 in the Global Hunger Index (GHI) out of 117 qualified countries.
 Nearly 47 million or four out of 10 children in India do not meet their potential because of chronic
under-nutrition or stunting.
4. Consequence of such a situation:
 This situation leads to diminished learning capacity, increased chronic diseases, and low birth-weight
infants from malnourished parents.
 The global nutrition report pegs 614 million women and more than half the women in India aged 15-
49 as being anemic.
5. How is India trying to address this situation? Apart from various programmes such as National Food
security act, food fortification etc. Some other innovative programmes have also been instituted like:

La Excellence IAS 116 Website: www.laex.in/testprep


9052 29 29 29 / 9052 49 29 29 (Hyderabad)
9121 41 29 29 / 9121 44 29 29 (Bangalore)
https://t.me/pdf4exams https://t.me/allupscmaterials

1. ‘Nutrition garden’: Instituted by Ministry of Human resources development. The programme aims to
identify fruits and vegetables best suited to topography, soil and climate. This exercise apart from
imparting various skills will also be useful in fighting micro-nutrient deficiency when these are fruits
are used in the noon meal
6. Agro-biodiversity and its role in fighting Hunger: Agro-biodiversity helps nutrition-sensitive farming and
bio-fortified foods. Example: Moringa (drumstick) has micro nutrients and sweet potato is rich in Vitamin
A. There are varieties of pearl millet and sorghum rich in iron and zinc.
7. However, in recent times, the impact of agro-biodiversity on human diet is reducing. Today, only 30 crops
form the basis of the world’s agriculture and just three species of maize, rice and wheat supply more than
half the world’s daily calories. This will impact our fight on hunger
8. What are some of the reasons for the loss in agro-biodiversity?
 Adoption of new crop varieties without conserving traditional varieties
 Unsustainable growth in consumption patter in present society
 High output breeds for production of meat, milk and egg which are affecting agro-biodiversity
9. Measures taken on the global stage to prevent agro-biodiversity loss
 Globally Important Agricultural Heritage Systems (GIAHS) has been instituted to ensure we preserve
natural systems of ensuring agro-biodiversity. (There are 37 such identified sites across the world. Of
which, three are in India- Kashmir, Koraput and Kuttanad)
 The UN sustainable development goals (SDG) also advocates for Zero Hunger. It emphasizes that
countries develop strategies and action plans to halt biodiversity loss and reduce direct pressure on
biodiversity.
 The Aichi Biodiversity Target focuses on countries conserving genetic diversity of plants, farm livestock
and wild relatives.
10. What are the steps taken by India in this regard? National Biodiversity Authority has come up with
recommendation to preserve agro-biodiversity in India. Some of its recommendations are:
1. Policy on ‘ecological agriculture’ to enhance native pest and pollinator population providing
ecosystem services for the agricultural landscape
2. Promotion of bio-village concept of MS Swaminathan which will help in conserving ecologically
sensitive farming; conserving crop wild relatives of various important crops
3. Providing incentives for farmers cultivating native crop varieties. Also conserving indigenous breeds of
livestock and poultry varieties.
4. encouraging community seed banks to preserve native seeds
5. preparing an agro-biodiversity index
6. documenting traditional practices through People’s Biodiversity Registers
7. Identifying Biodiversity Heritage Sites under provisions of the Biological Diversity Act, 2002
8. Strengthening Biodiversity Management Committees to conserve agro-biodiversity and traditional
knowledge
9. Proposal for national level invasive alien species policy

La Excellence IAS 117 Website: www.laex.in/testprep


9052 29 29 29 / 9052 49 29 29 (Hyderabad)
9121 41 29 29 / 9121 44 29 29 (Bangalore)
https://t.me/pdf4exams https://t.me/allupscmaterials

Takeaway point: Agricultural policies and hunger prevention programmes should take into account of
biodiversity goals while formulating programmes so that India’s food and nutrition security are enhanced and
also to minimize genetic erosion.
14. Editorial: Getting organ donation to tick again
Syllabus: GS2: Issues related to Health
Context: India has among the lowest rates of organ donation in the world at 0.5 donor per million population,
which is far less compared to over 30 donors per million in some western countries
Highlights
1. What is organ donation and why is it important? Donation is the giving of an organ and tissue to help
someone that needs a transplant. Transplants can save or transform the life of a person. One organ and
tissue donor can help transform the lives of more than 10 people.
2. Current situation with respect to organ donation in India: India still remains a country with one of the
lowest organ donation rates in the world.
3. The Indian government has been encouraging for more organ donation in the country. In this regard,
Indian Organ Donation Day is observed by the Government of India on November 30 to create awareness
about the importance of organ donation. Apart from this, the government has also instituted awards for
donor families
4. However, as explained above the quantum of organ donation is still less compared to other countries.
Some of the reasons for this are:
 There is a perception that private hospitals are declaring a person brain dead in an unscrupulous
manner so that they can benefit from the organ harvesting. Such perception resulted in many donor
families to opt out of the donation.
 The growing trust gap between the private healthcare facilities and the patients is also hampering the
organ donation process in the country
5. Also, the cost of organ transplant in India is high. This high cost is the key reason why more than three
quarters of donated hearts and lungs do not get taken. This situation undermines the altruistic sacrifices
made by the donors and their families
6. Public hospitals have been suggested as a solution to address the before mentioned concerns. However,
there are some concerns associated with this solution. Such as:
 Public hospitals lack the necessary infrastructure to carry out the transplants
 Financial allocation made in this regard by the government is also very negligible
7. What should be done then? Instead of public hospitals as the solution for the organ donation rate in
India. Other measures could be sought out for. Such as:
 Allocation should be made on organ failure prevention rather than on its prevention. According to
WHO report this is more viable for government hospitals and also for the general welfare of the public
 Reduction in bureaucratic approach in organ donation procedure should be bought in. Transplant
approval by self-declaration and mandatory sample verification involving civil society will improve
compliance can help in this regard
 Autonomy for states is needed to ensure it takes substantive steps to encourage more organ
donations in India
La Excellence IAS 118 Website: www.laex.in/testprep
9052 29 29 29 / 9052 49 29 29 (Hyderabad)
9121 41 29 29 / 9121 44 29 29 (Bangalore)
https://t.me/pdf4exams https://t.me/allupscmaterials

 online organ distribution norms and the full details on every organ donation will help build public
confidence in the system
 measures can be taken to ensure that certain percentage of organ donation go to those who cannot
afford organ transplant free of cost
Takeaway point: Organ donation gives a new lease of life by using organs gifted by others who have lost
their lives. It would be prudent for the government and private hospitals to address the concerns that is
preventing organ donation numbers from growing in India
15. Editorial: Cleaning up the Kashmir issue.
Syllabus: GS1: Post-independent India.
Context: Nehru’s policies on Kashmir have come under criticism in recent times.
Highlights
1. Nehru’s policies such as agreeing to separate constitution for J&K and even approaching United Nations
against Pakistani aggression in the valley have been believed by most to contribute the present crisis in
the valley
2. Analysis of Nehru’s action in this regard:
 Nehru was sensitive to the global opinions and obligations on many issues. His approach to J&K was
clearly dictated by such an understanding
 Nehru held the belief that Article 370 would gradually hollow out, creating conditions for J&K’s
complete integration into India.
3. Real reason for the crisis in the valley then: Since 1990s, the decision of the New Delhi are reasons for the
crisis rather than Nehru. Example: implementation of AFSPA act, failure to implement interlocutor report
etc
4. Abrogation of Article 370 might further add to the crisis in the region.
5. Furthermore this move might impact the feelings of Indian Muslims towards the central government and
the country
Takeaway point: Integration of Kashmir which was envisioned by Nehru was more broad-based. It aimed to
achieve more than just political integration. Though Article 370 has achieved political integration; the question
still remains if the people of the valley feel positively towards India.

La Excellence IAS 119 Website: www.laex.in/testprep


9052 29 29 29 / 9052 49 29 29 (Hyderabad)
9121 41 29 29 / 9121 44 29 29 (Bangalore)
5. In the context of single use plastics, consider the
Multiple Choice Questions following statements.
1. Consider the following statements: 1. In India, Single use plastics are categorised into
3 types at national level.
1. Arabian Sea is the zone of frequent cyclones
2. India has determined to eliminate single use
compared to Bay of Bengal.
plastics by 2022.
2. Kyarr is the recent super cyclone storm in
Which of the above statements is/are correct?
Arabian sea
(a) 1 only (b) 2 only
Select the correct statements using the code
(c) Both 1 and 2 (d) Neither 1 or 2
provided below:
(a) 1 only (b) 2 only 6. What does the presence of Aflatoxins in the milk
(c) Both 1 and 2 (d) Neither 1 nor 2 being given to a child indicate?
(a) Presence of Fungi in the cattle feed.
2. Which of the following statements is/are correct (b) Bio Accumulation
about Quantum Computing? (c) Bio Magnification
1. It uses the data fragments in both 1 and 0 (d) Child is prone to develop Cancers.
Select the correct answer using the code provided
qubits together.
below:
2. It is equipped with Super positioning
(a) 1, 2, and 3 only
technology.
(b) 2, 3, and 4 only
3. India has no quantum computer yet.
(c) 1 and 4 only
Select the correct answer using the code provided
(d) 1,2,3 and 4
below:
(a) 1 and 2 only (b) 2 and 3 only 7. Which of the following statements is/are correct
(c) 1 and 3 only (d) 1, 2 and 3 regarding “Thirty-meter telescope”?
1. It is one of the extremely large telescopes
3. With reference to the “Indigen” programme,
placed on longest mountain on the earth -
identify correct statements.
Mauna Kea
1. It aims to collect and sequence the genes of all
2. It aims to observe mid-Infra-red waves and near
Indian citizens.
UV rays.
2. It is an Indian counterpart of International
3. It is sharper than Hubble telescope
human genome project. Select the correct answer using the code provided
Select the correct answer using the code provided
below:
below:
(a) 1 and 2 only (b) 2 and 3 only
(a) 1 only (b) 2 only
(c) 1 and 3 only (d) 1, 2 and 3.
(c) Both 1 and 2 (d) Neither 1 or 2
8. Consider the following statements about
4. Consider the following statements about “Snow
Nitrogen:
Leopard”:
1. It is a quantitative pollutant.
1. It is the flagship specie of high altitude
2. It turns as a carcinogen, when it enters the food
Himalayas.
chain
2. It is categorized as vulnerable under IUCN red 3. To half the nitrogen waste levels by 2030,
list. Colombo declaration is adopted.
Select the correct statements using the code
Select the correct statements using the code
provided below:
provided below:
(a) 1 only (b) 2 only
(a) 1 and 2 only (b) 1 and 3 only
(c) Both 1 and 2 (d) Neither 1 nor 2 (c) 2 and 3 only (d) 1, 2 and 3
La Excellence IAS 120 Website: www.laex.in/testprep
9052 29 29 29 / 9052 49 29 29 (Hyderabad)
9121 41 29 29 / 9121 44 29 29 (Bangalore)
9. Which of the following statements are correct 13. Consider the following statements.
about “East Antarctica Protected area”? 1. Kyoto protocol is binding in implementing its
1. It aims at protecting marine plants and animals. goals and India ratified it.
2. It is the first ever sanctuary for Antarctic region 2. The Paris agreement will come into force in
being proposed. 2020, when the Kyoto Protocol comes to an
3. It is proposed by France and Australia. end.
Select the correct answer using the code provided Select the correct answer using the code given
below:
below:
(a) 1 only (b) 2 only
(a) 1 and 2 only (b) 2 and 3 only
(c) Both 1 and 2 (d) Neither 1 nor 2
(c) 1 and 3 only (d) 1,2 and 3
14. Which of the following is incorrect regarding
10. Consider the following statements about Rajgir: “Hunar Haat”?
1. It was the ancient capital of Magadha. (a) It is organized under “Upgrading the Skills and
2. First Buddhist Council was held here. Training in Traditional Arts/ Crafts for
3. World’s highest peace pagoda exists here. Development” (USTTAD) scheme.
Select the correct statements using the code (b) Ministry of Tribal Affairs is the nodal agency for
provided below: the organization of this event.
(a) 1 and 2 only (b) 2 and 3 only (c) It is an all India programme.
(c) 1 and 3 only (d) 1,2 and 3 (d) It fulfills the commitment of “Startup India” and
11. Which of the following statements is/are correct? “Standup India”.
1. “Petroleum and Explosive Safety Organization 15. Which of the following statements is/are correct
(PESO)” gives licenses for manufacturing Green regarding “Ozone Hole”?
crackers. 1. It is the destruction of ozone layer in the
2. Barium chemical is the primary fuel in fire troposphere layer by free radicals like chlorine
crackers. and bromine.
3. “Bureau of Energy Efficiency” developed Green 2. Kyoto protocol was adopted to tackle this
crackers such as ‘SAFAL’ and ‘SWAS’. phenomenon.
Select the correct answer using the code below: 3. Recently, It had shrinked to the smallest since
(a) 1 only (b) 2 and 3 only its discovery over Antarctica.
(c) 1 and 2 only (d) 1 and 3 only Select the correct answer using the code below:
(a) 1 only (b) 1 and 2 only
12. Which of the following best describes the
(c) 2 and 3 only (d) 3 only
mechanism “Common but Differentiated
Responsibilities”? 16. Which of the following countries share border
with Syria?
(a) It assists the UN member nations to bring
1. Turkey
reforms in Security Council for addressing
2. Israel
terrorism.
3. Iran
(b) It acknowledges the differing responsibilities of
4. Jordan
individual countries in addressing climate
5. Armenia
change.
Select the correct answer using the code below:
(c) It considers the capacities of different nations
(a) 1, 2 and 3 only (b) 1, 3 and 4 only
while setting goals for combating space junk.
(c) 1, 2 and 4 only (d) 1, 3 and 5 only
(d) None of the above.
La Excellence IAS 121 Website: www.laex.in/testprep
9052 29 29 29 / 9052 49 29 29 (Hyderabad)
9121 41 29 29 / 9121 44 29 29 (Bangalore)
17. Which of the following statements is/are correct 22. With reference to a grouping of countries known
regarding “Organoids”? as Non-Aligned Movement (NAM), consider the
1. These are the structures that possess all the following statements
functional maturity of human organs. 1. The NAM summit was recently held in Baku,
2. They lack blood vessels. Azerbaijan.
3. Organoids assists in identifying the causes of
2. No country of European Union (EU) is a
mutations in a gene that causes a disease.
member of NAM
Select the correct answer using the code below:
Which of the statement given above is/are
(a) 1 only (b) 1 and 2 only
correct?
(c) 2 and 3 only (d) 3 only
(a) 1 only (b) 2 only
18. In which of the following sectors one can find the (c) Both 1 and 2 (d) Neither 1 nor 2
use of the technology called “Electronic
interlocking system”? 23. Consider the following statements with respect to
(a) Construction of highways Intensified Mission Indradhanush (IMI) 2.0
(b) Submarine designing 1. Vaccine for Malaria and Dengue are included
(c) Electronic chip printing recently.
(d) Signal system in railways 2. It is launched by Ministry of Health and Family
19. Which of the following is correct regarding Welfare.
“Thotlakonda Monastery” seen in news? 3. IMI 2.0 aims to cover low immunization regions.
1. It has remnants which are related to Buddhism. Which of the statements given above are correct?
2. It is located in the state of Arunachal Pradesh. (a) 1 and 2 only (b) 2 and 3 only
Select the correct answer using the code given (c) 1 and 3 only (d) All the above
below:
(a) 1 only (b) 2 only 24. With respect to World Bank’s Ease of Doing
(c) Both 1 and 2 (d) Neither 1 nor 2 business report of 2019, consider the following
statements
20. Consider the following statements regarding
1. India’s rankings improved continuously in the
“Global Wealth report”-2019.
last 3 years.
1. Credit Suisse Research Institute releases the
report. 2. It takes reform measures into consideration
2. According to it, India has most people in the top from Delhi and Mumbai only
10% of global wealth distribution. 3. The report places India ahead of china in the
3. India was among the regions with the highest ease of doing business for the first time.
percentage change in wealth per adult. Select the correct statements using the code given
Select the correct answer using the code below: below:
(a) 1 and 2 only (b) 2 only (a) 1 and 2 only (b) 2 and 3 only
(c) 3 only (d) 1 and 3 only
(c) 1 and 3 only (d) All the above
21. With reference to Bharat Interface for Money 25. Consider the following matches:
(BHIM) 2.0, consider the following features Pilgrimages Rivers
1. It facilitates entry to Capital Market
1. Kartarpur Corridor Ravi
2. It supports vernacular languages other than
English and Hindi 2. Kailash Manasarovar Sutlej
3. Facilitate Foreign currency transactions 3. Gangotri Ganga
Which of the above statements are correct? Which of the above matches is/are correctly
(a) 1 and 2 only (b) 2 and 3 only matched?
(c) 1 and 3 only (d) All the above (a) 3 only (b) 2 and 3 only
(c) 1 and 3 only (d) All the above

La Excellence IAS 122 Website: www.laex.in/testprep


9052 29 29 29 / 9052 49 29 29 (Hyderabad)
9121 41 29 29 / 9121 44 29 29 (Bangalore)
26. With reference to “National Institute of Ocean 30. Consider the following statements.
Technology”, consider the following statements. 1. Both Iran and Caspian Sea do not share
1. It is an autonomous body established under boundary with this nation.
2. It is a member of Shanghai Cooperation
Ministry of Science and Technology.
Organization (SCO).
2. Tsunami Early Warning System has been
3. India conducts exercise “Dustlik” with this
established under it in the wake of 2004
nation.
tsunami in the Indian Ocean. Which of the following Central-Asian country is
3. It will develop technologies to solve described in the above statements?
engineering problems associated with (a) Turkmenistan (b) Tajakistan
harvesting the resources in the Indian Exclusive (c) Kazakistan (d) Uzbekistan
Economic Zone. 31. Which of the following statements is/are correct
Which of the above statements is/are correct? with regard to “National Health Profile”?
(a) 1 and 2 only (b) 2 only 1. “Central Bureau of Health Intelligence” releases
(c) 2 and 3 only (d) 3 only the report annually.
2. There has been a consistent decrease in the
27. The Agreement on Reciprocal Logistics Support
birth rate, death rate and natural growth rate in
(ARLS) is an agreement between India and which India since 1991 to 2017.
of the following country? 3. Communicable diseases dominate over non-
(a) United States of America communicable in the total disease burden of
(b) United Kingdom the country.
(c) Russia Select the correct answer using the code given
(d) Japan below:
(a) 1 only (b) 1 and 2 only
28. Which of the following cities in India are present (c) 2 and 3 only (d) 3 only
in “UNESCO Creative Cities Network” (UCCN)?
32. Which of the following is most probable incorrect
1. Mumbai statement with regard to ‘Sugar Industry’ in
2. Chennai India?
3. Tiruvanantapuram (a) At present, there is no need to take a separate
4. Kolkata environment clearance to produce additional
5. Hyderabad ethanol from ‘B-heavy molasses’.
Select the correct answer using the code below: (b) Fair and Remunerative is the minimum
(a) 1 and 2 only (b) 2, 3 and 5 only benchmark price below which government
(c) 1, 2 and 5 only (d) 1, 2, 3, 4 and 5 cannot purchase cane from the farmers.
(c) To enhance its production government has
29. ‘Oligomannate’ is seen in the news. It is best been implementing Sugarcane Development
related to Programme under National Food Security
(a) A newly discovered exoplanet by NASA Mission - Commercial Crops
(b) A newly approved drug for Alzheimer disease (d) Making ethanol form B-heavy molasses can
(c) An invasive species found in the Western-Ghats bring stable sugar prices in the market.
(d) An organism which relies on Chemosynthesis
for the synthesis of organic compounds.

La Excellence IAS 123 Website: www.laex.in/testprep


9052 29 29 29 / 9052 49 29 29 (Hyderabad)
9121 41 29 29 / 9121 44 29 29 (Bangalore)
33. In the context of ‘Serious Fraud Investigation 37. With reference to the term “Predatory Pricing”,
Office’, consider the following statements. choose the best option given below.
1. It is under “Ministry of Corporate Affairs”. (a) Selling commodities in X country’s market by Y
2. It may take up cases on its own and has the country at prices below which is sold in Y
power to arrest people for violation of country
Company law. (b) This term is generally associated with abuse of
Select the correct answer using the code given dominant position by a market player
below: (c) type of pricing used by companies that have a
(a) 1 only (b) 2 only significant competitive advantage and which
(c) Both 1 and 2 (d) Neither 1 nor 2 can gain maximum revenue advantage before
34. “Global Microscope on Financial Inclusion Report” other competitors begin offering similar
is release by products
(a) Financial Stability Board (d) Setting a price based on what competitor
(b) The Economist Intelligence Unit charges
(c) World Bank
38. Consider the following statements about
(d) International Monetary Fund ‘Environment Pollution (Prevention and Control)
35. With reference to National waterways (NW) in Authority (EPCA)’
India, consider the following matches. 1. It was constituted with the objective of
Name of the waterway Route ‘protecting and improving’ the quality of the
environment and ‘controlling environmental
A NW-4 1. Sadiya to Dhubri
pollution’ in NCT of Delhi only
B NW-1 2. Allahabad to Haldia 2. It was constituted by the National Green
C NW-3 3. Kakinada to Puducherry Tribunal
3. It is mandated to stop stubble burning in
D NW-2 4. Kollam to Kottapuram
Which of the above statements is/are correct? Punjab and Haryana
Which of the statements given above are correct?
(a) A-3, B-2, C-4, D-1
(a) 1 and 2 only (b) 1 only
(b) A-2, B-3, C-4, D-1
(c) 2 and 3 only (d) 2 only
(c) A-3, B-2, C-1, D-4
(d) A-2, B-3, C-1, D-4 39. With reference to the recent news item “Indi
Gen” which one of the following statements is
36. Consider the following statements regarding Bio- incorrect?
Plastics.
(a) This initiative was undertaken by CSIR
1. Bioplastics has no negative environmental
(b) IndiGen programme aims to undertake whole
impacts
genome sequencing of thousands of individuals,
2. Bioplastics are made from agricultural by-
animals of India
products and also from used plastic bottles
(c) Applications of this project include predictive
3. All Bioplastics are soil and marine-safe and they
and preventive medicine with faster and
safely degrade in the environment
efficient diagnosis of rare genetic diseases.
Which of the above statements are incorrect?
(d) The objective is to enable genetic epidemiology
(a) 1 and 2 only (b) 2 and 3 only
and develop public health technologies
(c) 1 and 3 only (d) All the above
applications using population genome data.

La Excellence IAS 124 Website: www.laex.in/testprep


9052 29 29 29 / 9052 49 29 29 (Hyderabad)
9121 41 29 29 / 9121 44 29 29 (Bangalore)
40. With reference to the Indian Regional Navigation 45. With reference to Ethanol, consider the following
Satellite System (IRNSS), consider the following 1. The new Bio fuel Policy 2018 has fixed a target
statements: of achieving 50% ethanol blending with petrol
1. IRNSS has 4 satellites in geostationary and 3 by 2030.
satellites the geosynchronous orbits. 2. Ethanol Blended Petrol releases fewer
2. IRNSS covers entire India and about 15000 km
emissions into the environment and is
beyond its borders.
considered cleaner in nature.
3. India is the 2nd nation in Asia to have their own
Which of the statement given above is/are
navigation system
correct?
Which of the statements given above is/are
(a) 1 only (b) 2 only
correct?
(a) 1 only (b) 1 and 2 only (c) Both 1 and 2 (d) Neither 1 nor 2
(c) 2 and 3 only (d) None of the above 46. Consider the following statements:
41. The terms “Pegasus, DTrack” sometimes 1. Lord Wellesley was responsible for the Fourth
mentioned in the news recently are related to Anglo Mysore war and british victory.
(a) Drones 2. Treaty of Madras was signed between Tipu
(b) Cyber attacks Sultan and British.
(c) Mini satellites 3. Tipu Sultan crushed the Hindu coorgs and Nairs.
(d) Black money tracking tools Which of the above statements are correct?
42. Look at the following statements about Program (a) 1 and 2 only (b) 2 and 3 only
for International Student Assessment (PISA) 2021 (c) 1 and 3 only (d) 1,2 and 3
1. UNESCO coordinates PISA initiative 47. With respect to Nuclear Power Corporation of
2. It is an international assessment that measures India Limited (NPCIL), Consider the following
15-year-old students’ reading, mathematics, statements.
and science literacy conducted every 3 years 1. NPCIL is responsible for Uranium mining and
3. India is participating in PISA for the first time processing.
Which of the above statements is/are Correct?
2. NPCIL is administered by the department of
(a) 1 and 2 only (b) 2 only
Atomic Energy.
(c) 3 only (d) 2 and 3 only
3. It constructs and operates India’s commercial
43. Consider the following statements about
nuclear power plants.
Organoids: Which of the above statements are correct?
1. These are 3-dimensional organs developed
(a) 1 and 2 only (b) 2 and 3 only
from 3-D Printing technology
(c) 1 and 3 only (d) 1, 2 and 3
2. They achieve the same functional maturity of
various human organs. 48. Through which of the following Countries does
Which of the above statements is/are correct? Mekong river flows?
(a) 1 only (b) 2 only 1. China
(c) Both 1 and 2 (d) Neither 1 nor 2 2. India
44. With reference to Bru Tribes which are news 3. Myanmar
recently, which of the following states has an 4. Thailand
agreement 5. Cambodia
(a) Mizoram and Manipur Select the correct answer from code given below:
(b) Manipur and Nagaland (a) 1, 2 and 3 only (b) 2, 3 and 4 only
(c) Tripura and Mizoram (c) 1, 3, 4 and 5 only (d) 1, 2, 3, 4 and 5
(d) Arunachal Pradesh and Assam
La Excellence IAS 125 Website: www.laex.in/testprep
9052 29 29 29 / 9052 49 29 29 (Hyderabad)
9121 41 29 29 / 9121 44 29 29 (Bangalore)
49. With respect to Greater adjutant Stork, consider 53. A polygon has been drawn by Ramya by joining
the following statements. the five capital cities of BRICS nations, Which
1. According IUCN list they were endangered. among the following nations completely lie in this
2. They were now mostly confined to India and
Polygon.
Cambodia.
1. Belarus 2. Kazakhstan
Which of the above statements is/are correct?
3. Algeria 4. Turkey
(a) 1 only (b) 2 only
5. Chad 6. Myanmar
(c) Both 1 and 2 (d) Neither 1 nor 2
Select the correct answer using the code below:
50. With reference to Lieutenant Governor, Which of
(a) 1, 2, 4 and 5 only (b) 1, 3, 5 and 6 only
the following statement is correct?
(c) 4 and 5 only (d) 1, 2, 4, 5 and 6 only
(a) Lieutenant governors hold the same rank as a
governor of a state in the list of precedence. 54. Which of the following is incorrect with regard to
(b) Lieutenant Governors are appointed by the “Green Climate Fund”?
president for a term of 5 years. (a) It will support the efforts of developing
(c) Lieutenant Governor acts as the administrator countries to respond to the challenge of climate
of Chandigarh. change.
(d) Raj Bhavan is the official residences of (b) It was setup during Earth Summit of 1992.
Lieutenant Governors. (c) The fund comes from both developed and
51. With reference to “Maternal Mortality Ratio” in developing countries.
the context of India, consider the following (d) It pays particular attention to the needs of
statements. Least Developed Countries and Small Island
1. It is the proportion of maternal deaths per
Developing States.
10,000 live births.
2. It is declined by more than 50 percent during 55. The question of disqualification under the “Anti-
1990 to 2015. defection Law” under Tenth schedule of Indian
3. It will be released by Ministry of Health and constitution is decided by the
Family Welfare. (a) Election Commission of India
Which of the above statements is/are incorrect? (b) President in case of MPs and Governor in case
(a) 1 and 2 only (b) 2 only of MLAs
(c) 1 and 3 only (d) 3 only (c) Political party of defected member
52. Which of the following correctly describes (d) Presiding officer of the house.
“Project Zero” seen in the news?
56. Arrange the following Inter basin water transfer
(a) Eliminating Particulate matter emissions to
links in North to South direction.
zero-level in the vicinity of National Capital
1. Krishna – Pennar
Territory region.
2. Ken-Betwa
(b) Customers receive authentic goods when they
3. Pamba-Achankovil-Vaippar
shop on Amazon online platform.
4. Damanganga-Pinjal
(c) Constructing a zero gravity camber for
Select the correct answer using the code given
“Gaganyan Project” of ISRO.
below:
(d) Bringing down the number of dengue affected
(a) 2-4-1-3 (b) 2-1-4-3
patients to zero digit.
(c) 2-1-3-4 (d) 2-4-3-1

La Excellence IAS 126 Website: www.laex.in/testprep


9052 29 29 29 / 9052 49 29 29 (Hyderabad)
9121 41 29 29 / 9121 44 29 29 (Bangalore)
57. Recent data from the National Statistics Office 60. With respect to Pardoning powers of President,
showed that industrial production contracted to
Consider the following statements.
the lowest in the current series. Which of the
following sectors are parts of core industries of 1. If a state law prescribes death sentence, the
“Index of Industrial Production”? power to grant pardon lies with Governor and
1. Fertilizers President.
2. Software services
2. When President Decision is arbitrary during
3. Natural Gas
exercising of his pardoning power then it is
4. Telecom
subject to judicial review.
5. Crude oil
Select the correct answer using the code given Which of the above statements is/are correct?
below: (a) 1 only (b) 2 only
(a) 1, 2, 3 and 5 only (b) 2, 3 and 5 only (c) Both 1 and 2 (d) Neither 1 nor 2
(c) 1, 2, 4 and 5 only (d) 1, 3 and 5 only
61. Consider the following with regard to
58. With reference to Chief Justice of India, Consider
the following statements. disqualification under tenth schedule:
1. Chief Justice on advice of President is 1. It bars disqualified members from contesting
responsible for the appointment of re-elections
constitutional benches. 2. A disqualified member can be appointed to a
2. Senior most judge of Supreme Court is
remunerative post except government minister
appointed to the office of the Chief Justice of
till re-elected.
India.
3. It is the speaker’s discretion to disqualify a
3. Constitution has mentioned that CJI holds office
until he attained the age of 65 years. member for a certain period of time.
Which of the above statements is/are correct? Select the incorrect statement(s) using the code
(a) 1 and 2 only (b) 2 and 3 only below:
(c) 2 only (d) 1 and 3 only (a) 1only (b) 2 and 3 only
59. Consider the following statements with respect to (c) 3 only (d) 1, 2 and 3
President’s Rule?
1. During President’s Rule, President acquires the 62. With regard to the concept Negative interest
powers vested in the Governor. rates, which is seen in news sometimes, which of
2. The President can either suspend or dissolves the following is not correct?
the state legislative assembly during President’s (a) These rates are charged on all the borrowings
Rule. retrospectively
3. If approved by both the houses of Parliament,
(b) It helps the economy in avoiding deflation
the President’s Rule continues for one year.
(c) It makes the consumer save less and spend
Identify the correct answer using the code given
below: more
(a) 1 and 2 only (d) Financial institutions will pay interest for their
(b) 2 and 3 only deposits
(c) 1 and 3 only
(d) 1,2 and 3

La Excellence IAS 127 Website: www.laex.in/testprep


9052 29 29 29 / 9052 49 29 29 (Hyderabad)
9121 41 29 29 / 9121 44 29 29 (Bangalore)
63. Consider the following 67. Consider the following mechanisms which
The Steel Scrap Recycling Policy envisages to: deteriorated air quality in Delhi recently?
1. Make India Self-sufficient in scrap availability 1. Stubble burning
2. Conserve vital natural resources 2. Air stagnation
3. Structure the informal recycling sector 3. Failure of electro static precipitators in Thermal
4. Devise a method to treat waste streams power plants
Select the correct answer using the given code Select the correct answer using the codes given
below: below
(a) 1 and 2 only (b) 3 only (a) 1 and 2 only (b) 2 and 3 only
(c) 2 and 4 only (d) 1, 2, 3 and 4 (c) 1 and 3 only (d) 1, 2 and 3

64. Which of the following is related to India Skills 68. SAANS Campaign which was in news recently is
2020? related to
(a) Initiative to train Indian graduates in Artificial (a) Air pollution awareness campaign in Delhi
Intelligence (AI) (b) To reduce child mortality due to pneumonia
(b) A platform for skilled and talented Indian (c) Campaign launched by Telecom companies
youngsters to showcase their abilities at against Adjusted Gross revenue Supreme court
regional and national level competitions judgement
(c) A platform to explore the talent of outshining (d) Cancer awareness campaign
and meritorious students by enriching their
69. Consider the following statements regarding
skills and knowledge under expert’s guidance
International Treaty of Plant Genetic Resources
(d) Flagship initiative by NITI Aayog to promote
for Food and Agriculture (ITPGRFA).
innovation and entrepreneurship across the
1. It is a comprehensive international agreement
country.
for ensuring food security through sustainable
65. Which of the following states is going to get the use of the world’s Plant Genetic Resources for
World’s first Compressed Natural Gas (CNG) port Food and Agriculture.
terminal? 2. It aims to ensure that recipients share benefits
(a) Telangana (b) Delhi they derive from the use of the plant genetic
(c) Maharashtra (d) Gujarat materials with the countries where they have
been originated.
66. The term “Spaghetti bowl phenomenon”
3. It is not mandatory to ratify the treaty for
sometimes mentioned in the news recently is
accessing Plant Genetic materials.
related to
Which of the above statements is/are correct?
(a) Capital controls
(a) 1 and 2 only (b) 2 and 3 only
(b) Foreign direct investment
(c) 1 and 3 only (d) All the above
(c) Free trade agreements
(d) Foreign portfolio investments

La Excellence IAS 128 Website: www.laex.in/testprep


9052 29 29 29 / 9052 49 29 29 (Hyderabad)
9121 41 29 29 / 9121 44 29 29 (Bangalore)
70. Which of the following are considered as public 74. Consider the following statements with respect to
authorities under RTI ‘Jal Jeevan Mission’
1. President 1. It is under Ministry of Rural development
2. Prime minister 2. It aimed at piped water supply to all rural
3. Chief Justice of India households by 2024.
4. Board of Cricket for control in India 3. It will focus on the creation of local
5. Central Bureau of Investigation infrastructure for water sustainability.
Choose the correct answer using the code given Select the correct statements
below (a) 1 and 2 Only (b) 2 and 3 only
(a) 1, 2, 3, 4 and 5 (b) 1, 2, 3 and 4 (c) 1 and 3 only (d) 1,2 and 3
(c) 1, 2 and 3 (d) 3 and 5 only
75. Which of the following indexes are released by
71. Consider the following options regarding orphan Central Statistical Organisation (CSO)?
drugs and select the correct one? 1. Consumer Price Index (Industrial workers) (CPI)
(a) They are drugs meant for the disadvantageous 2. Wholesale Price Index (WPI)
sections of the population I.e., orphans 3. Index of Industrial Production (IIP)
(b) Low cost drugs provided under Jan Aushadi 4. Consumer price Index (Rural)
scheme Select the correct answer using the codes given
(c) It is a drug needed for treatment of a rare below.
disease (a) 1, 2, 3 and 4 (b) 2,3 and 4 only
(d) A rarely used drug in the medical field-due to its (c) 1 and 2 only (d) 3 and 4 only
low therapeutic efficiency
76. Tally Valley wildlife Sanctuary is located in which
72. Consider the following Books/ organisations state?
regarding Lala Lajapat Rai (a) Arunachal Pradesh (b) Assam
1. Indian Home Rule League of America (c) Manipur (d) Nagaland
2. Young India
77. Consider the following statements with respect to
3. England’s debt to India
Madden-Julian Oscillation (MJO)?
Which of the above books/ organisations has a
1. It is a moving band of clouds that travels
link with Lala Lajpat Rai?
around the globe across the tropical oceans.
(a) 2 only (b) 1 only
2. It is most prominent over Indian and Pacific
(c) 1 and 3 only (d) All the above
Oceans.
73. Recently, two varieties of tea, Green Tea and 3. Change in the residence time of MJO clouds has
White Tea received GI Tags. It belongs to which of altered the weather patterns across the globe.
the following States? Which of the above statements is/are correct?
(a) Assam (b) Arunachal pradesh (a) 1 and 2 only (b) 2 and 3 only
(c) West Bengal (d) Tamilnadu (c) 1 and 3 only (d) 1, 2 and 3

La Excellence IAS 129 Website: www.laex.in/testprep


9052 29 29 29 / 9052 49 29 29 (Hyderabad)
9121 41 29 29 / 9121 44 29 29 (Bangalore)
78. With reference to the Pink Ball Cricket, consider 82. Tenth schedule of the Indian constitution was in
the following statements: the news recently. Consider the following
1. A red ball is stitched with a white thread while statements in this regard.
the pink ball is stitched with a black thread. 1. A candidate voluntarily gives up the
2. Wax is used on the red ball which gives the red membership of his political party.
ball its cherry colour.
2. An independent candidate joins a political party
Which of the statements given above is/are
after the election.
correct?
3. A nominated member joins a party six months
(a) 1 only (b) 2 only
after he becomes a member of the legislature.
(c) Both 1 and 2 (d) Neither 1 nor 2
In which of the above scenarios the candidates
79. What is the meaning of a Latin expression ‘Quid will be disqualified?
pro quo’, recently seen in news?
(a) 1 and 2 only (b) 2 and 3 only
(a) Something given or received for something else
(c) 1 and 3 only (d) 1, 2 and 3
(b) A sincere intention to be fair
(c) Seize the day 83. “West Bank” is seen in the news best related to
(d) Work undertaken voluntarily and without (a) India’s research camp in Antarctic ice cover.
payment (b) Israeli’s settlers in a conflict zone of Middle East
80. With reference to the legal framework in India for Asia.
illegal migrants, consider the following (c) Demilitarized zone between North Korea and
statements: South Korea.
1. In the Foreigners Act, 1946, ‘burden of proof’ (d) None of the Above.
lies with the authorities, and not with the
84. The “Jing kiengjri” seen in the news related to:
person.
(a) Tree species used for biodiesel in the state of
2. The Foreigners (Tribunals) Order, 1964
Kerala
empowers district magistrates in all States and
(b) Invasive species in Andaman and Nicobar
Union Territories to set up tribunals to decide
claims of citizenship Islands
Which of the statements given above is/are (c) Living root bridges in Meghalaya
correct? (d) Newly grown vegetation cover in the ‘Thar
(a) 1 only (b) 2 only Desert’ to stop desertification.
(c) Both 1 and 2 (d) Neither 1 nor 2 85. Recently, Indian constitution has completed its
81. “Himayat Mission” is an initiative of? 70th year anniversary. Consider the following
(a) Ministry of Environment, Forests and Climate matches in the context of Indian constitution.
Change to maintain the sustainabilityof Feature Borrowed from
Himalayan ecosystem. 1. Suspension of Fundamental Rights Weimar
(b) Ministry of Rural Development to generate constitution of Germany during emergency
sustainable livelihood opportunities through 2. Procedure established by Law Japanese
self employment for the youth
Constitution
(c) NITI Aayog to provide drinking water to the
3. Concurrent List Australian Constitution
disadvantaged society.
Which of the above matches are correct?
(d) Ministry of Environment, Forests and Climate
(a) 1 and 2 only (b) 2 and 3 only
Change for protecting the habitat of Snow
leopard. (c) 1 and 3 only (d) 1, 2 and 3

La Excellence IAS 130 Website: www.laex.in/testprep


9052 29 29 29 / 9052 49 29 29 (Hyderabad)
9121 41 29 29 / 9121 44 29 29 (Bangalore)
86. The “Bodoland dispute” is majorly related to the 3. Article 280 of the Indian Constitution provides
state of the establishment of the finance commission
(a) Assam (b) Tripura Which of the above statements is/are correct?
(c) Arunachal Pradesh (d) Mizoram (a) 1 and 2 only (b) 2 and 3 only
(c) 1 and 3 only (d) 1, 2 and 3
87. Which of the following statements is/are correct
with regard to “Swacch Bharat Mission (SBM)”? 91. With reference to Intended Nationally
1. Under it, NITI Aayog will certify the local bodies Determined Contributions (INDCs), sometimes
as "Open Defecation Free".
seen in the news, consider the following
2. There is no monetary incentive for the
statements:
construction of individual Household Latrines.
Select the correct answer using the following 1. INDCs are voluntary national climate targets
code: which countries have committed under the
(a) 1 only (b) 2 only Kyoto Protocol.
(c) Both 1 and 2 (d) Neither 1 nor 2 2. Developed country's INDC commitments can be
88. Which of the following statements is not an challenged by developing countries, in UNFCCC,
application with regard to “Cartosat” Series if latter feel the commitments violate Common
satellites?
but Differentiated Responsibility Principle.
(a) civil navigation
Which of the statements given above is/are
(b) infrastructural planning
(c) monitoring road networks correct?
(d) Creation of land use maps (a) 1 only (b) 2 only

89. With reference to “FASTag”, consider the (c) Both 1 and 2 (d) Neither 1 nor 2
following statements. 92. Which of the following options is incorrect with
1. It employs Near Field Communication (NFC)
respect to Golden rice, which is often in news
technology for toll payments on National
recently?
Highways.
2. It is valid for unlimited lifetime after one time (a) It is a genetically modified form of rice
recharge. (b) It counters Vitamin D deficiency
Select the correct answer using the code given (c) It counters Vitamin A deficiency
below: (d) Water efficiency improves with Golden rice
(a) 1 only (b) 2 only
(c) Both 1 and 2 (d) Neither 1 nor 2 93. With reference to Ghadar Party, Consider the
following statements
90. Recently, Cabinet approved the extension of the
term and coverage of the Fifteenth Finance 1. The founding president of Ghadar Party was
Commission. In this context, consider the Udham Singh
following statements. 2. Komagata Maru tragedy was associated with
1. Its recommendations are valid for the period of Ghadar Party
2020-21 to 2024-25. Which of the given above statements is/are
2. The committee will examine the possibility of
correct?
setting up a permanent non lapsable funding
for India’s defence needs. (a) 1 only (b) 2 only
(c) Both 1 and 2 (d) Neither 1 nor 2
La Excellence IAS 131 Website: www.laex.in/testprep
9052 29 29 29 / 9052 49 29 29 (Hyderabad)
9121 41 29 29 / 9121 44 29 29 (Bangalore)
94. One of theisland is in the Sundarban Delta has 98. Due to some reasons, if there is a huge fall in the
been facing the devastating effects of climate population of species of butterflies, what could be
its likely consequence/consequences?
change. With rising sea levels, high tides and
1. Pollination of some plants could be adversely
floods part of the island is losing. In the last two
affected.
decades, the island’s size has reduced by several
2. There could be a drastic increase in the fungal
times.
infections of some cultivated plants.
Choose the correct option about the island
3. It could lead to a fall in the population of some
described above
species of wasps, spiders and birds.
(a) Ghoramara island (b) Bhasan char island
Select the correct answer using the code given
(c) Baratang Island (d) Chatham Island below
95. The term INSTEX which is in news recently, is (a) 1 and 2 only (b) 2 and 3 only
related to: (c) 1 and 3 only (d) 1, 2 and 3
(a) China Russia Gas pipeline 99. Consider the following statements about National
(b) Iran Nuclear Deal register of citizens
(c) Strategy of USA to deal with ISIS terrorism 1. National Register of Citizens was introduced to
(d) Paris climate deal to counter Climate change identify illegal immigrants from Bangladesh and
96. With reference to the Global Migration Report recognize the Indian citizens in Assam
2020, consider the following statements: 2. Assam is the only state having the arrangement
1. China is the largest country of origin of of National Register of Citizens.
international migrants across the world. 3. It was first prepared in 1971, in the wake of
2. The top destination country for international Bangladesh liberation.
Which of the statements given above is/are
migrants is United States.
correct?
3. China is also the highest remittances recipient
(a) 1 only (b) 2 and 3 only
country in the world.
(c) 1 and 2 only (d) 3 only
Which of the statements given above is/are
100. The term Open Acreage Licensing Policy (OALP),
correct?
frequently in news, is related to?
(a) 1 and 2 only (b) 2 and 3 only
(a) Extraction of different types of coal from coal
(c) 2 only (d) 1 only
blocks in India.
97. With reference to Assam Accord, consider the (b) Company exploring hydrocarbons by selecting
following statements the exploration blocks on its own, without
1. The accord provided for expulsion of all waiting for the formal bid round from the
foreigners, who entered Assam after 1985 government.
2. There is a provision in the Accord which (c) Regulation of marine fishing in Exclusive
provides for the protection and promotion of Economic Zone.
Assamese culture (d) Allocation of spectrum for the
Which of the given above statements is/are telecommunication business.
correct?
(a) 1 only (b) 2 only
(c) Both 1 and 2 (d) Neither 1 nor 2

La Excellence IAS 132 Website: www.laex.in/testprep


9052 29 29 29 / 9052 49 29 29 (Hyderabad)
9121 41 29 29 / 9121 44 29 29 (Bangalore)
Keys
Qn. Ans. Qn. Ans. Qn. Ans.
1. (b) 2. (d) 3. (b)
4. (c) 5. (b) 6. (d)
7. (d) 8. (b) 9. (c)
10. (d) 11. (a) 12. (b)
13. (c) 14. (b) 15. (d)
16. (c) 17. (c) 18. (d)
19. (a) 20. (d) 21. (a)
22. (c) 23. (b) 24. (a)
25. (d) 26. (d) 27. (c)
28. (c) 29. (b) 30. (d)
31. (b) 32. (b) 33. (c)
34. (b) 35. (a) 36. (c)
37. (b) 38. (b) 39. (b)
40. (d) 41. (b) 42. (b)
43. (d) 44. (c) 45. (b)
46. (c) 47. (b) 48. (c)
49. (c) 50. (b) 51. (c)
52. (b) 53. (c) 54. (b)
55. (d) 56. (a) 57. (d)
58. (b) 59. (d) 60. (b)
61. (d) 62. (a) 63. (d)
64. (b) 65. (d) 66. (c)
67. (a) 68. (b) 69. (a)
70. (b) 71. (c) 72. (d)
73. (c) 74. (b) 75. (d)
76. (a) 77. (d) 78. (c)
79. (a) 80. (b) 81. (b)
82. (d) 83. (b) 84. (c)
85. (c) 86. (a) 87. (d)
88. (a) 89. (d) 90. (b)
91. (d) 92. (b) 93. (b)
94. (a) 95. (b) 96. (c)
97. (b) 98. (c) 99. (c)
100. (b)

La Excellence IAS 133 Website: www.laex.in/testprep


9052 29 29 29 / 9052 49 29 29 (Hyderabad)
9121 41 29 29 / 9121 44 29 29 (Bangalore)

Das könnte Ihnen auch gefallen